SlideShare une entreprise Scribd logo
1  sur  48
125) A patient with Bloom Syndrome is most likely to have which laboratory abnormalities:

A. Elevated IgE

B. Positive ANA

C. Thrombocytopenia

D. Macrocytic anemia

E. Decreased immunoglobulinsCorrect Choice
Bloom syndrome is an autosomal recessive disorder due to a mutation in the BLM gene which codes
for a DNA helicase. Patients have impaired DNA repair after UV exposure and increased
photosensitivity. Clinical features include photodistributed erythema, cheilitis, high-pitched voice,
hypogonadism, and increased risk for leukemia, lymphoma and GI adenocarcinoma. Laboratory
evaluation reveals decreased IgA, IgM and IgG leading to increased risk of respiratory infections


126) A patient with hypohydrosis and hyperpyrexia, anodontia, and sparse hair has which
syndrome:


A. Dyskeratosis congenita

B. Papillon Lefevre

C. Pachyonychia congenita

D. Hidrotic ectodermal dysplasia

E. Anhidrotic ectodermal dysplasiaCorrect Choice
Anhidrotic ectodermal dysplasia is an x-linked recessive disorder which presents with the triad of
hypohydrosis (or anhidrosis) with hyperpyrexia, anodontia (other dental findings include peg teeth,
molars with hooked cusps) and sparse hair. Patients tend to overheat


127) A mentally retarded patient is found to also have a malar rash and downward lens
displacement. Another clinical feature might include:


A. periodontitis

B. hypertrichosis

C. deep venous thrombosesCorrect Choice

D. multiple nevi

E. short stature
The above described patient has homocystinuria, an autosomal recessive disorder caused by a
deficiency in cystathionine beta-synthase, leading to an accumulation of homocysteine. Clinical
features include: malar flush, DVT’s, livedo reticularis, leg ulcers, downward lens displacement,
myopia, glaucoma, sparse fine hair, mental retardation, and musculoskeletal anomalies


128) Which type of epidermolysis bullosa is associated with mitten deformities of the hands?

A. Generalized atrophic benign epidermolysis bullosa (GABEB)

B. Dominant dystrophic

C. Weber-Cockayne

D. Recessive dystrophic Correct Choice

E. Herlitz type



                                                   47
The recessive dystrophic type of epidermolysis bullosa is associated with chronic hand ulcers
resulting in scarring that causes mitten deformities. These chronic scars often result in the
formation of fatal squamous cell carcinomas.


129) Nevoid basal carcinoma syndrome (Gorlin syndrome) is autosomal dominant transmitted
mutation of the patched gene. Symptoms include innumberable basal cell carcinomas, painful
odontogenic jaw keratocysts, palmoplantar pits, frontal bossing, bifid ribs and what other bony
abnormality?


A. Osteopoikilosis

B. Polyostotic fibrous dysplasia

C. Calcification of falx cerebriCorrect Choice

D. Sphenoid wing dysplasia

E. Stippled epiphyses
Calcification of falx cerebri is seen in Gorlin's syndrome. CHILD syndrome and chondrodysplasia
punctata both can exhibit stippled epiphyses. Polyostotic fibrous dysplasia is found in McCune-
Albright syndrome, osteopoikilosis in seen in Buschke-Ollendorf syndrome. Sphenoid wing dysplasia
is seen in neurofibromatosis type I


130) A double row of eyelashes is associated with:

A. Lymphedema-distichiasis syndromeCorrect Choice

B. Hunters syndrome

C. Cornelia de Lange syndrome

D. Rubinstein-Taybi syndrome

E. Russell-Silver syndrome
A double row of eyelashes is defined as distichiasis and is associated with the Lymphedema-
distichiasis syndrome. This syndrome is transmitted in an autosomal dominant fashion and is
related to a mutation in FOXC2. Findings include late onset lymphedema, distichiasis, corneal
irritation, ectropion, webbed neck and congenital heart defects. The remaining syndromes do not
include distichiasis as a feature.


131) Patients with Russell-Silver syndrome exhibit:

A. Broad thumbs

B. Clinodactyly of fifth fingerCorrect Choice

C. Peg teeth

D. Shortened 4th and 5th metacarpals

E. Osteopathia striata
Characteristic features of Russell-Silver include short stature, bony asymmetry, triangular facies,
clinodactyly of fifth finger, and precocious sexual development with cryptochordism/hypospadias.
Broad thumbs are seen in Rubinstein-Taybi and shortened fourth and fifth metacarpals are seen in
Turner syndrome. Osteopathia striata is characteristic of focal dermal hypoplasia


132) A child presents with macroglossia, exopthalmos and gigantism. He has a history of
omphalocele repair and has circular depressions on the rim of the posterior helices. Although this
syndrome is most often transmitted in a sporadic manner, 15% of cases have defects in which
gene?



                                                 48
A. FLT4

B. VHL

C. FOXC2

D. ATM

E. KIP2Correct Choice
KIP2 can be mutated in 15% of cases of Beckwith-Wiedemann Syndrome. KIP2 is a cyclin-
dependent kinase inhibitor gene which acts as a negative regulator of cell proliferation. These
children have an increase risk of Wilms' tumor and organomegaly. In addition to the circular ear
depression, they can also have a linear earlobe crease. The remaining genes are mutated in other
syndromes with vascular disorders as a component: ATM in ataxia telangectasia, VHL in Von Hippel-
Lindau, FLT4 in Hereditary lymphedema and FOXC2 in Lymphedema-distichiasis syndrome


133) Which of the following is defective in Ehlers-Danlos syndrome (EDS) with congenital adrenal
hyperplasia?


A. Lysyl hydroxylase

B. Tenascin-XCorrect Choice

C. Lysyl oxidase

D. All of these answers are correct

E. None of these answers are correct
Tenascin-X defects are associated with EDS and with congenital adrenal hyperplasia. The phenotype
is that of typical EDS with hyperextensible skin, hypermobile joints, and tissue fragility. Lysyl
oxidase is defective in X-linked EDS (type V) and Occipital horn syndrome (type IX). Lysyl
hydroxylase is defective in ocular-scoliotic (type VI) EDS


134) What is the most likely gene mutation in this individual who has migratory patches and fixed
plaques as depicted in this picture?


A. Connexin 26

B. Connexin 30.3 and 31Correct Choice

C. Keratins 1 and 10

D. Calcium ATPase 2C1

E. Loricrin
Erythrokeratodermis variabilis is an autosomal dominant disease characterized by transient patches
of geographic erythema and fixed hyperkeratotic plaques. The disease is due to gene defects in
connexin 30.3 and 31


135) The gene that is responsible for this disease also plays a pathogenic role in what other
disorder?


A. MastocytosisCorrect Choice

B. Sarcoidosis

C. Hereditary Angioedema

D. Mycosis Fungoides

E. Waardenburg Syndrome




                                                 49
Piebaldism is an autosomal dominant disease characterized by depigmented patches and a white
forelock. It is caused by a mutation in the c-kit proto-oncogene. Mutations in c-kit have also been
found in the peripheral cells of individuals with mastocytosis


136) Menkes kinky hair syndrome is associated most commonly with which of the following hair
abnormalities?


A. Pili triangulati et canaliculati

B. Pili tortiCorrect Choice

C. None of these options are correct

D. Trichorrhexis invaginata

E. Trichorrhexis nodosa
While pili torti is not exclusively found in Menkes kinky hair syndrome, this is the most common hair
abnormality found. Trichorrhexis nodosa can be also seen. Trichorrhexis invaginata is commonly
found in Netherton syndrome. Pili triangulati et canaliculati is the finding seen in Uncombable hair
syndrome.


137) Medulloblastomas are seen in which of the following syndromes?

A. Familial cylindromatosis

B. Nevoid basal cell carcinomas syndrome Correct Choice

C. Bazex syndrome

D. Nicolau-Balus syndrome

E. Birt-Hogg-Dube syndrome
Nevoid basal cell carcinomas syndrome (Gorlin syndrome) is an autosomal dominant condition
caused by mutations in the patched gene, which is involved in the hedgehog signaling pathway.
Patients develop innumerable BCCs, palmoplantar pits, painful odontogenic jaw keratocysts, frontal
bossing, bifid ribs, and calcification of the falx cerebri. Medulloblastomas and ovarian fibromas and
fibrosarcomas are associated with this condition


138) Which of the following is caused by a defect in a gap junction protein?

A. Hailey-hailey

B. Epidermolysis bullosa simplex

C. Bullous ichthyosis of siemens

D. Erythrokeratoderma variabilisCorrect Choice

E. Dyskeratosis congenita
Erythrokeratoderma variabilis is also known as Mendes da Costa Syndrome. It is caused by a defect
in connexin 31, a gap junction protein. EB simplex is caused by a mutation in keratins 5 & 14,
Hailey-Hailey is caused by a mutation in calcium transporters, dyskeratosis congenita is caused by a
defect in rRNA synthesis, and bullous icthyosis of Siemens is caused by mutations in keratin 2e


139) Which of the following syndromes is associated with markedly increased IgE levels, cold
abscesses and a characteristic coarse facies?


A. Chronic granulomatous disease




                                                 50
B. Wiskott-Aldrich syndrome

C. Job syndromeCorrect Choice

D. Leiner's disease

E. Severe combined immunodeficiency
Job syndrome or Hyper IgE syndrome is characterized by these findings. In addition, there is a
peripheral eosinophilia, eczematous dermatitis, frequent bronchitis and pneumonia, otitis media and
sinusitis. The other listed conditions are associated with immunodeficiency. Wiskott-Aldrich can
have eczematous dermatitis and all of these syndromes will have abnormal infections. They do not
have markedly increased levels of IgE like Job syndrome.


140) Pruritus is Sjogren Larsson syndrome is attributed to accumulation of what molecule(s)?

A. LeukotrieneCorrect Choice

B. Histamine

C. Bile salts

D. None of these answers are correct

E. All of these answers are correct
Accumulation of leukotriene B4 contributes to pruritus in Sjogren Larsson syndrome. Leukotriene
inhibitors may be helpful in controlling symptoms.


141) Which of the following syndromes is associated with tricholemmomas?

A. Rasmussen's

B. Bannayan-Riley-RuvacalbaCorrect Choice

C. Basex

D. Birt-Hogg-Dube

E. Brooke-Spiegler
Tricholemmomas are seen in Bannayan-Riley-Ruvacalba syndrome. This is an autosomal dominant
condition with macrocephaly, lipomas, hemangiomas, skeletal abnormalities, lymphangioma
circupscriptum, angiokeratomas, penile lentigines, acanthosis nigricans, and achrocrodons. There is
an increased incidence of breast, thyroid, and GI cancers. Tricholemmomas are also seen in Cowden
syndrome


142) Electron microscopic examination of a hair shaft reveals a canal-like groove along the shaft of
a triangular-shaped hair. This patient has:


A. Menke’s Kinky Hair syndrome

B. Spun-glass hairCorrect Choice

C. Trichothiodystrophy

D. Netherton’s syndrome

E. Bjornstad syndrome
Pili trianguli et canaliculi is also known as Spun-glass hair or Uncombable Hair Syndrome. Netherton
patients have trichorexis invaginata, Menke’s kinky hair patients have short, brittle sparse hairs,
“tiger tail” hair is seen in trichothiodystrophy, and pili torti is seen in bjornstad syndrome




                                                 51
143) A patient with coarse facies, broad nasal bridge, and extensive eczema might be expected to
have which abnormal laboratory value?


A. Hypertriglyceridemia

B. High Copper levels

C. High IgE levelsCorrect Choice

D. Low Zinc levels

E. Anemia
Patients’ with Hyper IgE Syndrome (Job syndrome) have impaired regulation of the IgE function and
increased susceptibility to infections. In addition to recurrent cutaneous infections including cold
abscesses, patients have widespread eczematous dermatitis, recurrent sinopulmonary infections
and typically have coarse facies with broad nasal bridge and a prominent nose


144) Reticulate pigmentation of skin, poikiloderma, alopecia, nail atrophy, premalignant oral
leukoplakia, and a Fanconi-type pancytopenia resulting in early death in addition to posterior fossa
malformations is characteristic of which of the following syndromes


A. Cockayne syndrome

B. Hoyeraal-Hreidarsson syndromeCorrect Choice

C. Bloom syndrome

D. Dyskeratosis congenita

E. Wiskott-Aldrich syndrome
Hoyeraal-Hreidarsson syndrome is has all of the features of dyskeratosis congenita plus posterior
fossa malformations. Bloom syndrome and Cockayne syndrome both have poikiloderma as features,
but do not include posterior fossa malformations as part of the syndrome. Wiskott-Aldrich syndrome
does not include any of these findings


145) Dyskeratosis congenita has all of the findings listed below EXCEPT:

A. reticulate pigmentation of skin/poikiloderma

B. alopecia

C. Mental retardationCorrect Choice

D. Is allelic to Hoyerall-Hreidarsson syndrome

E. Fanconi-type pancytopenia resulting in early death
Mental retardation is not part of the spectrum of disease of dyskeratosis congenita. The remaining
options are correct. Hoyerall-Hreidarsson syndrome has all of the findings of dyskeratosis congenita
with the addition of posterior fossa malformations


146) In ataxia telangectasia, the ATM gene is mutated. The product of the ATM gene is an enzyme
which:


A. participates in NF-kB activation

B. binds transforming growth factor beta protein

C. responds to DNA damage by phosphorylating key DNA repair substratesCorrect Choice

D. is the VEGF receptor 3




                                                   52
E. is an inhibitor of G1 cyclin/Cdk complexes
The ATM gene is a member of the phosphatidylinositol-3 family of proteins that respond to DNA
damage by phosphorylating key substrates involved in DNA repair according to OMIM. Defects in
endoglin (TGF beta3 binding protein) is deficient in Osler-Weber-Rendu syndrome. The VEGF
receptor 3 is defective in hereditary lymphedema. The NEMO gene is defective in Incontinentia
Pigmenti. Its product, NF-kB essential modulator (NEMO) is a key activator in the NF-kB pathway.
KIP2 is involved in AD Beckwith-Wiedemann syndrome and is an inhibitor of G1 cyclin/Cdk
complexes


147) Ichthyosis hystrix is characterized by the following gene defects?

A. keratins 5 and 14

B. keratins 1 and 10Correct Choice

C. keratins 1 and 9

D. none of these answers are correct

E. keratins 6 and 16
Ichthyosis hystrix or extensive epidermal nevi occurs secondary to a somatic mosaicism for keratins
1 and 10. If the mosaicism occurs on gonadal cells, offspring may have full blown epidermolytic
hyperkeratosis (EHK).


148) Mucosal malignancy is a complication of:

A. Oral hairy leukoplakia

B. Chronic candidiasis

C. Focal epithelial hyperplasia

D. White sponge nevus

E. Dyskeratosis congenitalCorrect Choice
Dyskeratosis congenital, also called Zinsser-Engman-Cole syndrome, is an X-linked recessive
genodermatosis caused by a mutation in DKC1. DKC1 encodes for dyskerin, which helps to maintain
telomeres through the pseudouridylation of rRNA. Features of this condition include reticulate gray-
brown hyperpigmentation, dystrophic nails, alopecia and Fanconi’s type pancytopenia. Patients may
have premalignant leukoplakia which should be followed closely


149) What condition is associated with this finding of inflammatory keratotic facial papules which
may result in scarring and atrophy?


A. Reiter's syndrome

B. Chloracne

C. Keratosis pilaris

D. Ulerythema ophryogenesCorrect Choice

E. Systemic lupus erythematosus
Ulerythema ophryogenes is a rare disorder that affects children and young adults. It is characterized
by keratosis pilaris atrophicans and loss of lateral third of eyebrow


150) Ichthyosis linearis circumflexa is one of the findings seen in the syndrome caused by which of
the following genes?




                                                 53
A. SLURP1

B. CYLD

C. GJB2

D. LMX1B

E. SPINK5Correct Choice
The SPINK5 gene encodes for LEKTI, a serine protease inhibitor important in the regulation of
proteolysis in epithelia formation and keratinocyte terminal differentiation, is mutated in Netherton’s
Syndrome. Other findings include: trichorrhexis invaginata (bamboo, ball and socket hair), atopic
dermatitis, and anaphylaxis from food allergy. SLURP1 is mutated in Mal de Maleda, GJB2 in
Vohwinkel’s syndrome, LMX1B in Nail-Patella syndrome, and CYLD in Familial Cylindromiasis


151) What is the mode of transmission for lamellar ichthyosis?

A. sporadic

B. X-linked dominant

C. autosomal dominant

D. autosomal recessiveCorrect Choice

E. X-linked recessive
Lamellar ichthyosis which is characterized by collodian membrane in newborns and platelike scale in
children and adults is an autosomal recessive syndrome. The gene defect is transglutaminase 1
(TGM1


152) Which of the following diseases is caused by a mutation in a gap junction protein?

A. Schopf-Schulz-Passarge syndrome

B. Vohwinkel syndrome (classic) Correct Choice

C. Striated PPK

D. Mal de Meleda

E. Vohwinkel syndrome (ichthyotic)
Classic Vohwinkel syndrome is caused by mutations in connexin 26, a gap junction protein.
Ichthyotic Vohwinkel syndrome is caused by mutations in loricrin and has ichthyosis but not
deafness


153) Which of the following are cutaneous features of Marfan syndrome?

A. Dermatofibrosis lenticularis and striae

B. Fat herniation and cutaneous atrophy

C. Striae and elastosis perforans serpiginosa Correct Choice

D. Loose skin and crumpled ears

E. Sclerodermoid changes and dyspigmentation
Marfan syndrome is an autosomal dominant disorder caused by mutations in fibrillin 1 and 2.
Patients have tall stature, arachnodactyly, pectus excavatum, high-arched palate, joint laxity,
ectopia lentis with upward dislocation, dilated aorta with rupture, mitral valve prolapse, striae, and
elastosis perforans serpiginosa




                                                  54
154) What gene defect would you expect to find in a child with white forelock, dystopia canthorum,
and upper limb abnormalities?


A. MITF

B. Endothelin-3

C. Pax3Correct Choice

D. SOX10

E. C-kit proto-oncogene
Waardenburg's syndrome is characterized by depigmented patches, white forelock, and deafness.
Both type 1 and 3 are caused by mutations in Pax3. Type 3 is also associated with limb
abnormalities. MITF and SOX10 defects are responsible for types 2 and 4 respectively. C-kit proto-
oncogene mutatios are seen in piebaldism


155) Palmoplantar keratoderma with deafness is caused by a defect in which gene?

A. unknown

B. Plakophilin

C. lysosomal papain like cysteine proteinase

D. SLURP-1

E. mitochondrial serine transferase RNACorrect Choice
Palmoplantar keratoderma with deafness is caused by a defect in mitochondrial serine transferase
RNA. A defect in SLURP-1 causes Mal de Meleda. A defect in plakophilin causes ectodermal dysplasia
with skin fragility. A defect in cathepsin C lysososomal papain like cysteine proteinase causes
Papillon LeFevre and Haim Munk.


156) A Puerto Rican woman is seen in clinic for a pruritic rash on her trunk. A punch biopsy is
performed. The biopsy site continues to bleed, with hematoma formation. The bleeding is eventually
controlled. On further exam, her skin and hair are light brown. She has a history of granulomatous
colitis. What it the most likely reason she had excess bleeding with a simple procedure?


A. Her platelets lack dense bodies, causing excess bleeding Correct Choice

B. Her Factor VIII levels are low

C. Her intrinsic factor is deficient

D. She is congentially deficient in platelets

E. None of the answers are correct
Platelets without dense bodies cause excess bleeding in Hermansky-Pudlak syndrome. Other
features of this condition include oculocutaneous albinism, ceroid lysosomal storage disease
resulting in pulmonary fibrosis, granulomatous colitis


157) Which of the following syndromes are linked to a PTEN gene mutation?

A. Cowden

B. All of these options are correctCorrect Choice

C. Lhermitte-Duclos disease

D. Banayan-Zonana




                                                    55
E. Proteus
PTEN gene produces a phosphatase the regulates the cell-cycle and apoptosis, therefore acting as a
tumor suppressor gene. Tissues affected by this mutation are those with increased proliferation
such as epidermis, the oral and gastrointestinal mucosa, and the thyroid and breast epithelium. All
of the syndromes listed have mutations in PTEN.


158) The combination of painful palmoplantar keratoderma and pseudoherpetic keratitis is
characteristic of which of the following syndromes?


A. Naxos syndrome

B. Howel-Evans syndrome

C. Richner-Hanhart syndrome Correct Choice

D. Schopf-Schulz-Passarge syndrome

E. Vohwinkel syndrome
Richner-Hanhart syndrome (tyrosenemia type II) is an autosomal recessive disorder caused by a
deficiency in hepatic tyrosine aminotransferase. This disease is characterized by painful PPK,
pseudoherpetic keratitis and blindness. Treatment is low-tyrosine/phenylalanine diet


159) Epidermal nevus syndromes inheritance pattern is:

A. Autosomal dominant

B. X-linked dominant

C. X-linked recessive

D. Autosomal recessive

E. sporadicCorrect Choice
Epidermal nevus syndrome has many findings, including: sporadic inheritance, nevus unius lateris,
capillary malformations, café au lait macules, mantal retardation and seizures, deafness,
hemiparesis, hemihypertrophy of limbs, kyphoscoliosis and rare solid tumors. A biopsy is helpful to
rule out epidermolytic hyperkeratosis. If positive, the patient’s offspring are at risk for generalized
epidermolytic hyperkeratosis


160) Ichthyosis bullosa of Siemens is a condition characterized by fragile blisters at birth,
hyperkeratotic plaques on elbows/knees later in life, and a gene mutation in:


A. Keratin 1/10

B. Keratin 6a/16

C. None of these options are correct

D. Keratin 5/14

E. Keratin 2eCorrect Choice
Keratin 2e is mutated in ichythosis bullosa of Siemens. Keratin 5 and 14 are defective in
epidermolysis bullosa simplex, 6a/16 in Pachyonychia congenital type I (Jadassohn-Lewandowsky),
1/10 in epidermolytic hyperkeratosis and Unna-Thost PPK


161) Spastic ditetraplegia is associated with which of the following disorders?

A. Sjogren-Larsson syndrome Correct Choice




                                                  56
B. X-linked ichthyosis

C. Lamellar ichthyosis

D. Refsum syndrome

E. KID syndrome
Sjogren-Larsson syndrome is an autosomal recessive disorder caused by mutations in the fatty
aldehyde oxidoreductase/alcohol dehydrogenase gene. This disorder is characterized by ichthyosis,
spastic ditetraplegia, mental retardation, epilepsy, glistening dot retinal pigmentation, and dental
enamel dysplasia
162) A sporadic syndrome affecting transcriptional coactivator CREB-binding protein is:

A. Nonne-Milroy disease

B. Rubinstein-Taybi syndromeCorrect Choice

C. Cornelia de Lange syndrome

D. Maffucci syndrome

E. Blue rubber bleb nevus syndrome
Rubinstein-Taybi syndrome is caused by a sporadically transmitted defect in transcriptional
coactivator CREB-binding protein. This gene is responsible for encoding a nuclear protein which acts
as a co-activator of cAMP regulated gene expression. Findings of this syndrome include: capillary
malformation, short stature, broad thumbs, craniofacial abnormalities including beaked nose,
mental retardation, strabismus, congenital heart defects and cryptorchidism. The other listed
conditions are not related to this defect
163) The arylsulfatase C gene is mutated in which disease?

A. Naxos syndrome

B. X-linked ichthyosisCorrect Choice

C. Griscelli syndrome

D. Haim-Munk syndrome

E. Refsum syndrome
Arylsulfatase C is also known as steroid sulfatase and is mutated in X-linked ichthyosis. This
condition is inherited in a X-linked recessive pattern. Clinical findings include: brown scale sparing
palms, soles and flexures, comma-shaped corneal opacities, failure of labor progression and
cryptorchidism. It is also mutated in X-linked recessive type chondrodysplasia punctata
164) In one variant of epidermolysis bullosa simplex, those affected have muscular dystrophy in
addition to the skin findings. Which protein mutation has been linked to this finding?


A. plectinCorrect Choice

B. plakophilin

C. plakoglobin

D. desmocollin

E. desmoglein
A plectin mutation is linked with this subtype of epidermolysis bullosa simplex.
165) A 3 year old boy presents with the findings seen in the image. He also has thrombocytopenia
with purpura and a history of recurrent pyogenic bacterial infections. What is the most likely
diagnosis in this child?


A. Hyper-IgE syndrome



                                                  57
B. Severe combined immunodeficiency

C. Wiskott-Aldrich syndromeCorrect Choice

D. Leiner syndrome

E. Chronic Granulomatous disease
The findings described are consistent with Wiskott-Aldrich syndrome. The characteristic triad can be
simplified to the 3 P's - Pruritus (atopic dermatitis), Purpura (thrombocytopenia leading to purpura
and other bleeding) and pyogenic infections. The remaining options are related to Wiskott-Aldrich in
that they all have immunodeficiency as a feature, but not the same spectrum of disease



166) Pseudoxanthoma elasticum is caused by mutations in which of the following genes?

A. Fibrillin 1

B. Collagen 5

C. ABCC6 gene Correct Choice

D. Lysyl oxidase

E. Elastin gene
Pseudoxanthoma elasticum is caused by mutations in the ABCC6 gene, which is an ATP-using cell
transporter. Elastin and lysyl oxidase mutations cause cutis laxa, fibrillin 1 mutations cause Marfan
syndrome, and collagen 5 mutations cause Ehlers-Danlos syndrome



167) A 4 year old boy presents with 2 soft, dark-blue, compressible nodules on her extremities. His
mother has noted that these lesions have increased sweating and that they were present at birth.
No one else in the family has had similar skin lesions. What step is indicated first to help determine
the diagnosis?


A. Immediate referral to a gastroenterologist

B. Biopsy of a skin lesion

C. MRI of the abdomen

D. Stool guiacCorrect Choice

E. CBC
Blue rubber bleb nevus syndrome is described above. There are multiple venous malformations on
the extremities and trunk, often present at birth to early childhood. The number of these lesions
increase with age. The lesions may have increased sweating and can be combined with lymphatic-
venous malformations. Skin lesions can be a clue to gastrointestinal venous malformations which
can lead to secondary bleeding and anemia. The most reasonable screening test to determine if the
patient has GI hemorrhage is a stool guiac. An MRI or complete blood count can be helpful, but are
not the best test to start with. A skin biopsy is not indicated. If there is GI blood loss, evaluation by
a gastroenterologist is useful



168) A patient presents with starfish keratoses, pseudoainhum, honeycombed PPK, and generalized
ichthyosis. What is the most likely genetic defect?


A. Connexin 31

B. Connexin 26




                                                   58
C. LoricrinCorrect Choice

D. Connexin 30

E. Connexin 33
The patient has Vohwinkel syndrome. This is an autosomal dominant syndrome with 2 clinical
variants. The variant described above with generalized ichthyosis is due to a loricrin mutation. In
the classic form with nonprogressive hearing loss connexin 26 is mutated



169) The best screening test for hemochromatosis is:

A. Complete Blood Count

B. FerritinCorrect Choice

C. Hematocrit

D. Copper levels

E. Transferrin
The ferritin is the best screening test for hemochromatosis, an autosomal disease of increased
intestinal iron absorption. The other tests are not the best method for screening



170) A 32 year old woman is 5 weeks pregnant and is diagnosed with hyperthyroidism. Her doctor
gives her a prescription for on methimazole 10 mg PO tid. Which of the following fetal abnormalities
could be caused by this exposure?


A. Dermoid cyst

B. Aplasia cutis congenitaCorrect Choice

C. Spina bifida

D. Encephalocele

E. Meningocele
In-utero methimazole exposures has been linked to aplasia cutis congenita and should not be used
in pregnant women. The FDA pregnancy class is D. The other listed options are not linked with
maternal methimazole usage



171) Ichthyosiform erythroderma in lines of Blaschko, follicular atrophoderma, and stippled
epiphyses are characteristic of which of the following disorders?


A. Refsum syndrome

B. Netherton syndrome

C. Erythrokeratodermia variabilis

D. Sjogren-Larsson syndrome

E. Chondrodysplasia punctata Correct Choice
Chondrodysplasia punctata is a peroxisomal disorder caused by mutations in several genes that is
associated with ichthyosiform erythroderma in lines of Blaschko, follicular atrophoderma, and
stippled epiphyses




                                                 59
172) Yellow-brown depositions in Descemet's membrane of the corneas is diagnostic for:

A. Neurofibromatosis

B. Hemochromatosis

C. Hepatolenticular degenerationCorrect Choice

D. Pseudoxanthoma elasticum

E. Diabetes mellitus
Hepatolenticular degeneration or Wilson Disease is caused by a defect in biliary excretion of copper
leading to accumulation in the liver, brain, cornea, pretibial hyperpigmentation, hepatomegaly and
cirrhosis. The Kayser-Fleischer ring is the yellow-brown copper deposition in Descemet's membrane
of the cornea. Other findings include ataxia, dysarthria and dementia. Hemochromatosis does not
have an eye finding. Neurofibromatosis has Lisch nodules and Pseudoxanthoma elasticum has
angioid streaks of the retina



173) The gene defect in Griscelli Syndrome is:

A. None of these answers are correct

B. Myosin Va or Rab27aCorrect Choice

C. LYST or CHS1

D. P gene

E. TRP1
Myosin Va or Rab27a are defective in Griscelli syndrome, an AR syndrome with mild albinism,
pancytopenia, immunodeficiency, neurologic symptoms and an accelerated phase similar to
Chediak-Higashi syndrome. LYST/CHS1 is defective in Chediak-Higashi syndrome. The P-gene is
mutated in oculocutaneous albinism type 2 and TRP in oculocutaneous albinism type 3



174) Mutation in lamin A (nuclear envelope protein) has been found in:

A. Peutz-Jeghers syndrome

B. Albright’s syndrome

C. Marfan syndrome

D. Buschek-Ollendorf syndrome

E. Progeria (Hutchinson-Gilford) Correct Choice
Progeria (Hutchinson-Gilford syndrome) is a sporadic condition characterized by lipoatrophy,
sclerodermoid skin, alopecia, nail atrophy, craniomegaly with small face, muscle/bone wasting, and
severe premature atherosclerosis resulting in early death. Recent studies have shown that
mutations in nuclear envelope protein lamin A is associated with progeria



175) The characteristic dental findings in patients with tuberous sclerosis are:

A. Odontogenic cysts

B. Retention of primary teeth

C. Peg teeth



                                                  60
D. Anodontia

E. Enamel pitsCorrect Choice
Enamel pits are the characteristic dental findings in tuberous sclerosis. Peg teeth are found in
incontinentia pigmenti and anhidrotic ectodermal dysplasia. Anodontia is found in hypomelanosis of
ito and incontinentia pigmenti. Odontogenic cysts are seen in Gorlin syndrome, and retention of
primary teeth is characteristic of Job syndrome



176) Urticaria pigmentosa is linked to a defect in the c-kit protooncogene. What autosomal
dominant skin disease also has been linked to this defect?


A. PiebaldismCorrect Choice

B. Hypomelanosis of Ito

C. Incontinentia pigmenti

D. Waardenburg syndrome

E. Hermansky-Pudlak syndrome
Piebaldism is linked to a defect in the c-kit protooncogene. Hypomelanosis of Ito has whorled
hypopigmentation, occasional CNS defects, scoliosis and anodontia, a sporadic mutation.
Waardenburg syndrome has 4 types, 1-3 are AD, 4 is AR. Type 1 & 3 have defects in PAX-3, 2 in
MITF, and 4 in Sox10, endothelin-3 ligand or receptor genes. Hermansky-Pudlak syndrome is AR
and most commonly linked to defects in HPS, a lysosomal transport protein and AP3B1, a protein
important in endocytic/exocytic sorting. Incontinentia pigmenti is an X-linked dominant syndrome
with a defect in the NEMO gene



177) Which of the following pairs of diseases are caused by mutations in DNA helicases?

A. Xeroderma pigmentosum and Cockayne syndrome

B. Muir-Torre syndrome and Birt-Hogg-Dube syndrome

C. Cowden syndrome and Bannayan-Riley-Ruvalcaba syndrome

D. Dyskeratosis congenita and Peutz-Jeghers syndrome

E. Bloom syndrome and Rothmund-Thompson syndrome Correct Choice
Bloom Syndrome and Rothmund-Thompson syndromes are caused by autosomal recessive
mutations in DNA helicases. The common features include photodistributed poikiloderma. Patients
with Bloom syndrome also demonstrate facial dysmorphism, hypogammaglobulinemia with
recurrent respiratory and gastrointestinal tract infections, hypogonadism, leukemias and
lymphomas, gastrointestinal adenocarcinomas, and oral/esophageal SCCs. Patients with Rothmund-
Thompson syndrome develop premalignant acral keratoses, alopecia, nail dystrophy, cataracts,
hypogonadism, and occasional solid tumors



178) Premature aging seen in Cockayne’s syndrome is caused by a mutation in which gene?

A. ATM gene

B. DNA helicase gene Correct Choice

C. NEMO gene

D. Patched gene




                                                61
E. Transglutaminase
Cockayne’s syndrome is caused by a mutation in a DNA helicase gene, CSA or ERCC8. The condition
is defined by growth deficiency, premature aging, and pigmentary retinal degeneration. 75% of
patient have photosensitive eruptions and severe cataracts before the age of 3 are associated with
poor prognosi



179) Which of the following metals is deficient in the serum of patients with Menkes kinky hair
syndrome?


A. Biotin

B. Selenium

C. Zinc

D. Iron

E. CopperCorrect Choice
Menkes kinky hair syndrome is transmitted in an X-linked recessive manner and is caused by a
mutation in ATP7A, an ATP-dependent copper tranporter. This defect results in low serum levels of
copper. These individuals will have hair abnormalities such as sparse, hypopigmented brittle hair,
eyelashes and eyebrows, lax skin, a "cupid's bow" upper lip, CNS progressive deterioration,
seizures, skeletal abnormalities and tortuous arteries. The other listed items are not associated with
Menkes syndrome



180) Which of the following is NOT a complication of Kasabach-Merritt Syndrome:

A. Thrombocytopenia

B. Disseminated intravascular coagulation

C. Gastrointestinal bleeding

D. CHF

E. AtaxiaCorrect Choice
Kasabach-Merritt Syndrome results from platelet trapping due to the presence of a single or
multiple large hemangiomas. Hematologic complications include thrombocytopenia,
microangiopathic hemolytic anemia, DIC, and acute hemorrhage. The presence of large
hemangiomas leads to high output failure (CHF) and can also compress on surrounding structures



181) Patients with progeria typically die of which of the following conditions?

A. Atherosclerotic heart disease Correct Choice

B. Metastatic carcinoma

C. Progressive systemic sclerosis

D. Infection

E. Nail atrophy
Progeria (Hutchinson-Gilford syndrome) is a sporadic condition characterized by lipoatrophy,
sclerodermoid skin, alopecia, nail atrophy, craniomegaly with small face, muscle/bone wasting, and
severe premature atherosclerosis resulting in early death




                                                  62
182) On cutaneous exam, angiokeratoma corporis diffusum is characteristic of which of the
following conditions?


A. Fabry disease

B. Sialodosis

C. Fucosidosis

D. All of these options are correctCorrect Choice

E. None of these options are correct
Findings of angiokeratoma corporis diffusum are found in all three listed conditions. They cannot by
distinguished by skin exam



183) The histologic image shown in this picture is associated with which of the following
syndromes?


A. Dyskeratosis congenita

B. Gorlin syndrome

C. Banayan-Riley-Ruvalcaba syndrome

D. Chediak-Higashi syndrome

E. Multiple hamartoma syndromeCorrect Choice
The histologic picture shown is that of a sclerotic fibroma with the characteristic storiform/whorled
pattern of collagen. The lesion has low cellularity with spindle cells. Banayan-Riley-Ruvalcaba
syndrome shares the same gene defect as Multiple hamartoma syndrome (Cowden syndrome) but is
not associated with sclerotic fibromas. Dyskeratosis congenita is not associated with sclerotic
fibromas



184) A patient with Crowe’s sign and an optic glioma has which of the following disorders?

A. Tuberous sclerosis

B. Neurofibromatosis II

C. Watson syndrome

D. Lester iris syndrome

E. Neurofibromatosis I Correct Choice
The diagnostic criteria for neurofibromatosis I include meeting 2 or more of the following 7 criteria:
(1) >5 café au lait macules (CALMs) that are >5mm in a prepubertal person or >15 mm in a
postpubertal person, (2) >1 neurofibroma or 1 plexiform neurofibroma, (3) axillary/inguinal
freckling (Crowe’s sign), (4) optic glioma, (5) >1 Lisch nodule (iris hamartoma), (6) sphenoid
dysplasia, (7) 1st degree relative with neurofibromatosis I



185) The most common cutaneous neoplasm associated with Muir Torre Syndrome is:

A. Basal cell carcinoma with sebaceous differentiation

B. Sebaceous adenomaCorrect Choice




                                                    63
C. Keratoacanthoma

D. Sebaceous carcinoma

E. Squamous cell carcinomas
Muir-Torre syndrome is an autosomal dominant disorder due to a DNA mismatch repair gene
(MSH2). Patients present with sebaceous tumors, with adenomas being the most common
(sebaceous carcinomas next common) and associated visceral malignancies such as colon cancer
(most common).



186) The finding of 'maltese crosses' in the urine is characteristic of which of the following
conditions?


A. Fabry diseaseCorrect Choice

B. Alkaptonuria

C. Neimann-Pick disease

D. Hunter syndrome

E. Gaucher disease
The 'maltese cross' finding in urine is characteristic of Fabry disease. Alkaptonuria will show dark
urine with a pH > 7.0. There are no urinary findings in Hunter syndrome, Gaucher or Neimann-Pick
disease



187) The gene PTEN (phosphatase and tensin homolog deleted on chromosome ten) is implicated in
which of the following syndromes?


A. Cowden

B. None of the options are correct

C. Banayan-Riley-Ruvalcaba

D. All of the options are correctCorrect Choice

E. Proteus
Mutations in the PTEN gene are implicated in all three syndromes listed. There are conflicting
reports in the literature regarding the linkage of Proteus and PTEN mutations. For further details,
see OMIM #176920



188) Which cutaneous finding is seen in patients with phenylketonuria?

A. Pigment dilution of hair and skinCorrect Choice

B. Ichthyosis

C. Phyrnoderma

D. Angular stomatitis

E. Erosive diaper dermatitis
Phenylketonuria is an autosomal recessive condition caused by a mutation in the gene coding for
phenylalanine hydroxylase. Defect in this enzyme results in accumulation of phenylalanine and its
metabolites. Increased phenylalanine has toxic effects on the central nervous system in addition to
competitively inhibiting tyrosine in melanogenesis. Inhibition of melanogenesis results in pigmentary



                                                  64
dilution of the hair and skin. Other features of this condition include a predisposition to eczema,
sclerodermoid changes of the skin, urine that has a distinctive "mousy" odor, psychomotor delay,
mental retardation, seizures and hyperreflexia. A low-phenylalanine diet instituted early on can
prevent these manifestations of the disease. The morbidity of phenylketonuria has improved since
the advent of routine neonatal screening for this condition



189) All of the following disorders are exacerbated by UV radiation except:

A. Rothmund-Thomopson syndrome

B. Hartnup’s disease

C. Bloom syndrome

D. Cockayne syndrome

E. Refsum syndromeCorrect Choice
Refsum’s syndrome is an autosomal recessive disorder caused by mutations in phytanoyl-CoA
hydroxylase. Clinically, patients have mild icthyosis, cerebellar ataxia, polyneuropathy, salt and
pepper retinitis pigmentosa, sensorineural deafness, and arrhythmias with heart block. They are not
overly sensitive to UV radiation



190) Angioid streaks on retinal exam are characteristic of which of the following syndromes?

A. Pseudoxanthoma elasticumCorrect Choice

B. Choroid malformations

C. Salt & pepper retinitis pigmentosa

D. Eyelid papillomas

E. Lester iris
Angioid streaks are characteristic of pseudoxanthoma elasticum. They are caused by rupture of
Bruch's membrane of the choroid. Choroid malformations are found in Sturge-Weber syndrome,
eyelid papillomas in xeroderma pigmentosum, Lester iris in Nail-patella syndrome and salt & pepper
retinitis pigmentosa in Refsum syndrome



191) Which one of the following is the most common oncogenic virus in patients with
epidermodysplasia verruciformis?


A. HPV-16

B. HPV-5 Correct Choice

C. HPV-13

D. HPV-8

E. HPV-33
Epidermodysplasia verruciformis is a rare autosomal recessive disorder in which an impaired cellular
immunity allows widespread infection with certain subtypes of the human papilloma virus (HPV).
Some of these lesions have a tendency for malignant transformation, most commonly those verruca
caused by HPV type 5




                                                 65
192) Crumpled ears are associated with which disorder?

A. Marfan syndrome

B. Buschke-Ollendorf syndrome

C. Cutis laxa

D. Ehlers-Danlos Syndrome

E. Congenital contractural arachnodactyly Correct Choice
Congenital contractural arachnodactyly is an autosomal dominant disorder caused by mutations in
fibrillin 2. Affected patients have long limbs, arachnodactyly, scoliosis, and crumpled ears



193) Which of the following conditions is inherited in an X-linked dominant (XD) manner?

A. Focal dermal hypoplasia

B. Chodrodysplasia punctata

C. None of the answers are correct

D. All of the answers are correctCorrect Choice

E. CHILD Syndrome
All of the syndromes listed are XD. Other XD syndromes are: Incontinentia pigmenti and Bazex
syndrome



194) What is the characteristic radiographic finding in type I Gaucher disease?

A. Osteopoikilosis

B. Melorheostosis

C. Enchondromas

D. Ehrlenmeyer flask deformityCorrect Choice

E. Supernumerary vertebrae with extra ribs
The Ehrlenmeyer flask deformity is found in the femoral midshaft as well as aseptic necrosis of the
femoral head and widening of the distal femur. Endochondromas are seen in Maffucci syndrome,
Osteopoikilosis in Buschke-Ollendorf syndrome, Melorheostosis (linear hyperostosis under affected
skin) in linear scleroderma and supernumerary vertebrae with extra ribs in incontinentia pigmenti



195) A child presents with pretibial hyperpigmentation, ataxia, decreased motor coordination,
cirrhosis, and decreased motor coordination. The physical exam which would reveal the most
specific finding for this disease is:


A. Slit-lamp eye examCorrect Choice

B. Renal ultrasound

C. Hearing test

D. Colonoscopy

E. EKG




                                                  66
Wilson’s disease (also known as hepatolenticular degeneration) is an autosomal recessive disorder
result in defective biliary excretion of copper, leading to copper accumulation in the liver, brain and
cornea. Clinical features include hepatomegaly, cirrhosis, ataxia, dysarthria, decreased motor
coordination, pretibial hyperpigmentation, blue lunulae, and copper deposition in the cornea—
Kayser-Fleisher ring, which can be diagnosed using a slit-lamp



196) Eyelid string of pearls are seen in which of the following conditions?

A. Hutchinson-Gilford syndrome

B. Lipoid proteinosis Correct Choice

C. Focal dermal hypoplasia

D. Gaucher’s disease

E. Beare-Stevenson cutis gyrata syndrome
Lipoid proteinosis is an autosomal recessive condition characterized by yellow papules on the face
and oropharynx, eyelid string of pearls, hoarse voice, verrucous nodules of elbows and knees, and
bean-shaped temporal and hippocampal calcification with occasional seizures. Histologically, there
are PAS+ deposits in the affected tissue



197) What finding is seen on brain imaging of patients with Papillon-Lefevre Syndrome?

A. calcification of the falx cerebri

B. calcification of the hippocampus

C. tram track calcifications

D. calcification of the duraCorrect Choice

E. agenesis of the corpus callosum
Pappilon Lefevre Syndrome is an autosomal recessive syndrome characterized by transgredient PPK
and periodontitis. There is a defect in cathepsin C. One sees dural calcification at the tentorium and
choroid attachments. Tram track calcifications are seen in STurge-WEber. CAlcification of the falx
cerebri and agenesis of the corpus callosum is seen in basal cell nevus syndrome. Hippocampal
calcification is seen in lipoid proteinosi



198) A patient with Klinefelter Syndrome may be expected to experience which of the following:

A. Recurrent leg ulcersCorrect Choice

B. Scarring alopecia

C. Recurrent pulmonary infections

D. Gastroesophageal reflux

E. Pulmonary valve stenosis
Klinefelter syndrome results from nondisjunction during meiosis, leading to the XXY genotype.
Patients are characteristically tall (long lower extremities) with scant body and pubic hair. Klinefelter
patients have numerous varicosities predisposing them to recurrent leg ulcers



199) Which of the following is the first symptom of ataxia telangiectasias?


                                                   67
A. Facial telangiectases

B. Conjunctival telangictases

C. Breast cancer

D. Hematologic malgignancy

E. Cerebellar ataxia Correct Choice
Ataxia Telangiectasia (Louis-Bar syndrome) is an autosomal recessive disorder usually caused by
mutations in the ATM gene, which is a chromosomal strand break repair enzyme. Cerebellar ataxia
is the first sign, followed by telangiectases of the conjunctiva and skin. Thymic hypoplasia
predisposes to increased infections. There is increased sensitivity to ionizing radiation resulting in
hematologic and solid tumors. Female carriers have increased risk of breast cancer



200) Which type of epidermolysis bullosa simplex is associated with early death?

A. Weber-Cockayne

B. Dowling-Maera Correct Choice

C. Non-Herlitz variant

D. Ogna variant

E. Generalized (Koebner)
The Dowling-Maera variant of epidermolysis bullosa simplex is associated with widespread bullae,
significant mucous membrane and laryngeal/esophageal involvement, nail dystrophy, and early
death



201) Patients with Darier’s disease are at increased risk for:

A. Melanoma

B. Basal cell carcinoma

C. Lipid abnormalities

D. Kaposi’s varicelliform eruptionCorrect Choice

E. Decreased life span
Kaposi’s varicelliform eruption is the condition in which viral infection occurs in a patient with a pre-
existing chronic dermatitis. Darier’s disease is an autosomal dominant genodermatosis caused by a
mutation in ATP2A2 which encodes SERCA2. Cutaneous manifestations of warty, hyperkeratotic
papules in a seborrheic dermatitis, which may be infected with HSV or bacteria



202) A patient with port wine stain on a lower extremity, hemihypertrophy of the limb and
lymphatic and deep venouse insufficiency of the affected limb would be considered to have Klippel-
Trenaunay-Weber syndrome. What additional feature would need to be present to define the patient
as having Parkes-Weber syndrome?


A. Macroglossia

B. Multiple cafe-au-lait macules

C. Arteriovenous fistulasCorrect Choice




                                                   68
D. Cutis marmorata

E. Distichiasis
Parkes-Weber syndrome has the additional feature of arteriovenous fistulas. The remaining features
are not part of these syndromes



203) What is the first sign seen in children with ataxia telangectasia syndrome?

A. nystagmus

B. cutaneous and bulbar conjunctival telangectasias Correct Choice

C. recurrent viral or bacterial infections

D. oculomotor apraxia

E. cerebellar ataxia
Telangectasias are the first sign of ataxia telangectasia syndrome (Louis-Bar). Other findings other
than those listed above include: increased risk of lymphoma or breast carcinoma in heterozygotes,
granulomas, cafe au lait macules. IgA, IgG2 and IgE will be decreased or absent. There is an
increased sensitivity to ionizing radiation



204) A patient with this autosomal recessive disorder caused by a defect in helicase is an increased
risk for which malignancy?


A. Medullary thyroid carcinoma

B. Renal cell carcinoma

C. Acute leukemiaCorrect Choice

D. Squamous cell carcinoma of the lung

E. Prostate carcinoma
Bloom's syndrome is an autosomal recessive disorder caused by a mutation in DNA helicase. It is
characterized by photodistributed erythema in a butterfly distribution, malar hypoplasia with a
prominent nose, high pitched voice, and an increased risk for malignancy (acute leukemia,
lymphoma, and GI adenocarcinoma



205) In biopsies from blisters in patients with junctional epidermolysis bullosa, the split is found in
the:


A. Basal cell layer of the epidermis

B. Lamina densa

C. Lamina lucidaCorrect Choice

D. Squamous cell layer of the epidermis

E. None of the answers are correct
The split seen in junctional epidermolysis bullosa is in the lamina lucida. The other locations can be
involved in blistering disease, but not junctional epidermolysis bullosa




                                                  69
206) The NEMO gene is defective in Bloch-Sulzberger syndrome. What other syndrome has been
linked with defects in the NEMO gene?


A. Hypomelanosis of Ito

B. Tuberous sclerosis

C. Piebaldism

D. Waardenburg syndrome

E. Hypohidrotic ectodermal dysplasia with immune deficiency Correct Choice
Hypohidrotic ectodermal dysplasia with immune deficiency, is caused by mutations in the NEMO
(IKK-gamma gene). As opposed to the X-linked dominant inheritance of Bloch-Sulzberger syndrome
(incontinentia pigmenti), this is a X-linked recessive disorder. Hypomelanosis of Ito is sporadically
inherited and is not linked with a gene defect. Tuberous sclerosis is autosomal dominant and has
been linked to defects in tuberin and hamartin tumor suppressor genes. Waardenburg syndrome
has four subtypes, linked with the PAX-3, MITF and SOX10/endothelin-3 receptor genes. Piebaldism
is linked to defects in the c-kit protooncogene



207) What is the gene defect in harlequin fetus?

A. transglutaminase

B. none of these answers are correct

C. ABCC6

D. steroid sulfatse

E. ABCA12Correct Choice
Harlequin fetus is an autosomal recessive disorder. The gene defect is ABCA12



208) Regarding the inheritance of Ehlers-Danlos syndrome, which subtype is inherited in an X-
linked recessive manner?


A. The type with the lysyl oxidase deficiency Correct Choice

B. The type with a collagen 5 defect

C. The type with the lysyl hydroxylase deficiency

D. The type with a defect in procollagen aminopeptidase

E. The type with a collagen 3 defect
Lysyl oxidase deficiency is related to type V or X-linked Ehlers-Danlos syndrome (EDS). Type IX,
also has x-linked recessive inheritance with mild symptoms of EDS with occipital exostoses and
hernias. Lysyl hydroxylase deficiency is seen in type VI EDS, linked with severe kyphoscoliosis,
retinal detachment and other eye abnormalities. Collagen V deficiencies are seen in type I (Gravis)
and type II (Mitis) EDS, associated with skin elasticity, gaping wounds, hypermobile joints, Gorlin’s
sign, blue sclera and mitral valve prolapse



209) Anodontia is a bone finding seen in which of the following conditions:

A. Tuberous sclerosis




                                                    70
B. Letterer-Siwe disease

C. Hyper-IgE syndrome

D. Jackson Sertoli syndrome

E. Hypomelanosis of ItoCorrect Choice
Hypomelanosis of Ito, or Incontinentia pigmenti achromians is a condition characterized by marble-
cake hypopigmentation, epilepsy, alopecia, scoliosis and mental/motor retardation. The
characteristic dental abnormality is anodontia. The remaining syndromes are not associated with
anodontia



210) Multiple cylindromas and trichoepitheliomas are associated with which of the following
syndromes?


A. Gardner

B. Cronkhite-Canada

C. Brook-SpieglerCorrect Choice

D. Cowdens

E. Nicolau-Balus
These findings are characteristic of Brook-Spiegler syndrome. Gardner syndrome is associated with
gastrointestinal carcinoma and adnexal neoplasms, Cronkhite-Canada syndrome, Nicolau-Balus and
Cowdens are not associated with cylindromas or trichoepitheliomas



211) Which of the following features is not associated with Cornelia de Lange Syndrome?

A. Normal intelligenceCorrect Choice

B. Fifth finger clinodactyly

C. Recurrent lung infections

D. Cryptorchidism

E. Characteristic facies with downturned mouth, hirsutism, synophrys, trichomegaly, anteverted
nostrils, long philtrum and low set ears
Children with Cornelia de Lange are usually severly retarded with an IQ <35. In addition to the
features listed above, other features include cutis marmorata, hypoplastic nipples and umbilicus,
low-pitched cry in infancy and congenital heart defects. While most cases are inherited in a sporadic
manner, those cases which are familial are thought to be autosomal dominant and associated with
the NIPBL (nipped-beta-like) gene. Prognosis is poor with premature death often secondary to
sspiration or recurrent pulmonary infection



212) Erythematous keratotic plaques of KID Syndrome most commonly occur in which location?

A. Chest

B. Neck

C. Back

D. FaceCorrect Choice




                                                 71
E. Abdomen
The erythematous keratotic plaques of KID Syndome occur on face, extremities > trunk. KID
Syndrome is characterized by keratitis, icthyosis, and deafness. It is inherited in an autosomal
dominant fashion and caused by a mutation in the GJB2 gene which encodes connexin 26



213) Findings of milia, cylindromas and the condition shown in the pathology image are
characteristic of which of the following syndromes?


A. Brook-Spiegler syndrome

B. Gorlin's syndrome

C. Rombo syndrome

D. Rasmusen syndromeCorrect Choice

E. Familial cylindromatosis
Rasmusen syndrome is characterized by milia, trichoepitheliomas and cylindromas. The other
options have trichoepitheliomas as a feature of the syndrome, but not with the other listed findings



214) In addition to pheochromocytoma and medullary thyroid carcinoma and the skin finding in the
image in a patient with a marfanoid body habitus is associated with which of the following
syndromes?


A. MEN type I

B. Basex syndrome

C. Cowden syndrome

D. MEN type IIaCorrect Choice

E. MEN type IIb
The image show amyloidosis, which is found in addition to pheochromocytoma, medullary thyroid
carcinoma and parathyroid abnormalities in Sipple syndrome or MEN type IIa. Cowden disease is
not associated with any of the listed findings.



215) Argininosuccinic aciduria is associated with a hair abnormality shown in the image. Which of
the following hair finding is seen in these patients?


A. Trichorrhexis nodosaCorrect Choice

B. Trichoschisis

C. Trichorrhexis invaginata

D. Pili torti

E. None of these options are correct
Trichorrhexis nodosa is shown in the figure and is found in ~50% of affected patients with
argininosuccinic aciduria. Affected individuals will have short, broken scalp hairs, often increased
with late onset disease. Usually these findings are increased in the occipital region of the scalp. The
remaining options are not found in affected individual



216) The most common cutaneous association with monilethrix is:


                                                   72
A. Atrophy

B. Eczema

C. Hypopigmentation

D. Keratosis PilarisCorrect Choice

E. Hyperpigmentation
Monilethrix is an autosomal dominant condition which, by definition, presents with “beaded” hear.
Clinically, patients present with short, sparse lusterless hair. Keratosis pilaris is the most common
associated feature



217) An infant with doughy, redundant skin and short sparse hairs is likely to show which features
on x-ray?


A. Osteopoikilosis

B. Sphenoid wing dysplasia

C. Stippled epiphyses

D. Periosteal thickening

E. Metaphyseal widening in the long bonesCorrect Choice
The patient described has Menkes Kinky Hair syndrome, an X-linked recessive disease due to a
defect in an intestinal copper transport protein. Clinical features include pili torti, short, brittle
“steel-wool” hair, and spare eyelashes and sparse broken eyebrows. The skin is often
hypopigmented with a soft, “doughy” consistency and redundancy. Musculoskeletal manifestations
include metaphyseal widening with spurs in the long bones



218) Familial macular and lichen amyloidosis is a feature of which of the following conditions?

A. Sipple syndrome Correct Choice

B. Peutz-Jeghers syndrome

C. Birt-Hogg-Dube syndrome

D. Dyskeratosis congenita

E. Marfan syndrome
Sipple syndrome (MEN 2a) is caused by autosomal dominant mutations in the ret protooncogene.
Patients develop parathyroid cancers, pheochromocytomas, and medullary cancer of the thyroid
gland. Familial macular and lichen amyloidosis is also a feature of this syndrome



219) Which of the following is a feature of Neurofibromatosis type II?

A. Lisch nodules

B. Optic gliomas

C. Juvenile posterior subcapsular lenticular opacities Correct Choice

D. Congenital hypertrophy of the retinal pigment epithelium

E. Lester iris




                                                  73
Neurofibromatosis type II is an autosomal dominant disorder caused by mutations in schwannomin/
merlin. Clinical features include cutaneous schwannomas and neurofibromas, bilateral vestibular
schwannomas, and juvenile posterior subcapsular lenticular opacities



220) Findings of dysplastic nevi and melanoma inherited in an autosomal dominant fashion is linked
with which of the following?


A. Pancreatic malignancyCorrect Choice

B. Renal cell carcinoma

C. Breast malignancy

D. Thyroid malignancy

E. Colon carcinoma
Familial dysplastic nevi/melanoma syndrome is linked with increased risk of pancreatic cancers and
astrocytomas. There are no reported increased risks for the other types of cancers listed



221) Which gene is defective in Wiskott-Aldrich syndrome?

A. NCF2

B. NCF1

C. WASCorrect Choice

D. CYBB

E. CYBA
The WAS gene is defective in Wiskott-Aldrich syndrome. WAS is an Arp2/3 complex interacting
protein. The remaining options are genes related to Chronic Granulomatous Disease and are not
active in the pathogenesis of Wiskott-Aldrich syndrome



222) Refsum syndrome is due to a deficiency in phytanyl coenzyme A hydroxylase. Treatment for
this condition is:


A. Diet low in green vegetables, dairy and ruminant fats Correct Choice

B. No treatment is available at this time

C. Enzyme replacement

D. Avoid phenylalanine

E. Diet high in green vegetables, dairy and ruminant fats
Treatment is with a diet low in green vegetables, dairy and ruminant fats is the treatment of choice
for Refsum syndrome. Avoidance of specific amino acids is not helpful



223) Dermatofibrosis lenticularis disseminata and osteopoikilosis are findings seen with mutations
of which of the following genes?


A. Lysyl hydroxylase

B. Lysyl oxidase




                                                 74
C. LEMD3Correct Choice

D. Fibrillin 2

E. ABCC6
Buschke-Ollendorf syndrome is caused by a loss-of-function mutation in LEMD3



224) An infant girl of short stature and shortened 4th and 5th metacarpals is being evaluated for
coarctation of the aorta and horseshoe kidneys. Physical examination most likely reveals:


A. Arachnodactyly

B. Webbed neckCorrect Choice

C. Hemangioma

D. Giant congenital melanocytic nevus

E. Alopecia
Turner’s syndrome results from nondysjunction during gametogeneiss leading to the XO genotype.
Clinical features include short stature, redundant neck folds/webbed neck, multiple pigmented nevi,
low set hairline, triangular facies, low-set ears, ptosis, wide-set nipples, shortened 4th and 5th
metacarpals, hypoplasia of lymphatics, coarctation of the aorta, and horseshoe kidneys



225) Mutations in nuclear lamins are associated with which clinical feature?

A. Lipomatosis

B. Sarcomas

C. Aplasia cutis congenita

D. Lipodystrophy Correct Choice

E. Lymphedema
Bernardinelli-Seip congenital lipodystrophy and familial partial lipodystrophy are caused by
mutations in nuclear lamins. The former is characterized by generalized lipodystrophy and the latter
is characterized by by partial lipodystrophy



226) A patient presents with multiple tumors on the scalp with pathology shown. Which of the
following genes is most likely mutated?


A. NEMO

B. APC

C. CYLDCorrect Choice

D. SPINK5

E. PTEN
The pathology shown is that of a cylindroma. Multiple cylindromas of the scalp, associated with
eccrine spiradenomas can occur in the autosomal dominant familial cylindromatosis sydrome. The
remaining genes are not associated with formation of cylindromas




                                                 75
227) The most common ocular association with cutis marmorata telangiectatica congenital is:

A. Cataracts

B. Retinoblastoma

C. GlaucomaCorrect Choice

D. Corneal opacity

E. Angioid streaks
Glaucoma is the most common associated eye finding in CMTC patients. Glaucoma is also seen in
patients with neurofibromatosis type 1 and Sturge Weber patients



228) The following enzyme defect is most commonly seen in CHILD Syndrome.

A. 3-beta-hydroxysteroid isomerase

B. NAD oxido reductase

C. 3-beta-hydroxysteroid dehydrogenaseCorrect Choice

D. Aryl sulfatase E

E. DNA helicase
CHILD Syndrome is a X-linked dominant disorder characterized by unilateral ichthyosiform
erythroderma, ipsilateral limb deformity, and ipsilateral organ hypoplasia. The most commom gene
defect is NSDHL which encodes 3-beta hydroxysteroid dehydrogenase. EBP gene defects which
encode 3-beta-hydroxysteroid isomerase have been described, however this is the usual defect in
Conradi-Hunermann Syndrome. Aryl sulfatase E is mutated in X-linked recessive chondrodysplasia
punctata



229) Which of the following syndromes demonstrate atrophoderma vermiculatum?

A. All of these answers are correctCorrect Choice

B. Nicolau-Balus Syndrome

C. ROMBO Syndrome

D. Tuzun Syndrome

E. Braun-Falco-Marghescu Syndrome
All of the listed syndromes include atrophoderma vermiculatum as part of their constellation of
symptoms. Atrophoderma vermiculatum is characterized by honeycomb pattern of atrophic scars on
the face. Tuzun Syndrome also has scrotal tongue. ROMBO has BCCs, milia, peripheral vasodilation,
trichoepitheliomas. Nicolau-Balus has eruptive syringomas and milia. Braun-Falco-Marghescu has
keratosis pilaris and palmoplantar hyperkeratosis



230) Which opthamologic disease is associated with this disorder?

A. Posterior subcasular lentiular opacity

B. Cataracts

C. GlaucomaCorrect Choice




                                                    76
D. Ectopia lentis

E. Retinitis pigmentosa
Sturge-Weber syndrome is a sporadic disease characterized by a capillary malformation in the
trigeminal distribution. Patients may have associated cerebral atrophy, vascular malformations of
the leptomeninges, and seizures. All patients with Sturge-Weber should be referred to the
opthamologist for glaucoma screening



231) Ivory-colored papules between the angles of the scapulae are characteristic of which
syndrome:


A. Scheie

B. Morquio

C. HunterCorrect Choice

D. Hurler

E. Sanfilippo
These syndromes are all mucopolysaccharidoses. These papules are characteristic of Hunter
syndrome which is caused by a deficiency in iduronate sulfatase



232) You receive a hospital consult from the gastroenterology service for a 42-year old woman with
esophageal cancer. They would like your opinion on the yellow, thickened areas on her palms and
soles in areas of pressure. When you speak with her, she says that her father had similar problems
and it runs in her family. Which of the following is defective?


A. Connexin 31

B. Connexin 30.3

C. TOC geneCorrect Choice

D. Plakoglobin

E. Desmoplakin
This case describes Howell-Evans syndrome. This AD syndrome characteristically has a PPK in areas
of pressure, oral leukoplakia and esophageal carcinomas. Desmoplakin is defective in Carvajal
syndrome and plakoglobin in Naxos syndrome. Connexin 31 and 30.3 are linked to
erythrokeratoderma variabilis, which includes a PPK, but not esophageal carcinoma



233) Defects in Fibrillin 2 are linked with:

A. Congenital contractural arachnodactylyCorrect Choice

B. Lipoid proteinosis

C. Arthrochalasis multiplex congenita

D. Occipital horn syndrome

E. Cutis Laxa
Fibrillin 2 defects arelinked primarily with congenital contractural arachnodactyly. This syndrome is
associated with long limbs, arachnodactyly, scoliosis and crumpled ears. Occasionally, fibrillin 2 can
be associated with Marfan syndrome also. The other conditions are not linked to fibrillin mutations




                                                  77
234) A patient with multiple deeply pigmented papules has a skin biopsy which reveals an
epitheloid blue nevus. The next appropriate step is:


A. Order an echocardiogramCorrect Choice

B. Refer to genetics

C. Schedule prophylactic excision of the lesion

D. Begin a malignancy work-up

E. Reassure the patient and follow up as needed
Epithelioid blue nevi have been reported with and without association with cardiac myxomas as a
component of the Carney complex (NAME/LAMB syndrome). Carney complex is an autosomal
dominant disorder caused by mutations in PRKAR1A. Patients have cutaneous and atrial myxomas,
blue nevi, ephelides, adrenocortical disease, and testicular tumors



235) The nucleotide excision DNA repair pathway is defective in which disease

A. Xeroderma pigmentosaCorrect Choice

B. Bourneville's disease

C. Severe combined deficiency syndrome

D. Griscelli syndrome

E. Sjogren-Larssen syndrome
The pathogensis of Xeroderma Pigmentosum shows mutations i genes encoding DNA repair
enzymes, leading to defective DNA excision repair upon exposure to UV radiation (D). Severe
combined deficiency syndrome-major defect in cell-mediated and humoral immunity; most lack
antibody-dependent
cellular cytotoxicity and natural killer cell function (thus (B) is incorrect). The pathogensis for
Griscelli Syndrome is a mutation in gene encoding for myosin Va or RAB27 a (thus (C) is incorrect).
Sjogren-Larsson Syndrome has mutations in the FALDH gene (thus (E) is incorrect). Bourneville's
syndrome (AKA Tuberous Sclerosis) shows a mutation in either TSC1 ancoding hamartin or TSC2
encoding tuberin (thus (A) is incorrect



236) A child presenting with the scalp findings shown and a right arm hypoplasia would be
diagnosed with which of the following syndromes?


A. Bart's syndrome

B. Progeria

C. Adams-Oliver syndromeCorrect Choice

D. None of these options are correct

E. Dunnigan syndrome
Adams-Oliver syndrome is defined by aplasia cutis congenita (ACC) (shown in the image), usually of
the midline scalp with limb hypoplasia. Bart's syndrome also has ACC as a finding, but it is usually
present on the lower extremities and associated dominant dystrophic epidermolysis bullosa.
Progeria is a premature aging syndrome associated with a lamin-A mutation. Dunnigan syndrome is
also known as Familial partial lipodystrophy and is associated with a mutation in the BSCL2 gene.
Neither are associated with findings of ACC




                                                  78
237) Papillon-Lefevre and Haim-Munk syndromes have which of the following symptoms?

A. Pseudoainhum

B. Esophageal cancer

C. Eccrine syringofibradenoma

D. Right-ventricular cardiomyopathy

E. Periodontitis with tooth loss Correct Choice
Right-ventricular cardiomyopathy is associated with Naxos syndrome, pseudoainhum is associated
with Vohwinkel syndrome, esophageal cancer is associated with Howel-Evans syndrome, and eccrine
syringofibradenomas are associated with Schopf-Schulz-Passarge syndrome. Periodontitis with tooth
loss is associated with Papillon-Lefevre and Haim-Munk syndromes, which are caused by mutations
in Cathepsin C



238) The development of which malignancy is most commonly associated with lymphomatoid
papulosis?


A. non-Hodgkin's lymphoma

B. immunoblastic lymphoma

C. mycosis fungoidesCorrect Choice

D. Waldenstrom's macroglobulinemia

E. multiple myeloma
Lymphomatoid papulosis is a recurrant eruption of unclear etiology characterized by the appearance
of red-brown papules and nodules which spontaneously disappear in 3 to 8 weeks. It is notable for
histologic features which suggest a CD30 positive malignant lymphoma. There is controversy
regarding whether lymphomatoid papulosis (LyP) is a malignant, premalignant or benign condition.
The disease may last from months to years and in up to 20% of patients, it may be preceded by,
followed by, or associated with another type of cutaneous malignancy. Generally, this is mycosis
fungoides, a CD30-positive large T-cell lymphoma or Hodgkin's disease. Because of this potential
risk, long-term follow-up of these patients is required.

(Fitzpatrick's Dermatology in General Medicine, 5th ed 1999 p1269)
(Bolognia Dermatology 1st ed 2003, p1935-1937)



239) Pseudoxanthoma elasticum (PXE) can be transmitted in an autosomal dominant, recessive or
sporadic manner. Which of the following genes is mutated in PXE?


A. Collagen III

B. Collagen I

C. ABCC6Correct Choice

D. MAN1

E. Collagen V
ABCC6 is mutated in PXE. Other findings include fragmented and calcified elastin in skin, eyes,
arteries. There is the appearance of plucked chicken skin on the flexures and yellow papules on the
mucous membranes. Angioid streaks are present in the eye. Other findings include gastric
hemorrhage and arterial disease. The others are not involved in PXE




                                                  79
240) Pili trianguli et canaliculi is characteristic of which of the following syndromes?

A. Uncombable hair syndromeCorrect Choice

B. Bjornstad syndrome

C. Menkes kinky hair syndrome

D. Netherton's syndrome

E. Leiners disease
Uncombable hair syndrome is characterized by pili trianguli et canaliculi. On examination of the hair,
it is triangular with a canal-like groove runs along the shaft. The clinical findings are that of blond,
shiny, "spun glass" hair. It is an autosomal dominant syndrome with no known gene locus. The
remaining syndromes do not have this hair finding present



241) A BSCL2 gene mutation with the cutaneous findings of generalized lipodystrophy,
hyperlipemia, hepatomegaly, acanthosis nigricans, elevated basal metabolic rate and non-ketotic
insulin resistant diabetes mellitus are characteristic of which of the following syndromes?


A. None of the answers are correct

B. Familial partial lipodystrophy

C. All of the answers are correct

D. Bjornstad syndrome

E. Berardinelli-Seip congenital lipodystrophyCorrect Choice
Berardinelli-Seip congenital lipodystrophy is described above. Familial partial lipodystrophy is
characterized by a defect in LMNA and has symmetric lipoatrophy of trunk and limbs with sparing of
neck, shoulders, buffalo hump area and genitalia, tuboeruptive xanthomas, acanthosis nigricans and
hypertriglyceridemia. Bjornstad syndrome is characterized by pili torti and deafness



242) Collagen III is mutated in which type(s) of Ehlers-Danlos syndrome (EDS)?

A. Periodontitis (type VIII)

B. None of the answers are correct

C. Vascular (type IV)

D. Benign Hypermobile (type III)

E. All of the answers are correctCorrect Choice
Collagen III is mutated in all three types of EDS listed. Benign hypermobile type EDS is associated
with hypermobile joints and is autosomal dominant (AD) in transmission. Vascular type EDS is
associated with arterial and visceral rupture leading to early death, and visible venous patterns. It is
transmitted autosomal recessive (AR) or AD. Periodontitis type EDS is associated with mild EDS
symptoms and periodontitis



243) What is the classic radiologic findings associated with this disorer?

A. Calcifications of the falx-cerebri




                                                   80
B. Osteopatha striata

C. Dural calcifications

D. Osteopoikilosis

E. Tram-track calcifications of the temporal and occipital cortexCorrect Choice
Sturge-Weber syndrome is a sporadic disroder characterized by a facial capillary malformation in a
trigeminal nerve distribution. Patients with Sturge-Weber may have cerebral atrophy, ipsilateral
vascular malformations of the leptomeninges, seizures, and glaucoma. The classic radiologic finding
is tram-track calcifications of the temporal and occipital cortex



244) Findings of eyelid papules (string of pearls) and a hoarse cry in infants is characteristic of
which of the following syndromes?


A. Amyloidosis

B. Pseudoxanthoma elasticum

C. None of these answers are correct

D. Lipoid proteinosisCorrect Choice

E. Disseminated xanthomas
Findings of the eyelid string of pearls and a hoarse cry during the first years of life (due to vocal
cord infiltration) is characteristic of Lipoid Proteinosis (AKA Urbach-Wiethe disease or Hyalinosis
cutis et mucosae). It is an autosomal recessive condition with mutations in the extracellular matrix
protein 1 gene. Other findings include calcifications of the temporal lobe and hippocampus, hairloss,
atrophic scars and waxy papules on the face, verrucous nodules and a thick tongue. The other
conditions could be considered on the differential for Lipoid Proteinosis, but do not have the findings
described above



245) A 12 year-old boy with pits on his palms and lateral fingers may have:

A. Secondary syphilis

B. A corynebacteria infection

C. A hereditary keratoderma

D. Arsenic exposure

E. An inherited cancer syndromeCorrect Choice
Basal cell nevus syndrome is an autosomal dominant disease caused by mutations in the PTCH1
gene. Clinically, patients may have numerous basal cell carcinomas, palmoplantar pits, jaw cysts,
frontal bossing, bifid ribs, calcification of falx cerebri, medulloblastoma, ovarian fibromas and
fibrosarcomas



246) A patient with colon cancer is diagnosed with Muir-Torre syndrome. Which of the following
cutaneous lesions might the patient have?


A. Tricholemmomas

B. Basal cell carcinomas

C. Seborrheic keratoses




                                                  81
D. Keratoacanthomas Correct Choice

E. Arsenical keratoses
Muir-Torre syndrome is an autosomal dominant disease caused by mutations in MSH2 and MSH1,
DNA mismatch repair genes. Clinically, patients have multiple sebaceous tumors (adenomas are
most common), keratoacanthomas, and are at risk for adenocarcinoma of the colon



247) A patient with multiple sebaceous adenomas should be screened with which of the following
examinations?


A. Colonoscopy Correct Choice

B. Retinal examination

C. Renal ultrasound

D. MRI of the spine

E. Laryngoscopy
Muir-Torre syndrome is an autosomal dominant disorder caused by the HMSH2 and MLH1 DNA
mismatch repair genes. Clinically, there are numerous sebaceous adenomas, epitheliomas and
carcinomas and multiple keratoacanthomas associated with indolent colon and other visceral
adenocarcinomas. Patients and first-degree relatives should be screened by colonoscopy as colonic
adenocarcinomas may precede the development of cutaneous tumors



248) “Coast of Maine” café au lait macules are characteristic of which condition

A. Hypomelanosis of Ito

B. McCune-Albright syndrome Correct Choice

C. Tuberous sclerosis

D. Carney complex

E. Gaucher’s syndrome
McCune-Albright syndrome is a sporadic condition caused by somatic mutations in the Gs subunit of
adenylate cyclase. Key clinical features include “coast of Maine” café au lait macules, polyostotic
fibrous dysplasia, and precocious puberty.



249) Patients with junctional epidermolysis bullosa have been found to have mutations in:

A. All of the answers are correctCorrect Choice

B. laminin 5

C. collagen 17

D. bullous pemphigoid antigen 2

E. BP180
All of the answers are correct. Laminin 5 is a protein integral in the adhesion of the dermis to the
epidermis. Also involved in junctional epidermolysis bullosa is bullous pemphigoid antigen 2,
collagen 17 and BP180, which are synonymous for the same structure




                                                  82
250) Which of the following eye findings is caused by the rupture of Bruch's membrane?

A. Angioid streaksCorrect Choice

B. Blue sclerae

C. Keratoconus

D. Ruptured globe

E. Retinal detachment
The rupture of Bruch's membrane causes angioid streaks in pseudoxanthoma elasticum. Bruch's
membrane is the innermost layer of choroid with a central layer of elastic fibers. The other findings
are found in Ehlers-Danlos syndrome and are not related to Bruch's membrane



251) A 5 month old girl presents with failure to thrive. She has had life-long atopic dermatitis
treated with topical hydrocortisone cream and has persisent hypernatremia. On your exam, she has
generalized erythema and scaling of her body and trichorrhexis invaginata on examination of hairs
from her eyebrows. Which syndrome is she most likely to have?


A. Netherton SyndromeCorrect Choice

B. Omenn Syndrome

C. Severe atopic dermatitis

D. Wiskott-Aldrich Syndrome

E. Leiner syndrome
Netherton syndrome is caused by a mutation in the SPINK5 gene, encoding LEKT1. This is a serine
protease inhibitor which is important in downregulating inflammation. Early presentation is with
failure to thrive, generalized erythema/scale, hypernatremia, and sparse hair with the characteristic
finding of trichorrhexis invaginata. Pili torti and trichorrhexis nodosa also can be seen. Eyebrow hair
is most commonly affected. Omenn syndrome is an autosomal recessive form of severe combined
immunodeficiency (SCID) with findings of failure to thrive, erythroderma, scaling, chronic diarrhea,
lymphadenopathy, and hepatosplenomegaly. Leiner syndrome can present with failure to thrive,
immunodeficiency and seborrheic dermatitis. Wiskott-Aldrich syndrome is an x-linked recessive
condition with mutations of the WAS gene. Presentation includes atopic dermatitis,
thrombocytopenia, recurrent bacterial infection, lymphoreticular malignancy with non-Hodgkin's
lymphoma being the most common, and increased IgA, D and E. With the characteristic hair
changes, atopic dermatitis alone is not the most likely diagnosis



252) Patients with hemochromatosis are at increased risk for which of the following?

A. Polyarthritis

B. Yersenia infections

C. Vibrio vulnificus infections

D. Generalized metallic-grey hyperpigmentation

E. All of these options are correctCorrect Choice
Patients with hemochromatosis have increased intestinal iron absorption leading to systemic iron
overload. Signs inclued a generalized metallic-grey hyperpigmentation, koilonychia, alopecia
(especially pubic/axillary hair) cardiac failure/arrythmias/heart block, hepatomegaly with crrhosis,
diabetes (bronze diabetes), polyarthritis with chondrocalcinosis and are susceptible to Vibrio
vulnificus and Yersinia infections




                                                    83
253) The diagnostic test for chronic granulomatous disease is:

A. Bone marrow biopsy

B. Histamine skin test

C. Potassium hydroxide

D. Nitroblue tetrazolium reduction assayCorrect Choice

E. Dimethylglyoxime test
Chronic granulomatous disease is characterized by a defect in the ability to kill catalase positive
organisms within phagocytic leukocytes. This results from a neutrophilic defect in the cytochrome
found in the NADPH oxidative pathway responsible for a respiratory burst. The nitroblue tetrazolium
(NBT) reduction assay demonstrates the leukocyes’ ability to reduce the dye and produce a blue
color change. Patients with chronic granulomatous disease are unable to redue the dye



254) A 50 year man presents with generalized metallic-grey hyperpigmentation. His past medical
history includes diabetes, hepatomegaly and arrythmias. Laboratory tests should include:


A. Iron levelsCorrect Choice

B. Arsenic levels

C. Copper levels

D. Lead levels

E. Cyanide levels
Hemochromatosis is an autosomal recessive disease resulting in increased intestinal iron absorption
and iron deposition in a variety of organs. Clinical features include generalized metallic-grey
hyperpigmentation, koilonychia, sparse or absent hair, hepatomegaly, cardiac failure/arrhythmias,
insulin-dependent diabetes, hypogonadism and polyarthritis



255) Which of the following conditions is worsened by ingestion of lithium?

A. Darier’s DiseaseCorrect Choice

B. Hereditary lymphedema (Nonne-Milroy disease)

C. Hailey-Hailey Disease

D. Haim-Munk syndrome

E. Epidermolytic hyperkeratosis
Patients with Darier’s disease should not be treated with lithium due to its worsening or in some
cases unmasking the disease. The mechanism for this is not known. Lithium treatment does not
worsen the other listed conditions



256) Which of the following is caused by a defect in keratins 4 & 13?

A. Epidermolysis bullosa simplex

B. Epidermolysis bullosa simplex with myotonic dystrophy




                                                 84
C. Junctional EB with pyloric atresia

D. White sponge nevusCorrect Choice

E. Clouston’s syndrome
White sponge nevus is caused by a defect in keratins 4 & 13.
The remaining entities have the corresponding defects:
EB simplex—keratins 5 & 14
EB simplex with myotonic dystrophy—plectin
Junctional EB with pyloric atresia—Integrin ?-6, ?4
Clouston’s syndrome (hidrotic ectodermal dysplasia)—connexin 30.



257) The main cause of death in patients with dyskeratosis congenita is which of the following?

A. Leukemia

B. Pancytopenia Correct Choice

C. Oral squamous cell carcinoma

D. Atherosclerotic heart disease

E. Renal cell carcinoma
Dyskeratosis congenita is usually inherited in an X-recessive fashion due to mutations in the
dyskerin gene, which is involved in ribosomal RNA synthesis. The less common autosomal dominant
form is caused by mutations in the telomerase gene. Clinically, there is reticulated pigmentation of
skin, poikiloderma, alopecia, nail atrophy, premalignant oral leukoplakia, and Fanconi-type
pancytopenia resulting in early death



258) Which of the following is NOT a characteristic skin finding in patients with Down Syndrome?

A. Elastosis perforans serpiginosa

B. Syringomas

C. Flat nipples

D. Single palmar crease

E. Small tongueCorrect Choice
Down syndrome is caused by nondisjunction and results in trisomy 21. Clinical features include
single palmar crease, flat nipples, increased nuchal skin folds, syringomas, elastosis perforans
serpiginosa, xerosis, epicanthic folds of eyes, protruding scrotal tongue and fissured thickened lips.



259) Which vascular disorder is characterized by facial vascular malformation and ipsilateral
intracranial and retinal arteriovenous malformations(AVMs)?


A. Encephalotrigeminal angiomatosis

B. Von Lohuizen’s disease

C. PHACES

D. Bonnet Dechaune Blanc syndromeCorrect Choice

E. Sturge-Weber syndome




                                                  85
Bonnet Bechaune Blanc syndome, also know as Wyburn-Mason syndrome, is characterized by a
facial vascular malformation and ipsilateral intracranial and retinal AVMs. Encephalotrigeminal
angiomatosis is another name for Sturge-Weber. Von Lohuizen's disease is another name for cutis
marmorata telangiectatica congenita



260) The syndrome characterized by generalized mild hyperkeratosis, erythematous keratotic
plaques, palmoplantar keratoderma, non-progressive sensorineural deafness, progressive bilateral
keratitis with secondary blindness is:


A. Vohwinkel syndrome

B. CHILD syndrome

C. KID syndromeCorrect Choice

D. Refsum syndrome

E. Erythrokeratoderma variabilis
KID syndrome is described above. It is an autosomal dominant mutation in connexin 26. Vohwinkel
syndrome is also a connexin 26 mutation, but is characterized by diffuse honeycombed
palmoplantar keratoderma, pseudoainhum, starfish-shaped keratotic plaques over joints and
deafness. Erythrokeratoderma variabilis is an autosomal dominant mutation in connexin 31 and
30.3 characterized by erythematous migratory patches, fixed hyperkeratotic plaques and a
palmoplantar keratoderma. CHILD syndrome is an X-linked dominant mutation condition due to a
mutation in NAD(P)H Steroid dehydrogenas-like protein, lethal in males. Unilateral ichthyosiform
erythroderma, limb/visceral hypoplasias are characteristic. Refsum syndrome is an autosomal
recessive condition with a mutation in phytanoyl coenzyme A hydroxylase characterized by mild
ichthyosis, cerebellar ataxia, peripheral neuropathy, retinitis pigmentosa (salt & pepper) and
deafness



261) The most common cardiovascular defect in patients with Noonan syndrome is:

A. Ventricular septal defect

B. Aortic stenosis

C. Atrial septal defect

D. Enlarged aorta

E. Pulmonic valve stenosisCorrect Choice
Noonan’s syndrome is also known as cardiofaciocutaneous syndrome. Patients have short stature,
ptosis, hypertelorism, low-set ears, thick lips and curly hair. Pulmonic valve stenosis is the most
common cardiovascular defect, with atrial septal defects also common



262) What nail change is seen in patients with Mal de Meleda Syndrome?

A. leukonychia

B. pterygium

C. KoilonychiaCorrect Choice

D. Longitudinal ridging

E. Onycholysis




                                                 86
Mal de Meleda is an autosomal recessive disease characterized by transgedient malodorous PPK,
hyperhidrosis, keratotic plaques at knees and elbows, subungual hyperkeratosis, and koilonychia.
The gene defect is SLURP 1



263) Which of the following laboratory test might prove useful in the diagnosis of Fabry disease:

A. Stool guaiac

B. Bleeding time

C. Fasting lipids

D. Urinary sediment exam with polarizing light microscopyCorrect Choice

E. Complete blood count with differential
Patients with Fabry disease have a defect in the alpha-galactosidase A enzyme, leading to an
accumulation of glycosphingolipids in all tissues. Although patients are at increased risk for
myocardial infrctions and strokes, the serum lipid levels are normal. Ischemic events occur as a
result of glycosphingolipid accumulation in endothelial cells leading to swelling. In the brain, strokes
occur from direct vessel occlusion or stretching and distention of branches of dolichoectatic parent
vessels. Deposits in the kidneys leads to progressive renal failure with urine exam exhibiting
proteinuria and birefringent lipid globules (“maltese crosses”) seen with polarizing light microscopy



264) The x-linked recessive type of dyskeratosis congenita is:

A. PTEN

B. dyskerinCorrect Choice

C. CDKN2A

D. Menin

E. TERC
The dyskerin gene, whose product is involved in ribosomal RNA synthesis, is mutated in X-linked
recessive dyskeratosis congenita. TERC is linked with autosomal domininant transmission of the
syndrome. CDKN2A is involved in familial dysplastic nevi/melanoma syndrome, PTEN in Cowden
syndrome and Menin in MEN type I



265) Which of the following conditions is inherited in an X-linked recessive manner?

A. Sjogren-Larsson syndrome

B. Ichthyosis vulgaris

C. Wiskott-Aldrich SyndromeCorrect Choice

D. Epidermolysis bullosa simplex

E. Netherton’s Syndrome
Wiskott-Aldrich Syndrome is inherited in an X-linked recessive manner. Epidermolysis bullosa
simplex and ichthyosis vulgaris are inherited in an autosomal dominant (AD) manner. Sjogren-
Larsson and Netherton’s syndrome are inherited in an autosomal recessive manner




                                                  87
266) What is this syndrome which is histologically characterized by widely dispersed granular
material amidst normal fibers?


A. Focal Dermal Hypoplasia

B. Pseudoxanthoma ElasticumCorrect Choice

C. Buschke-Ollendorf Syndrome

D. Lipoid Proteinosis

E. Ehlers Danlos Syndrome
Pseudoxanthoma elasticum is genodermatosis characterized by redundant skin, angioid streaks,
yellow papules on the mucous membranes and bleeding from gastric artery. On histology, readily
apparent denerative changes of the elastic fibers are prominent, even without special stains



267) The most common neoplasm seen in Maffucci Syndrome is:

A. EnchondromasCorrect Choice

B. Angiosarcomas

C. chondrosarcoma

D. Lymphangiosarcomas

E. Osteosarcomas
Maffucci syndrome comprises of superficial and deep venous malformations, enchondromas, and
short stature. Enchondromas are the most common neoplasm, while chondrosarcomas are the most
common malignancies



268) Which of the following syndromes is X-linked dominant?

A. X-linked icthyosis

B. anhidrotic ectodermal dysplasia

C. dyskeratosis congenita

D. Menkes kinky hair syndrome

E. orofaciodigital syndrome 1Correct Choice
Orofaciodigital sydrome 1 is an X-linked dominantly inherited disorder caused by a defect in the
CXORF5 gene. The rest of the above conditions are inherited in an x-linked recessive pattern.



269) Which malignancy is seen in approximately 15-20% of people with the disease characterized
by a defect in a parathyroid hormone receptor protein?


A. osteosarcoma

B. chondrosarcomaCorrect Choice

C. angiosarcoma

D. rhabdomyosarcoma

E. epitheliod sarcoma




                                                 88
Approximately 15-20% of patients with Maffucci's syndrome will develop chondrosarcoma.
Maffucci's syndrome is due to a defect in a parathyroid hormone receptor protein



270) Medulloblastoma is seen in which syndrome?

A. Basal cell nevus syndromeCorrect Choice

B. Gardner’s syndrome

C. Multiple endocrine neoplasia 2b

D. Muir-Torre syndrome

E. Neurofibromatosis Type 1
Basal cell nevus syndrome is an autosomal dominant syndrome caused by a mutation in PTC gene,
which acts in the Sonic hedgehog pathway. Cutaneous manifestations of this genodermatosis
include basal cell carcinomas, palmoplantar pits, epidermoids cysts. Other findings include
odotogenic cysts, frontal bossing, bifid ribs, calcification of the falx cerebri and medulloblastomas



271) A 2 year old girl presents with sunken eyes, large ears, microcephaly and a photodistributed
eruption on her face. Eye exam reveals “salt and pepper” retina. The gene responsible for this
syndrome codes for a:


A. Transcription factor

B. DNA helicaseCorrect Choice

C. Lysosomal protease

D. Surface glycoprotein

E. Mismatch repair gene
The patient described has Cockayne syndrome, an autosomal recessive disorder believed to be due
to a mutation in either DNA helicase or defective nucleotide excision repair. UV irradiated cells have
decreased DNA and RNA synthesis and increased chromosomal breaks. Clinical features include
cachectic dwarfism with microcephaly, thin nose, large ears, photosensitive eruption, cataracts, salt
& pepper retina, and diffuse demyelination



272) Which of the following findings is characteristic of a mutation in lamin A?

A. Lipoatrophic sclerodermoid skin

B. All of the answers are correctCorrect Choice

C. Craniomegaly with small face

D. Severe premature atherosclerosis with early death

E. Alopecia
A mutation in Lamin A causes Progeria (Hutchinson-Gilford syndrome). Other findings include nail
atrophy and muscle/bone wasting. Presentation is in the first or second year of life. An increased
urine hyaluronic acid can be helpful in diagnosis



273) Which disease is found more commonly in mothers of patients with chronic granulomatous
disease?




                                                  89
A. Erythema nodosum

B. Sarcoidosis

C. Discoid lupus erythematousCorrect Choice

D. Wegener's disease

E. Churg-Straus disease
Female carriers of chronic granulomatous disease have an increase incidence of discoid lupus,
infections and apthous stomatitis



274) Osteopathia striata is seen in which of the following disorders?

A. Aplasia cutis congenita

B. Scleroderma

C. Focal dermal hypoplasia Correct Choice

D. Albright’s syndrome

E. Gaucher’s disease
Focal dermal hypoplasia (Goltz syndrome) is an X-linked dominant disorder that is lethal in males.
There is linear atrophy following Blaschko’s lines with areas of fat herniation with underlying
osteopathia striata, which is radiologically characterized by linear bony hyperdensity. Other features
include mucocutaneous papillomas and pits, alopecia, nail dystrophy, tooth abnormalities, and
colobomas



275) Xeroderma pigmentosum (XP) variant is different than classic XP in which of the following
way?


A. Increased chromosomal breakage and sister chromatid exchanges

B. Defective DNA nucleotide excision repair of the global genome

C. Defective DNA nucleotide excision repair of actively transcribing genes

D. Defective post-replication repairCorrect Choice

E. Low IgM
XP variant is DNA nucleotide excision repair proficient, but the defect is in post replication repair of
DNA. Increased chromosomal breakage and sister chromatid exchanges is found in Bloom’s
syndrome, an autosomal recessive syndrome caused by a defect in BLM gene, whose product
functions as a helicase. Clinical findings include: Telangiectasias, short stature, malar erythema,
recurrent infection, increased frequency of leukemia and lymphoma, normal intelligence. Defective
DNA nucleotide excision repair of actively transcribing genes is a feature of Cockayne’s syndrome,
an autosomal recessive syndrome with clinical findings including: Cachexia, short stature,
pigmentary retinal degeneration, progressive deafness and no increase in neoplasms. Xeroderma
pigmentosum has seven different complementation groups (A-G), each associated with a different
form of impairment of DNA nucleotide excision repair



276) A child presents with sparse, short hair and sensorineural deafness. On microscopic
examination of the hair, pili torti is noted. Which of the following syndromes is the most likely
diagnosis?


A. None of the options are correct



                                                   90
B. Menkes kinky hair syndrome

C. Bjornstad syndromeCorrect Choice

D. Argininosuccinic aciduria

E. Trichothiodystrophy
Bjornstad syndrome is the most likely diagnosis. This rare syndrome (~25 cases) is autosomal
recessive. Findings are of deafness and pili torti. The most common hair finding in Menkes
syndrome is pili torti, but it is not associated with hearing loss. Argininosuccinic aciduria is
associated with trichorrhexis nodosa and has no associated hearing loss



277) Which genetic defect could explain these cutaneous findings in addition to abnormal
immunoglobulin levels, recurrent respiratory infections, hypogonadism, and an increased risk of
leukemia and lymphoma?


A. Adenosine deaminase

B. ERCC6

C. WAS gene

D. RecQL3Correct Choice

E. NADPH oxidase
Bloom's syndrome is an autosomal recessive disorder caused by mutations in the RecQL3 gene
encoding a DNA helicase. Clinically, individuals with Bloom's syndrome have a photodistributed
erythema with telangectasia on the malar eminences. The may also have decreased IgM and IgA
levels, hypogonadism, and an increased risk for leukemia and lymphoma



278) A patient with multiple lentigines and blue nevi may also have:

A. mental retardation

B. deafness

C. atrial myxomaCorrect Choice

D. pulmonary valve stenosis

E. GI malignancy
This patient may have a constellation of features associated with NAME syndrome, otherwise known
as Carney complex or LAMB syndrome. This condition is inherited in an autosomal dominant pattern
and is due to a defect in the PRKAR1A gene. This condition is characterized by the following
features: blue Nevi, Atrial myxomas, cutaneous Myxomas, and Ephelides. In addition, testicular
tumors are seen as well as sexual precocity. Finally, patients may have endocrine abnormalities
including pigmented nodular adrenocorticoal disease and Cushing syndrome as well as pituitary
adenomas. Deafness, pulmonary stenosis, GI malignancies, and mental retardation are not features
of this condition.

(Spitz's Genodermatoses. 1996, p70-71)
(Abdelmalek, N. J Am Acad Dermatol 2002;46:161-183).



279) Ehlers-Danlos Syndrome with congenital adrenal hyperplasia is caused by mutations affecting
which of the following?




                                                 91
A. Fibronectin

B. Lysyl oxidase

C. Lysyl hydroxylase

D. Tenascin-X Correct Choice

E. Collagen 5
Ehlers-Danlos Syndrome type with congenital adrenal hyperplasia is caused by mutations in
tenascin-X



280) Which of the following diseases is seen only in females?

A. Hypomelanosis of Ito

B. Carney complex

C. Piebaldism

D. Griscelli syndrome

E. Incontinentia pigmenti Correct Choice
Incontinentia pigmenti (Bloch-Sulzberger syndrome) is an X-linked dominant disease that is lethal in
males. There are four stages: vesicular, verrucous, hyperpigmented and hypopigmented. Also seen
are peg and conical teeth, eye abnormalities, CNS defects, and alopecia. This condition is caused by
mutations in the NEMO gene



281) Which of the following is caused by a defect in cathepsin C?

A. Muir-Torre

B. Nail-Patella syndrome

C. Rubenstein-Taybi

D. Papillon-LefevreCorrect Choice

E. Bullous Icthyosis of Siemens
Papillon-Lefevre is a palmoplantar keratoderma caused by a mutation in chromosome 11q14,
leading to a defect in Cathepsin C, a lysosomal enzyme. Clinical manifestations of papillon lefevre
include sharply demarcated palmoplantar keratoderma with extension to dorsal surface
(transgrediens), spare hair, periodontitis, and pyoderma



282) Which of the following syndromes is characterized by follicular atrophoderma, hypohidrosis,
hypotrichosis and multiple basal cell carcinomas?


A. Bazex syndromeCorrect Choice

B. Rombo syndrome

C. Incontinentia Pigmenti

D. Rasmusen syndrome

E. Gorlin syndrome
Bazex syndrome has the findings of follicular atrophoderma, hypohidrosis, hypotrichosis and
multiple basal cell carcinomas (BCC). Rombo syndrome is associated with BCC and hypotrichosis,



                                                 92
but not the other listed findings. The atrophoderma in Rombo syndrome is vermicular, not follicular.
Rasmusen syndrome is not associated with BCC



283) Patients with x-linked icthyosis are more prone to getting which two malignancies?

A. Renal cell and ALL

B. Testicular and AML

C. Pancreatic and acute myelogenous leukemia (AML)

D. Testicular and ALLCorrect Choice

E. Pancreatic and acute lymphocytic leukemia (ALL)
Patients with x-linked icthyosis have a 20% chance of having cryptorchidism and are more prone to
both testicular cancer and acute lymphocytic leukemia



284) Odontogenic cysts and palmoplantar pits are seen in:

A. Bloom’s Syndrome

B. Refsum syndrome

C. Gorlin SyndromeCorrect Choice

D. Gardner’s syndrome

E. Goltz Syndrome
Gorlin syndrome (Basal Cell Nevus Syndrome)is an autosomal recessive disorder due to a defect in
the PTCH gene whose function normally inhibits “SMOOTHENED” signaling. Odontogenic cysts and
palmoplantar pits are characteristic features, in addition to multiple basal cell carcinomas. Other
features include: frontal bossing, kyphoscoliosis, calcification of falx cerebri, hypertelorism, ovarian
fibromas and rarely mental retardation



285) Comma shaped corneal opacities are seen in what disease?

A. Refsum Syndrome

B. Proteus syndrome

C. Sjogren-Larson Syndrome

D. X-linked ichthyosisCorrect Choice

E. Pseudoxanthoma elasticum
X-linked ichthyosis is a X-linked recessive disorder secondary to steroid sulfatase deficiency
characterized by brown adherent scale. Additional findings include comma-shaped corneal opacities,
cryptorchisdism, and failure to progress during labor.



286) Which of the following is caused by a mutation in a gene that leads to defective NF-KB
activation?


A. Vohwinkel’s

B. MEN IIa




                                                   93
ETAS_MCQ_03 b genodermatoses

Contenu connexe

Tendances

ETAS_MCQ_14 plants and creatures of dermatologic significance
ETAS_MCQ_14 plants and creatures of dermatologic significanceETAS_MCQ_14 plants and creatures of dermatologic significance
ETAS_MCQ_14 plants and creatures of dermatologic significance
Derma202
 
ETAS_MCQ_13 photobiology and photosensitivity disorders
ETAS_MCQ_13 photobiology and photosensitivity disordersETAS_MCQ_13 photobiology and photosensitivity disorders
ETAS_MCQ_13 photobiology and photosensitivity disorders
Derma202
 
ETAS_MCQ_15 dermatologic and cosmetic surgery
ETAS_MCQ_15 dermatologic and cosmetic surgeryETAS_MCQ_15 dermatologic and cosmetic surgery
ETAS_MCQ_15 dermatologic and cosmetic surgery
Derma202
 
ETAS_MCQ_01 structures of skin
ETAS_MCQ_01 structures of skinETAS_MCQ_01 structures of skin
ETAS_MCQ_01 structures of skin
Derma202
 
ETAS_MCQ_02 immunodermatology
ETAS_MCQ_02 immunodermatologyETAS_MCQ_02 immunodermatology
ETAS_MCQ_02 immunodermatology
Derma202
 
Previous year question on lichen planus based on neet pg, usmle, plab and fmg...
Previous year question on lichen planus based on neet pg, usmle, plab and fmg...Previous year question on lichen planus based on neet pg, usmle, plab and fmg...
Previous year question on lichen planus based on neet pg, usmle, plab and fmg...
Abhishek Gupta
 
Vitiligo in association with Erythema dyschromicum perstans
Vitiligo in association with Erythema dyschromicum perstansVitiligo in association with Erythema dyschromicum perstans
Vitiligo in association with Erythema dyschromicum perstans
VR Foundation
 

Tendances (20)

ETAS_MCQ_14 plants and creatures of dermatologic significance
ETAS_MCQ_14 plants and creatures of dermatologic significanceETAS_MCQ_14 plants and creatures of dermatologic significance
ETAS_MCQ_14 plants and creatures of dermatologic significance
 
ETAS_MCQ_13 photobiology and photosensitivity disorders
ETAS_MCQ_13 photobiology and photosensitivity disordersETAS_MCQ_13 photobiology and photosensitivity disorders
ETAS_MCQ_13 photobiology and photosensitivity disorders
 
Derma.
Derma.Derma.
Derma.
 
ETAS_MCQ_15 dermatologic and cosmetic surgery
ETAS_MCQ_15 dermatologic and cosmetic surgeryETAS_MCQ_15 dermatologic and cosmetic surgery
ETAS_MCQ_15 dermatologic and cosmetic surgery
 
ETAS_MCQ_01 structures of skin
ETAS_MCQ_01 structures of skinETAS_MCQ_01 structures of skin
ETAS_MCQ_01 structures of skin
 
ETAS_MCQ_02 immunodermatology
ETAS_MCQ_02 immunodermatologyETAS_MCQ_02 immunodermatology
ETAS_MCQ_02 immunodermatology
 
Previous year question on pemphigus vulgaris based on neet pg, usmle, plab an...
Previous year question on pemphigus vulgaris based on neet pg, usmle, plab an...Previous year question on pemphigus vulgaris based on neet pg, usmle, plab an...
Previous year question on pemphigus vulgaris based on neet pg, usmle, plab an...
 
Mrcp Part 2 Witten Exam
Mrcp Part 2 Witten ExamMrcp Part 2 Witten Exam
Mrcp Part 2 Witten Exam
 
Previous year question on lichen planus based on neet pg, usmle, plab and fmg...
Previous year question on lichen planus based on neet pg, usmle, plab and fmg...Previous year question on lichen planus based on neet pg, usmle, plab and fmg...
Previous year question on lichen planus based on neet pg, usmle, plab and fmg...
 
Previous year question on tonsils based on neet pg, usmle, plab and fmge or m...
Previous year question on tonsils based on neet pg, usmle, plab and fmge or m...Previous year question on tonsils based on neet pg, usmle, plab and fmge or m...
Previous year question on tonsils based on neet pg, usmle, plab and fmge or m...
 
Vohwinkel Syndrome vs. KID Syndrome
Vohwinkel Syndrome vs. KID Syndrome Vohwinkel Syndrome vs. KID Syndrome
Vohwinkel Syndrome vs. KID Syndrome
 
Internal Medicine Image Challenge MCQs
Internal Medicine Image Challenge MCQsInternal Medicine Image Challenge MCQs
Internal Medicine Image Challenge MCQs
 
MRCP MOCK EXAM
MRCP MOCK EXAMMRCP MOCK EXAM
MRCP MOCK EXAM
 
Previous year question on cataract based on neet pg, usmle, plab and fmge or ...
Previous year question on cataract based on neet pg, usmle, plab and fmge or ...Previous year question on cataract based on neet pg, usmle, plab and fmge or ...
Previous year question on cataract based on neet pg, usmle, plab and fmge or ...
 
Previous year question on leptospirosis based on neet pg, usmle, plab and fmg...
Previous year question on leptospirosis based on neet pg, usmle, plab and fmg...Previous year question on leptospirosis based on neet pg, usmle, plab and fmg...
Previous year question on leptospirosis based on neet pg, usmle, plab and fmg...
 
Previous year question on polio based on neet pg, usmle, plab and fmge or mci...
Previous year question on polio based on neet pg, usmle, plab and fmge or mci...Previous year question on polio based on neet pg, usmle, plab and fmge or mci...
Previous year question on polio based on neet pg, usmle, plab and fmge or mci...
 
Annular erythema
Annular erythemaAnnular erythema
Annular erythema
 
Previous year question on staining based on neet pg, usmle, plab and fmge or ...
Previous year question on staining based on neet pg, usmle, plab and fmge or ...Previous year question on staining based on neet pg, usmle, plab and fmge or ...
Previous year question on staining based on neet pg, usmle, plab and fmge or ...
 
Vitiligo in association with Erythema dyschromicum perstans
Vitiligo in association with Erythema dyschromicum perstansVitiligo in association with Erythema dyschromicum perstans
Vitiligo in association with Erythema dyschromicum perstans
 
Previous year question on otosclerosis from ent based on neet pg, usmle, plab...
Previous year question on otosclerosis from ent based on neet pg, usmle, plab...Previous year question on otosclerosis from ent based on neet pg, usmle, plab...
Previous year question on otosclerosis from ent based on neet pg, usmle, plab...
 

Similaire à ETAS_MCQ_03 b genodermatoses

Modes of inheritance (part 2)-Dr.Gourav
Modes of inheritance (part 2)-Dr.GouravModes of inheritance (part 2)-Dr.Gourav
Modes of inheritance (part 2)-Dr.Gourav
Gourav Thakre
 
congenital-bone-joint-diseases4000.pptx
congenital-bone-joint-diseases4000.pptxcongenital-bone-joint-diseases4000.pptx
congenital-bone-joint-diseases4000.pptx
Yasiele897
 

Similaire à ETAS_MCQ_03 b genodermatoses (20)

Hutchinson-Gilford Progeria Syndrome
Hutchinson-Gilford Progeria SyndromeHutchinson-Gilford Progeria Syndrome
Hutchinson-Gilford Progeria Syndrome
 
Ichthyoses and ichthyosiform
Ichthyoses and ichthyosiformIchthyoses and ichthyosiform
Ichthyoses and ichthyosiform
 
Congenital Bone & Joint Diseases
Congenital Bone & Joint DiseasesCongenital Bone & Joint Diseases
Congenital Bone & Joint Diseases
 
Genodermatoses
GenodermatosesGenodermatoses
Genodermatoses
 
Syndromes
SyndromesSyndromes
Syndromes
 
Approach to childhood poikiloderma.pptx
Approach to childhood poikiloderma.pptxApproach to childhood poikiloderma.pptx
Approach to childhood poikiloderma.pptx
 
Genetic disorders pdf
Genetic disorders pdfGenetic disorders pdf
Genetic disorders pdf
 
Modes of inheritance (part 2)-Dr.Gourav
Modes of inheritance (part 2)-Dr.GouravModes of inheritance (part 2)-Dr.Gourav
Modes of inheritance (part 2)-Dr.Gourav
 
GENETIC DISORDERS
GENETIC DISORDERSGENETIC DISORDERS
GENETIC DISORDERS
 
Duchenne muscular dystrophy -Prenatal Diagnosis & Genetic Counseling- Dr.Padmesh
Duchenne muscular dystrophy -Prenatal Diagnosis & Genetic Counseling- Dr.PadmeshDuchenne muscular dystrophy -Prenatal Diagnosis & Genetic Counseling- Dr.Padmesh
Duchenne muscular dystrophy -Prenatal Diagnosis & Genetic Counseling- Dr.Padmesh
 
X-chromosome and sex-linked diseases.pptx
X-chromosome and sex-linked diseases.pptxX-chromosome and sex-linked diseases.pptx
X-chromosome and sex-linked diseases.pptx
 
Lecture 15 genetic diseases
Lecture 15 genetic diseasesLecture 15 genetic diseases
Lecture 15 genetic diseases
 
Epidermolysis bullosa complete
Epidermolysis bullosa  completeEpidermolysis bullosa  complete
Epidermolysis bullosa complete
 
Martin-Bell Syndrome
Martin-Bell SyndromeMartin-Bell Syndrome
Martin-Bell Syndrome
 
congenital-bone-joint-diseases4000.pptx
congenital-bone-joint-diseases4000.pptxcongenital-bone-joint-diseases4000.pptx
congenital-bone-joint-diseases4000.pptx
 
Dermatology 5th year, 3rd lecture (Dr. Faraedon Kaftan)
Dermatology 5th year, 3rd lecture (Dr. Faraedon Kaftan)Dermatology 5th year, 3rd lecture (Dr. Faraedon Kaftan)
Dermatology 5th year, 3rd lecture (Dr. Faraedon Kaftan)
 
Malocclusion syndromes
Malocclusion  syndromesMalocclusion  syndromes
Malocclusion syndromes
 
Syndromes of Head & Neck
Syndromes of Head & NeckSyndromes of Head & Neck
Syndromes of Head & Neck
 
Genetics
GeneticsGenetics
Genetics
 
Vol 24 congenital 5
Vol 24 congenital 5Vol 24 congenital 5
Vol 24 congenital 5
 

Plus de Derma202 (6)

Phototherapy treatment protocol
Phototherapy treatment protocolPhototherapy treatment protocol
Phototherapy treatment protocol
 
Histopathplogical photos
Histopathplogical photosHistopathplogical photos
Histopathplogical photos
 
Slide study from ETAS
Slide  study from ETASSlide  study from ETAS
Slide study from ETAS
 
Arab board primary exam in dermatology 2012
Arab board primary exam  in dermatology 2012Arab board primary exam  in dermatology 2012
Arab board primary exam in dermatology 2012
 
Dermatology
DermatologyDermatology
Dermatology
 
Derm handbook for medical students and junior doctors 2010
Derm handbook for medical students and junior doctors 2010Derm handbook for medical students and junior doctors 2010
Derm handbook for medical students and junior doctors 2010
 

Dernier

Dernier (20)

Call Girls Dehradun Just Call 9907093804 Top Class Call Girl Service Available
Call Girls Dehradun Just Call 9907093804 Top Class Call Girl Service AvailableCall Girls Dehradun Just Call 9907093804 Top Class Call Girl Service Available
Call Girls Dehradun Just Call 9907093804 Top Class Call Girl Service Available
 
VIP Hyderabad Call Girls Bahadurpally 7877925207 ₹5000 To 25K With AC Room 💚😋
VIP Hyderabad Call Girls Bahadurpally 7877925207 ₹5000 To 25K With AC Room 💚😋VIP Hyderabad Call Girls Bahadurpally 7877925207 ₹5000 To 25K With AC Room 💚😋
VIP Hyderabad Call Girls Bahadurpally 7877925207 ₹5000 To 25K With AC Room 💚😋
 
♛VVIP Hyderabad Call Girls Chintalkunta🖕7001035870🖕Riya Kappor Top Call Girl ...
♛VVIP Hyderabad Call Girls Chintalkunta🖕7001035870🖕Riya Kappor Top Call Girl ...♛VVIP Hyderabad Call Girls Chintalkunta🖕7001035870🖕Riya Kappor Top Call Girl ...
♛VVIP Hyderabad Call Girls Chintalkunta🖕7001035870🖕Riya Kappor Top Call Girl ...
 
Night 7k to 12k Chennai City Center Call Girls 👉👉 7427069034⭐⭐ 100% Genuine E...
Night 7k to 12k Chennai City Center Call Girls 👉👉 7427069034⭐⭐ 100% Genuine E...Night 7k to 12k Chennai City Center Call Girls 👉👉 7427069034⭐⭐ 100% Genuine E...
Night 7k to 12k Chennai City Center Call Girls 👉👉 7427069034⭐⭐ 100% Genuine E...
 
Call Girls Nagpur Just Call 9907093804 Top Class Call Girl Service Available
Call Girls Nagpur Just Call 9907093804 Top Class Call Girl Service AvailableCall Girls Nagpur Just Call 9907093804 Top Class Call Girl Service Available
Call Girls Nagpur Just Call 9907093804 Top Class Call Girl Service Available
 
VIP Call Girls Indore Kirti 💚😋 9256729539 🚀 Indore Escorts
VIP Call Girls Indore Kirti 💚😋  9256729539 🚀 Indore EscortsVIP Call Girls Indore Kirti 💚😋  9256729539 🚀 Indore Escorts
VIP Call Girls Indore Kirti 💚😋 9256729539 🚀 Indore Escorts
 
Top Rated Bangalore Call Girls Richmond Circle ⟟ 9332606886 ⟟ Call Me For Ge...
Top Rated Bangalore Call Girls Richmond Circle ⟟  9332606886 ⟟ Call Me For Ge...Top Rated Bangalore Call Girls Richmond Circle ⟟  9332606886 ⟟ Call Me For Ge...
Top Rated Bangalore Call Girls Richmond Circle ⟟ 9332606886 ⟟ Call Me For Ge...
 
Premium Call Girls Cottonpet Whatsapp 7001035870 Independent Escort Service
Premium Call Girls Cottonpet Whatsapp 7001035870 Independent Escort ServicePremium Call Girls Cottonpet Whatsapp 7001035870 Independent Escort Service
Premium Call Girls Cottonpet Whatsapp 7001035870 Independent Escort Service
 
Call Girls Faridabad Just Call 9907093804 Top Class Call Girl Service Available
Call Girls Faridabad Just Call 9907093804 Top Class Call Girl Service AvailableCall Girls Faridabad Just Call 9907093804 Top Class Call Girl Service Available
Call Girls Faridabad Just Call 9907093804 Top Class Call Girl Service Available
 
Lucknow Call girls - 8800925952 - 24x7 service with hotel room
Lucknow Call girls - 8800925952 - 24x7 service with hotel roomLucknow Call girls - 8800925952 - 24x7 service with hotel room
Lucknow Call girls - 8800925952 - 24x7 service with hotel room
 
(Low Rate RASHMI ) Rate Of Call Girls Jaipur ❣ 8445551418 ❣ Elite Models & Ce...
(Low Rate RASHMI ) Rate Of Call Girls Jaipur ❣ 8445551418 ❣ Elite Models & Ce...(Low Rate RASHMI ) Rate Of Call Girls Jaipur ❣ 8445551418 ❣ Elite Models & Ce...
(Low Rate RASHMI ) Rate Of Call Girls Jaipur ❣ 8445551418 ❣ Elite Models & Ce...
 
Call Girls Tirupati Just Call 8250077686 Top Class Call Girl Service Available
Call Girls Tirupati Just Call 8250077686 Top Class Call Girl Service AvailableCall Girls Tirupati Just Call 8250077686 Top Class Call Girl Service Available
Call Girls Tirupati Just Call 8250077686 Top Class Call Girl Service Available
 
Call Girls Visakhapatnam Just Call 9907093804 Top Class Call Girl Service Ava...
Call Girls Visakhapatnam Just Call 9907093804 Top Class Call Girl Service Ava...Call Girls Visakhapatnam Just Call 9907093804 Top Class Call Girl Service Ava...
Call Girls Visakhapatnam Just Call 9907093804 Top Class Call Girl Service Ava...
 
Best Rate (Patna ) Call Girls Patna ⟟ 8617370543 ⟟ High Class Call Girl In 5 ...
Best Rate (Patna ) Call Girls Patna ⟟ 8617370543 ⟟ High Class Call Girl In 5 ...Best Rate (Patna ) Call Girls Patna ⟟ 8617370543 ⟟ High Class Call Girl In 5 ...
Best Rate (Patna ) Call Girls Patna ⟟ 8617370543 ⟟ High Class Call Girl In 5 ...
 
(Rocky) Jaipur Call Girl - 09521753030 Escorts Service 50% Off with Cash ON D...
(Rocky) Jaipur Call Girl - 09521753030 Escorts Service 50% Off with Cash ON D...(Rocky) Jaipur Call Girl - 09521753030 Escorts Service 50% Off with Cash ON D...
(Rocky) Jaipur Call Girl - 09521753030 Escorts Service 50% Off with Cash ON D...
 
Manyata Tech Park ( Call Girls ) Bangalore ✔ 6297143586 ✔ Hot Model With Sexy...
Manyata Tech Park ( Call Girls ) Bangalore ✔ 6297143586 ✔ Hot Model With Sexy...Manyata Tech Park ( Call Girls ) Bangalore ✔ 6297143586 ✔ Hot Model With Sexy...
Manyata Tech Park ( Call Girls ) Bangalore ✔ 6297143586 ✔ Hot Model With Sexy...
 
Call Girls Varanasi Just Call 9907093804 Top Class Call Girl Service Available
Call Girls Varanasi Just Call 9907093804 Top Class Call Girl Service AvailableCall Girls Varanasi Just Call 9907093804 Top Class Call Girl Service Available
Call Girls Varanasi Just Call 9907093804 Top Class Call Girl Service Available
 
VIP Service Call Girls Sindhi Colony 📳 7877925207 For 18+ VIP Call Girl At Th...
VIP Service Call Girls Sindhi Colony 📳 7877925207 For 18+ VIP Call Girl At Th...VIP Service Call Girls Sindhi Colony 📳 7877925207 For 18+ VIP Call Girl At Th...
VIP Service Call Girls Sindhi Colony 📳 7877925207 For 18+ VIP Call Girl At Th...
 
Call Girls Kochi Just Call 8250077686 Top Class Call Girl Service Available
Call Girls Kochi Just Call 8250077686 Top Class Call Girl Service AvailableCall Girls Kochi Just Call 8250077686 Top Class Call Girl Service Available
Call Girls Kochi Just Call 8250077686 Top Class Call Girl Service Available
 
Call Girls Haridwar Just Call 8250077686 Top Class Call Girl Service Available
Call Girls Haridwar Just Call 8250077686 Top Class Call Girl Service AvailableCall Girls Haridwar Just Call 8250077686 Top Class Call Girl Service Available
Call Girls Haridwar Just Call 8250077686 Top Class Call Girl Service Available
 

ETAS_MCQ_03 b genodermatoses

  • 1. 125) A patient with Bloom Syndrome is most likely to have which laboratory abnormalities: A. Elevated IgE B. Positive ANA C. Thrombocytopenia D. Macrocytic anemia E. Decreased immunoglobulinsCorrect Choice Bloom syndrome is an autosomal recessive disorder due to a mutation in the BLM gene which codes for a DNA helicase. Patients have impaired DNA repair after UV exposure and increased photosensitivity. Clinical features include photodistributed erythema, cheilitis, high-pitched voice, hypogonadism, and increased risk for leukemia, lymphoma and GI adenocarcinoma. Laboratory evaluation reveals decreased IgA, IgM and IgG leading to increased risk of respiratory infections 126) A patient with hypohydrosis and hyperpyrexia, anodontia, and sparse hair has which syndrome: A. Dyskeratosis congenita B. Papillon Lefevre C. Pachyonychia congenita D. Hidrotic ectodermal dysplasia E. Anhidrotic ectodermal dysplasiaCorrect Choice Anhidrotic ectodermal dysplasia is an x-linked recessive disorder which presents with the triad of hypohydrosis (or anhidrosis) with hyperpyrexia, anodontia (other dental findings include peg teeth, molars with hooked cusps) and sparse hair. Patients tend to overheat 127) A mentally retarded patient is found to also have a malar rash and downward lens displacement. Another clinical feature might include: A. periodontitis B. hypertrichosis C. deep venous thrombosesCorrect Choice D. multiple nevi E. short stature The above described patient has homocystinuria, an autosomal recessive disorder caused by a deficiency in cystathionine beta-synthase, leading to an accumulation of homocysteine. Clinical features include: malar flush, DVT’s, livedo reticularis, leg ulcers, downward lens displacement, myopia, glaucoma, sparse fine hair, mental retardation, and musculoskeletal anomalies 128) Which type of epidermolysis bullosa is associated with mitten deformities of the hands? A. Generalized atrophic benign epidermolysis bullosa (GABEB) B. Dominant dystrophic C. Weber-Cockayne D. Recessive dystrophic Correct Choice E. Herlitz type 47
  • 2. The recessive dystrophic type of epidermolysis bullosa is associated with chronic hand ulcers resulting in scarring that causes mitten deformities. These chronic scars often result in the formation of fatal squamous cell carcinomas. 129) Nevoid basal carcinoma syndrome (Gorlin syndrome) is autosomal dominant transmitted mutation of the patched gene. Symptoms include innumberable basal cell carcinomas, painful odontogenic jaw keratocysts, palmoplantar pits, frontal bossing, bifid ribs and what other bony abnormality? A. Osteopoikilosis B. Polyostotic fibrous dysplasia C. Calcification of falx cerebriCorrect Choice D. Sphenoid wing dysplasia E. Stippled epiphyses Calcification of falx cerebri is seen in Gorlin's syndrome. CHILD syndrome and chondrodysplasia punctata both can exhibit stippled epiphyses. Polyostotic fibrous dysplasia is found in McCune- Albright syndrome, osteopoikilosis in seen in Buschke-Ollendorf syndrome. Sphenoid wing dysplasia is seen in neurofibromatosis type I 130) A double row of eyelashes is associated with: A. Lymphedema-distichiasis syndromeCorrect Choice B. Hunters syndrome C. Cornelia de Lange syndrome D. Rubinstein-Taybi syndrome E. Russell-Silver syndrome A double row of eyelashes is defined as distichiasis and is associated with the Lymphedema- distichiasis syndrome. This syndrome is transmitted in an autosomal dominant fashion and is related to a mutation in FOXC2. Findings include late onset lymphedema, distichiasis, corneal irritation, ectropion, webbed neck and congenital heart defects. The remaining syndromes do not include distichiasis as a feature. 131) Patients with Russell-Silver syndrome exhibit: A. Broad thumbs B. Clinodactyly of fifth fingerCorrect Choice C. Peg teeth D. Shortened 4th and 5th metacarpals E. Osteopathia striata Characteristic features of Russell-Silver include short stature, bony asymmetry, triangular facies, clinodactyly of fifth finger, and precocious sexual development with cryptochordism/hypospadias. Broad thumbs are seen in Rubinstein-Taybi and shortened fourth and fifth metacarpals are seen in Turner syndrome. Osteopathia striata is characteristic of focal dermal hypoplasia 132) A child presents with macroglossia, exopthalmos and gigantism. He has a history of omphalocele repair and has circular depressions on the rim of the posterior helices. Although this syndrome is most often transmitted in a sporadic manner, 15% of cases have defects in which gene? 48
  • 3. A. FLT4 B. VHL C. FOXC2 D. ATM E. KIP2Correct Choice KIP2 can be mutated in 15% of cases of Beckwith-Wiedemann Syndrome. KIP2 is a cyclin- dependent kinase inhibitor gene which acts as a negative regulator of cell proliferation. These children have an increase risk of Wilms' tumor and organomegaly. In addition to the circular ear depression, they can also have a linear earlobe crease. The remaining genes are mutated in other syndromes with vascular disorders as a component: ATM in ataxia telangectasia, VHL in Von Hippel- Lindau, FLT4 in Hereditary lymphedema and FOXC2 in Lymphedema-distichiasis syndrome 133) Which of the following is defective in Ehlers-Danlos syndrome (EDS) with congenital adrenal hyperplasia? A. Lysyl hydroxylase B. Tenascin-XCorrect Choice C. Lysyl oxidase D. All of these answers are correct E. None of these answers are correct Tenascin-X defects are associated with EDS and with congenital adrenal hyperplasia. The phenotype is that of typical EDS with hyperextensible skin, hypermobile joints, and tissue fragility. Lysyl oxidase is defective in X-linked EDS (type V) and Occipital horn syndrome (type IX). Lysyl hydroxylase is defective in ocular-scoliotic (type VI) EDS 134) What is the most likely gene mutation in this individual who has migratory patches and fixed plaques as depicted in this picture? A. Connexin 26 B. Connexin 30.3 and 31Correct Choice C. Keratins 1 and 10 D. Calcium ATPase 2C1 E. Loricrin Erythrokeratodermis variabilis is an autosomal dominant disease characterized by transient patches of geographic erythema and fixed hyperkeratotic plaques. The disease is due to gene defects in connexin 30.3 and 31 135) The gene that is responsible for this disease also plays a pathogenic role in what other disorder? A. MastocytosisCorrect Choice B. Sarcoidosis C. Hereditary Angioedema D. Mycosis Fungoides E. Waardenburg Syndrome 49
  • 4. Piebaldism is an autosomal dominant disease characterized by depigmented patches and a white forelock. It is caused by a mutation in the c-kit proto-oncogene. Mutations in c-kit have also been found in the peripheral cells of individuals with mastocytosis 136) Menkes kinky hair syndrome is associated most commonly with which of the following hair abnormalities? A. Pili triangulati et canaliculati B. Pili tortiCorrect Choice C. None of these options are correct D. Trichorrhexis invaginata E. Trichorrhexis nodosa While pili torti is not exclusively found in Menkes kinky hair syndrome, this is the most common hair abnormality found. Trichorrhexis nodosa can be also seen. Trichorrhexis invaginata is commonly found in Netherton syndrome. Pili triangulati et canaliculati is the finding seen in Uncombable hair syndrome. 137) Medulloblastomas are seen in which of the following syndromes? A. Familial cylindromatosis B. Nevoid basal cell carcinomas syndrome Correct Choice C. Bazex syndrome D. Nicolau-Balus syndrome E. Birt-Hogg-Dube syndrome Nevoid basal cell carcinomas syndrome (Gorlin syndrome) is an autosomal dominant condition caused by mutations in the patched gene, which is involved in the hedgehog signaling pathway. Patients develop innumerable BCCs, palmoplantar pits, painful odontogenic jaw keratocysts, frontal bossing, bifid ribs, and calcification of the falx cerebri. Medulloblastomas and ovarian fibromas and fibrosarcomas are associated with this condition 138) Which of the following is caused by a defect in a gap junction protein? A. Hailey-hailey B. Epidermolysis bullosa simplex C. Bullous ichthyosis of siemens D. Erythrokeratoderma variabilisCorrect Choice E. Dyskeratosis congenita Erythrokeratoderma variabilis is also known as Mendes da Costa Syndrome. It is caused by a defect in connexin 31, a gap junction protein. EB simplex is caused by a mutation in keratins 5 & 14, Hailey-Hailey is caused by a mutation in calcium transporters, dyskeratosis congenita is caused by a defect in rRNA synthesis, and bullous icthyosis of Siemens is caused by mutations in keratin 2e 139) Which of the following syndromes is associated with markedly increased IgE levels, cold abscesses and a characteristic coarse facies? A. Chronic granulomatous disease 50
  • 5. B. Wiskott-Aldrich syndrome C. Job syndromeCorrect Choice D. Leiner's disease E. Severe combined immunodeficiency Job syndrome or Hyper IgE syndrome is characterized by these findings. In addition, there is a peripheral eosinophilia, eczematous dermatitis, frequent bronchitis and pneumonia, otitis media and sinusitis. The other listed conditions are associated with immunodeficiency. Wiskott-Aldrich can have eczematous dermatitis and all of these syndromes will have abnormal infections. They do not have markedly increased levels of IgE like Job syndrome. 140) Pruritus is Sjogren Larsson syndrome is attributed to accumulation of what molecule(s)? A. LeukotrieneCorrect Choice B. Histamine C. Bile salts D. None of these answers are correct E. All of these answers are correct Accumulation of leukotriene B4 contributes to pruritus in Sjogren Larsson syndrome. Leukotriene inhibitors may be helpful in controlling symptoms. 141) Which of the following syndromes is associated with tricholemmomas? A. Rasmussen's B. Bannayan-Riley-RuvacalbaCorrect Choice C. Basex D. Birt-Hogg-Dube E. Brooke-Spiegler Tricholemmomas are seen in Bannayan-Riley-Ruvacalba syndrome. This is an autosomal dominant condition with macrocephaly, lipomas, hemangiomas, skeletal abnormalities, lymphangioma circupscriptum, angiokeratomas, penile lentigines, acanthosis nigricans, and achrocrodons. There is an increased incidence of breast, thyroid, and GI cancers. Tricholemmomas are also seen in Cowden syndrome 142) Electron microscopic examination of a hair shaft reveals a canal-like groove along the shaft of a triangular-shaped hair. This patient has: A. Menke’s Kinky Hair syndrome B. Spun-glass hairCorrect Choice C. Trichothiodystrophy D. Netherton’s syndrome E. Bjornstad syndrome Pili trianguli et canaliculi is also known as Spun-glass hair or Uncombable Hair Syndrome. Netherton patients have trichorexis invaginata, Menke’s kinky hair patients have short, brittle sparse hairs, “tiger tail” hair is seen in trichothiodystrophy, and pili torti is seen in bjornstad syndrome 51
  • 6. 143) A patient with coarse facies, broad nasal bridge, and extensive eczema might be expected to have which abnormal laboratory value? A. Hypertriglyceridemia B. High Copper levels C. High IgE levelsCorrect Choice D. Low Zinc levels E. Anemia Patients’ with Hyper IgE Syndrome (Job syndrome) have impaired regulation of the IgE function and increased susceptibility to infections. In addition to recurrent cutaneous infections including cold abscesses, patients have widespread eczematous dermatitis, recurrent sinopulmonary infections and typically have coarse facies with broad nasal bridge and a prominent nose 144) Reticulate pigmentation of skin, poikiloderma, alopecia, nail atrophy, premalignant oral leukoplakia, and a Fanconi-type pancytopenia resulting in early death in addition to posterior fossa malformations is characteristic of which of the following syndromes A. Cockayne syndrome B. Hoyeraal-Hreidarsson syndromeCorrect Choice C. Bloom syndrome D. Dyskeratosis congenita E. Wiskott-Aldrich syndrome Hoyeraal-Hreidarsson syndrome is has all of the features of dyskeratosis congenita plus posterior fossa malformations. Bloom syndrome and Cockayne syndrome both have poikiloderma as features, but do not include posterior fossa malformations as part of the syndrome. Wiskott-Aldrich syndrome does not include any of these findings 145) Dyskeratosis congenita has all of the findings listed below EXCEPT: A. reticulate pigmentation of skin/poikiloderma B. alopecia C. Mental retardationCorrect Choice D. Is allelic to Hoyerall-Hreidarsson syndrome E. Fanconi-type pancytopenia resulting in early death Mental retardation is not part of the spectrum of disease of dyskeratosis congenita. The remaining options are correct. Hoyerall-Hreidarsson syndrome has all of the findings of dyskeratosis congenita with the addition of posterior fossa malformations 146) In ataxia telangectasia, the ATM gene is mutated. The product of the ATM gene is an enzyme which: A. participates in NF-kB activation B. binds transforming growth factor beta protein C. responds to DNA damage by phosphorylating key DNA repair substratesCorrect Choice D. is the VEGF receptor 3 52
  • 7. E. is an inhibitor of G1 cyclin/Cdk complexes The ATM gene is a member of the phosphatidylinositol-3 family of proteins that respond to DNA damage by phosphorylating key substrates involved in DNA repair according to OMIM. Defects in endoglin (TGF beta3 binding protein) is deficient in Osler-Weber-Rendu syndrome. The VEGF receptor 3 is defective in hereditary lymphedema. The NEMO gene is defective in Incontinentia Pigmenti. Its product, NF-kB essential modulator (NEMO) is a key activator in the NF-kB pathway. KIP2 is involved in AD Beckwith-Wiedemann syndrome and is an inhibitor of G1 cyclin/Cdk complexes 147) Ichthyosis hystrix is characterized by the following gene defects? A. keratins 5 and 14 B. keratins 1 and 10Correct Choice C. keratins 1 and 9 D. none of these answers are correct E. keratins 6 and 16 Ichthyosis hystrix or extensive epidermal nevi occurs secondary to a somatic mosaicism for keratins 1 and 10. If the mosaicism occurs on gonadal cells, offspring may have full blown epidermolytic hyperkeratosis (EHK). 148) Mucosal malignancy is a complication of: A. Oral hairy leukoplakia B. Chronic candidiasis C. Focal epithelial hyperplasia D. White sponge nevus E. Dyskeratosis congenitalCorrect Choice Dyskeratosis congenital, also called Zinsser-Engman-Cole syndrome, is an X-linked recessive genodermatosis caused by a mutation in DKC1. DKC1 encodes for dyskerin, which helps to maintain telomeres through the pseudouridylation of rRNA. Features of this condition include reticulate gray- brown hyperpigmentation, dystrophic nails, alopecia and Fanconi’s type pancytopenia. Patients may have premalignant leukoplakia which should be followed closely 149) What condition is associated with this finding of inflammatory keratotic facial papules which may result in scarring and atrophy? A. Reiter's syndrome B. Chloracne C. Keratosis pilaris D. Ulerythema ophryogenesCorrect Choice E. Systemic lupus erythematosus Ulerythema ophryogenes is a rare disorder that affects children and young adults. It is characterized by keratosis pilaris atrophicans and loss of lateral third of eyebrow 150) Ichthyosis linearis circumflexa is one of the findings seen in the syndrome caused by which of the following genes? 53
  • 8. A. SLURP1 B. CYLD C. GJB2 D. LMX1B E. SPINK5Correct Choice The SPINK5 gene encodes for LEKTI, a serine protease inhibitor important in the regulation of proteolysis in epithelia formation and keratinocyte terminal differentiation, is mutated in Netherton’s Syndrome. Other findings include: trichorrhexis invaginata (bamboo, ball and socket hair), atopic dermatitis, and anaphylaxis from food allergy. SLURP1 is mutated in Mal de Maleda, GJB2 in Vohwinkel’s syndrome, LMX1B in Nail-Patella syndrome, and CYLD in Familial Cylindromiasis 151) What is the mode of transmission for lamellar ichthyosis? A. sporadic B. X-linked dominant C. autosomal dominant D. autosomal recessiveCorrect Choice E. X-linked recessive Lamellar ichthyosis which is characterized by collodian membrane in newborns and platelike scale in children and adults is an autosomal recessive syndrome. The gene defect is transglutaminase 1 (TGM1 152) Which of the following diseases is caused by a mutation in a gap junction protein? A. Schopf-Schulz-Passarge syndrome B. Vohwinkel syndrome (classic) Correct Choice C. Striated PPK D. Mal de Meleda E. Vohwinkel syndrome (ichthyotic) Classic Vohwinkel syndrome is caused by mutations in connexin 26, a gap junction protein. Ichthyotic Vohwinkel syndrome is caused by mutations in loricrin and has ichthyosis but not deafness 153) Which of the following are cutaneous features of Marfan syndrome? A. Dermatofibrosis lenticularis and striae B. Fat herniation and cutaneous atrophy C. Striae and elastosis perforans serpiginosa Correct Choice D. Loose skin and crumpled ears E. Sclerodermoid changes and dyspigmentation Marfan syndrome is an autosomal dominant disorder caused by mutations in fibrillin 1 and 2. Patients have tall stature, arachnodactyly, pectus excavatum, high-arched palate, joint laxity, ectopia lentis with upward dislocation, dilated aorta with rupture, mitral valve prolapse, striae, and elastosis perforans serpiginosa 54
  • 9. 154) What gene defect would you expect to find in a child with white forelock, dystopia canthorum, and upper limb abnormalities? A. MITF B. Endothelin-3 C. Pax3Correct Choice D. SOX10 E. C-kit proto-oncogene Waardenburg's syndrome is characterized by depigmented patches, white forelock, and deafness. Both type 1 and 3 are caused by mutations in Pax3. Type 3 is also associated with limb abnormalities. MITF and SOX10 defects are responsible for types 2 and 4 respectively. C-kit proto- oncogene mutatios are seen in piebaldism 155) Palmoplantar keratoderma with deafness is caused by a defect in which gene? A. unknown B. Plakophilin C. lysosomal papain like cysteine proteinase D. SLURP-1 E. mitochondrial serine transferase RNACorrect Choice Palmoplantar keratoderma with deafness is caused by a defect in mitochondrial serine transferase RNA. A defect in SLURP-1 causes Mal de Meleda. A defect in plakophilin causes ectodermal dysplasia with skin fragility. A defect in cathepsin C lysososomal papain like cysteine proteinase causes Papillon LeFevre and Haim Munk. 156) A Puerto Rican woman is seen in clinic for a pruritic rash on her trunk. A punch biopsy is performed. The biopsy site continues to bleed, with hematoma formation. The bleeding is eventually controlled. On further exam, her skin and hair are light brown. She has a history of granulomatous colitis. What it the most likely reason she had excess bleeding with a simple procedure? A. Her platelets lack dense bodies, causing excess bleeding Correct Choice B. Her Factor VIII levels are low C. Her intrinsic factor is deficient D. She is congentially deficient in platelets E. None of the answers are correct Platelets without dense bodies cause excess bleeding in Hermansky-Pudlak syndrome. Other features of this condition include oculocutaneous albinism, ceroid lysosomal storage disease resulting in pulmonary fibrosis, granulomatous colitis 157) Which of the following syndromes are linked to a PTEN gene mutation? A. Cowden B. All of these options are correctCorrect Choice C. Lhermitte-Duclos disease D. Banayan-Zonana 55
  • 10. E. Proteus PTEN gene produces a phosphatase the regulates the cell-cycle and apoptosis, therefore acting as a tumor suppressor gene. Tissues affected by this mutation are those with increased proliferation such as epidermis, the oral and gastrointestinal mucosa, and the thyroid and breast epithelium. All of the syndromes listed have mutations in PTEN. 158) The combination of painful palmoplantar keratoderma and pseudoherpetic keratitis is characteristic of which of the following syndromes? A. Naxos syndrome B. Howel-Evans syndrome C. Richner-Hanhart syndrome Correct Choice D. Schopf-Schulz-Passarge syndrome E. Vohwinkel syndrome Richner-Hanhart syndrome (tyrosenemia type II) is an autosomal recessive disorder caused by a deficiency in hepatic tyrosine aminotransferase. This disease is characterized by painful PPK, pseudoherpetic keratitis and blindness. Treatment is low-tyrosine/phenylalanine diet 159) Epidermal nevus syndromes inheritance pattern is: A. Autosomal dominant B. X-linked dominant C. X-linked recessive D. Autosomal recessive E. sporadicCorrect Choice Epidermal nevus syndrome has many findings, including: sporadic inheritance, nevus unius lateris, capillary malformations, café au lait macules, mantal retardation and seizures, deafness, hemiparesis, hemihypertrophy of limbs, kyphoscoliosis and rare solid tumors. A biopsy is helpful to rule out epidermolytic hyperkeratosis. If positive, the patient’s offspring are at risk for generalized epidermolytic hyperkeratosis 160) Ichthyosis bullosa of Siemens is a condition characterized by fragile blisters at birth, hyperkeratotic plaques on elbows/knees later in life, and a gene mutation in: A. Keratin 1/10 B. Keratin 6a/16 C. None of these options are correct D. Keratin 5/14 E. Keratin 2eCorrect Choice Keratin 2e is mutated in ichythosis bullosa of Siemens. Keratin 5 and 14 are defective in epidermolysis bullosa simplex, 6a/16 in Pachyonychia congenital type I (Jadassohn-Lewandowsky), 1/10 in epidermolytic hyperkeratosis and Unna-Thost PPK 161) Spastic ditetraplegia is associated with which of the following disorders? A. Sjogren-Larsson syndrome Correct Choice 56
  • 11. B. X-linked ichthyosis C. Lamellar ichthyosis D. Refsum syndrome E. KID syndrome Sjogren-Larsson syndrome is an autosomal recessive disorder caused by mutations in the fatty aldehyde oxidoreductase/alcohol dehydrogenase gene. This disorder is characterized by ichthyosis, spastic ditetraplegia, mental retardation, epilepsy, glistening dot retinal pigmentation, and dental enamel dysplasia 162) A sporadic syndrome affecting transcriptional coactivator CREB-binding protein is: A. Nonne-Milroy disease B. Rubinstein-Taybi syndromeCorrect Choice C. Cornelia de Lange syndrome D. Maffucci syndrome E. Blue rubber bleb nevus syndrome Rubinstein-Taybi syndrome is caused by a sporadically transmitted defect in transcriptional coactivator CREB-binding protein. This gene is responsible for encoding a nuclear protein which acts as a co-activator of cAMP regulated gene expression. Findings of this syndrome include: capillary malformation, short stature, broad thumbs, craniofacial abnormalities including beaked nose, mental retardation, strabismus, congenital heart defects and cryptorchidism. The other listed conditions are not related to this defect 163) The arylsulfatase C gene is mutated in which disease? A. Naxos syndrome B. X-linked ichthyosisCorrect Choice C. Griscelli syndrome D. Haim-Munk syndrome E. Refsum syndrome Arylsulfatase C is also known as steroid sulfatase and is mutated in X-linked ichthyosis. This condition is inherited in a X-linked recessive pattern. Clinical findings include: brown scale sparing palms, soles and flexures, comma-shaped corneal opacities, failure of labor progression and cryptorchidism. It is also mutated in X-linked recessive type chondrodysplasia punctata 164) In one variant of epidermolysis bullosa simplex, those affected have muscular dystrophy in addition to the skin findings. Which protein mutation has been linked to this finding? A. plectinCorrect Choice B. plakophilin C. plakoglobin D. desmocollin E. desmoglein A plectin mutation is linked with this subtype of epidermolysis bullosa simplex. 165) A 3 year old boy presents with the findings seen in the image. He also has thrombocytopenia with purpura and a history of recurrent pyogenic bacterial infections. What is the most likely diagnosis in this child? A. Hyper-IgE syndrome 57
  • 12. B. Severe combined immunodeficiency C. Wiskott-Aldrich syndromeCorrect Choice D. Leiner syndrome E. Chronic Granulomatous disease The findings described are consistent with Wiskott-Aldrich syndrome. The characteristic triad can be simplified to the 3 P's - Pruritus (atopic dermatitis), Purpura (thrombocytopenia leading to purpura and other bleeding) and pyogenic infections. The remaining options are related to Wiskott-Aldrich in that they all have immunodeficiency as a feature, but not the same spectrum of disease 166) Pseudoxanthoma elasticum is caused by mutations in which of the following genes? A. Fibrillin 1 B. Collagen 5 C. ABCC6 gene Correct Choice D. Lysyl oxidase E. Elastin gene Pseudoxanthoma elasticum is caused by mutations in the ABCC6 gene, which is an ATP-using cell transporter. Elastin and lysyl oxidase mutations cause cutis laxa, fibrillin 1 mutations cause Marfan syndrome, and collagen 5 mutations cause Ehlers-Danlos syndrome 167) A 4 year old boy presents with 2 soft, dark-blue, compressible nodules on her extremities. His mother has noted that these lesions have increased sweating and that they were present at birth. No one else in the family has had similar skin lesions. What step is indicated first to help determine the diagnosis? A. Immediate referral to a gastroenterologist B. Biopsy of a skin lesion C. MRI of the abdomen D. Stool guiacCorrect Choice E. CBC Blue rubber bleb nevus syndrome is described above. There are multiple venous malformations on the extremities and trunk, often present at birth to early childhood. The number of these lesions increase with age. The lesions may have increased sweating and can be combined with lymphatic- venous malformations. Skin lesions can be a clue to gastrointestinal venous malformations which can lead to secondary bleeding and anemia. The most reasonable screening test to determine if the patient has GI hemorrhage is a stool guiac. An MRI or complete blood count can be helpful, but are not the best test to start with. A skin biopsy is not indicated. If there is GI blood loss, evaluation by a gastroenterologist is useful 168) A patient presents with starfish keratoses, pseudoainhum, honeycombed PPK, and generalized ichthyosis. What is the most likely genetic defect? A. Connexin 31 B. Connexin 26 58
  • 13. C. LoricrinCorrect Choice D. Connexin 30 E. Connexin 33 The patient has Vohwinkel syndrome. This is an autosomal dominant syndrome with 2 clinical variants. The variant described above with generalized ichthyosis is due to a loricrin mutation. In the classic form with nonprogressive hearing loss connexin 26 is mutated 169) The best screening test for hemochromatosis is: A. Complete Blood Count B. FerritinCorrect Choice C. Hematocrit D. Copper levels E. Transferrin The ferritin is the best screening test for hemochromatosis, an autosomal disease of increased intestinal iron absorption. The other tests are not the best method for screening 170) A 32 year old woman is 5 weeks pregnant and is diagnosed with hyperthyroidism. Her doctor gives her a prescription for on methimazole 10 mg PO tid. Which of the following fetal abnormalities could be caused by this exposure? A. Dermoid cyst B. Aplasia cutis congenitaCorrect Choice C. Spina bifida D. Encephalocele E. Meningocele In-utero methimazole exposures has been linked to aplasia cutis congenita and should not be used in pregnant women. The FDA pregnancy class is D. The other listed options are not linked with maternal methimazole usage 171) Ichthyosiform erythroderma in lines of Blaschko, follicular atrophoderma, and stippled epiphyses are characteristic of which of the following disorders? A. Refsum syndrome B. Netherton syndrome C. Erythrokeratodermia variabilis D. Sjogren-Larsson syndrome E. Chondrodysplasia punctata Correct Choice Chondrodysplasia punctata is a peroxisomal disorder caused by mutations in several genes that is associated with ichthyosiform erythroderma in lines of Blaschko, follicular atrophoderma, and stippled epiphyses 59
  • 14. 172) Yellow-brown depositions in Descemet's membrane of the corneas is diagnostic for: A. Neurofibromatosis B. Hemochromatosis C. Hepatolenticular degenerationCorrect Choice D. Pseudoxanthoma elasticum E. Diabetes mellitus Hepatolenticular degeneration or Wilson Disease is caused by a defect in biliary excretion of copper leading to accumulation in the liver, brain, cornea, pretibial hyperpigmentation, hepatomegaly and cirrhosis. The Kayser-Fleischer ring is the yellow-brown copper deposition in Descemet's membrane of the cornea. Other findings include ataxia, dysarthria and dementia. Hemochromatosis does not have an eye finding. Neurofibromatosis has Lisch nodules and Pseudoxanthoma elasticum has angioid streaks of the retina 173) The gene defect in Griscelli Syndrome is: A. None of these answers are correct B. Myosin Va or Rab27aCorrect Choice C. LYST or CHS1 D. P gene E. TRP1 Myosin Va or Rab27a are defective in Griscelli syndrome, an AR syndrome with mild albinism, pancytopenia, immunodeficiency, neurologic symptoms and an accelerated phase similar to Chediak-Higashi syndrome. LYST/CHS1 is defective in Chediak-Higashi syndrome. The P-gene is mutated in oculocutaneous albinism type 2 and TRP in oculocutaneous albinism type 3 174) Mutation in lamin A (nuclear envelope protein) has been found in: A. Peutz-Jeghers syndrome B. Albright’s syndrome C. Marfan syndrome D. Buschek-Ollendorf syndrome E. Progeria (Hutchinson-Gilford) Correct Choice Progeria (Hutchinson-Gilford syndrome) is a sporadic condition characterized by lipoatrophy, sclerodermoid skin, alopecia, nail atrophy, craniomegaly with small face, muscle/bone wasting, and severe premature atherosclerosis resulting in early death. Recent studies have shown that mutations in nuclear envelope protein lamin A is associated with progeria 175) The characteristic dental findings in patients with tuberous sclerosis are: A. Odontogenic cysts B. Retention of primary teeth C. Peg teeth 60
  • 15. D. Anodontia E. Enamel pitsCorrect Choice Enamel pits are the characteristic dental findings in tuberous sclerosis. Peg teeth are found in incontinentia pigmenti and anhidrotic ectodermal dysplasia. Anodontia is found in hypomelanosis of ito and incontinentia pigmenti. Odontogenic cysts are seen in Gorlin syndrome, and retention of primary teeth is characteristic of Job syndrome 176) Urticaria pigmentosa is linked to a defect in the c-kit protooncogene. What autosomal dominant skin disease also has been linked to this defect? A. PiebaldismCorrect Choice B. Hypomelanosis of Ito C. Incontinentia pigmenti D. Waardenburg syndrome E. Hermansky-Pudlak syndrome Piebaldism is linked to a defect in the c-kit protooncogene. Hypomelanosis of Ito has whorled hypopigmentation, occasional CNS defects, scoliosis and anodontia, a sporadic mutation. Waardenburg syndrome has 4 types, 1-3 are AD, 4 is AR. Type 1 & 3 have defects in PAX-3, 2 in MITF, and 4 in Sox10, endothelin-3 ligand or receptor genes. Hermansky-Pudlak syndrome is AR and most commonly linked to defects in HPS, a lysosomal transport protein and AP3B1, a protein important in endocytic/exocytic sorting. Incontinentia pigmenti is an X-linked dominant syndrome with a defect in the NEMO gene 177) Which of the following pairs of diseases are caused by mutations in DNA helicases? A. Xeroderma pigmentosum and Cockayne syndrome B. Muir-Torre syndrome and Birt-Hogg-Dube syndrome C. Cowden syndrome and Bannayan-Riley-Ruvalcaba syndrome D. Dyskeratosis congenita and Peutz-Jeghers syndrome E. Bloom syndrome and Rothmund-Thompson syndrome Correct Choice Bloom Syndrome and Rothmund-Thompson syndromes are caused by autosomal recessive mutations in DNA helicases. The common features include photodistributed poikiloderma. Patients with Bloom syndrome also demonstrate facial dysmorphism, hypogammaglobulinemia with recurrent respiratory and gastrointestinal tract infections, hypogonadism, leukemias and lymphomas, gastrointestinal adenocarcinomas, and oral/esophageal SCCs. Patients with Rothmund- Thompson syndrome develop premalignant acral keratoses, alopecia, nail dystrophy, cataracts, hypogonadism, and occasional solid tumors 178) Premature aging seen in Cockayne’s syndrome is caused by a mutation in which gene? A. ATM gene B. DNA helicase gene Correct Choice C. NEMO gene D. Patched gene 61
  • 16. E. Transglutaminase Cockayne’s syndrome is caused by a mutation in a DNA helicase gene, CSA or ERCC8. The condition is defined by growth deficiency, premature aging, and pigmentary retinal degeneration. 75% of patient have photosensitive eruptions and severe cataracts before the age of 3 are associated with poor prognosi 179) Which of the following metals is deficient in the serum of patients with Menkes kinky hair syndrome? A. Biotin B. Selenium C. Zinc D. Iron E. CopperCorrect Choice Menkes kinky hair syndrome is transmitted in an X-linked recessive manner and is caused by a mutation in ATP7A, an ATP-dependent copper tranporter. This defect results in low serum levels of copper. These individuals will have hair abnormalities such as sparse, hypopigmented brittle hair, eyelashes and eyebrows, lax skin, a "cupid's bow" upper lip, CNS progressive deterioration, seizures, skeletal abnormalities and tortuous arteries. The other listed items are not associated with Menkes syndrome 180) Which of the following is NOT a complication of Kasabach-Merritt Syndrome: A. Thrombocytopenia B. Disseminated intravascular coagulation C. Gastrointestinal bleeding D. CHF E. AtaxiaCorrect Choice Kasabach-Merritt Syndrome results from platelet trapping due to the presence of a single or multiple large hemangiomas. Hematologic complications include thrombocytopenia, microangiopathic hemolytic anemia, DIC, and acute hemorrhage. The presence of large hemangiomas leads to high output failure (CHF) and can also compress on surrounding structures 181) Patients with progeria typically die of which of the following conditions? A. Atherosclerotic heart disease Correct Choice B. Metastatic carcinoma C. Progressive systemic sclerosis D. Infection E. Nail atrophy Progeria (Hutchinson-Gilford syndrome) is a sporadic condition characterized by lipoatrophy, sclerodermoid skin, alopecia, nail atrophy, craniomegaly with small face, muscle/bone wasting, and severe premature atherosclerosis resulting in early death 62
  • 17. 182) On cutaneous exam, angiokeratoma corporis diffusum is characteristic of which of the following conditions? A. Fabry disease B. Sialodosis C. Fucosidosis D. All of these options are correctCorrect Choice E. None of these options are correct Findings of angiokeratoma corporis diffusum are found in all three listed conditions. They cannot by distinguished by skin exam 183) The histologic image shown in this picture is associated with which of the following syndromes? A. Dyskeratosis congenita B. Gorlin syndrome C. Banayan-Riley-Ruvalcaba syndrome D. Chediak-Higashi syndrome E. Multiple hamartoma syndromeCorrect Choice The histologic picture shown is that of a sclerotic fibroma with the characteristic storiform/whorled pattern of collagen. The lesion has low cellularity with spindle cells. Banayan-Riley-Ruvalcaba syndrome shares the same gene defect as Multiple hamartoma syndrome (Cowden syndrome) but is not associated with sclerotic fibromas. Dyskeratosis congenita is not associated with sclerotic fibromas 184) A patient with Crowe’s sign and an optic glioma has which of the following disorders? A. Tuberous sclerosis B. Neurofibromatosis II C. Watson syndrome D. Lester iris syndrome E. Neurofibromatosis I Correct Choice The diagnostic criteria for neurofibromatosis I include meeting 2 or more of the following 7 criteria: (1) >5 café au lait macules (CALMs) that are >5mm in a prepubertal person or >15 mm in a postpubertal person, (2) >1 neurofibroma or 1 plexiform neurofibroma, (3) axillary/inguinal freckling (Crowe’s sign), (4) optic glioma, (5) >1 Lisch nodule (iris hamartoma), (6) sphenoid dysplasia, (7) 1st degree relative with neurofibromatosis I 185) The most common cutaneous neoplasm associated with Muir Torre Syndrome is: A. Basal cell carcinoma with sebaceous differentiation B. Sebaceous adenomaCorrect Choice 63
  • 18. C. Keratoacanthoma D. Sebaceous carcinoma E. Squamous cell carcinomas Muir-Torre syndrome is an autosomal dominant disorder due to a DNA mismatch repair gene (MSH2). Patients present with sebaceous tumors, with adenomas being the most common (sebaceous carcinomas next common) and associated visceral malignancies such as colon cancer (most common). 186) The finding of 'maltese crosses' in the urine is characteristic of which of the following conditions? A. Fabry diseaseCorrect Choice B. Alkaptonuria C. Neimann-Pick disease D. Hunter syndrome E. Gaucher disease The 'maltese cross' finding in urine is characteristic of Fabry disease. Alkaptonuria will show dark urine with a pH > 7.0. There are no urinary findings in Hunter syndrome, Gaucher or Neimann-Pick disease 187) The gene PTEN (phosphatase and tensin homolog deleted on chromosome ten) is implicated in which of the following syndromes? A. Cowden B. None of the options are correct C. Banayan-Riley-Ruvalcaba D. All of the options are correctCorrect Choice E. Proteus Mutations in the PTEN gene are implicated in all three syndromes listed. There are conflicting reports in the literature regarding the linkage of Proteus and PTEN mutations. For further details, see OMIM #176920 188) Which cutaneous finding is seen in patients with phenylketonuria? A. Pigment dilution of hair and skinCorrect Choice B. Ichthyosis C. Phyrnoderma D. Angular stomatitis E. Erosive diaper dermatitis Phenylketonuria is an autosomal recessive condition caused by a mutation in the gene coding for phenylalanine hydroxylase. Defect in this enzyme results in accumulation of phenylalanine and its metabolites. Increased phenylalanine has toxic effects on the central nervous system in addition to competitively inhibiting tyrosine in melanogenesis. Inhibition of melanogenesis results in pigmentary 64
  • 19. dilution of the hair and skin. Other features of this condition include a predisposition to eczema, sclerodermoid changes of the skin, urine that has a distinctive "mousy" odor, psychomotor delay, mental retardation, seizures and hyperreflexia. A low-phenylalanine diet instituted early on can prevent these manifestations of the disease. The morbidity of phenylketonuria has improved since the advent of routine neonatal screening for this condition 189) All of the following disorders are exacerbated by UV radiation except: A. Rothmund-Thomopson syndrome B. Hartnup’s disease C. Bloom syndrome D. Cockayne syndrome E. Refsum syndromeCorrect Choice Refsum’s syndrome is an autosomal recessive disorder caused by mutations in phytanoyl-CoA hydroxylase. Clinically, patients have mild icthyosis, cerebellar ataxia, polyneuropathy, salt and pepper retinitis pigmentosa, sensorineural deafness, and arrhythmias with heart block. They are not overly sensitive to UV radiation 190) Angioid streaks on retinal exam are characteristic of which of the following syndromes? A. Pseudoxanthoma elasticumCorrect Choice B. Choroid malformations C. Salt & pepper retinitis pigmentosa D. Eyelid papillomas E. Lester iris Angioid streaks are characteristic of pseudoxanthoma elasticum. They are caused by rupture of Bruch's membrane of the choroid. Choroid malformations are found in Sturge-Weber syndrome, eyelid papillomas in xeroderma pigmentosum, Lester iris in Nail-patella syndrome and salt & pepper retinitis pigmentosa in Refsum syndrome 191) Which one of the following is the most common oncogenic virus in patients with epidermodysplasia verruciformis? A. HPV-16 B. HPV-5 Correct Choice C. HPV-13 D. HPV-8 E. HPV-33 Epidermodysplasia verruciformis is a rare autosomal recessive disorder in which an impaired cellular immunity allows widespread infection with certain subtypes of the human papilloma virus (HPV). Some of these lesions have a tendency for malignant transformation, most commonly those verruca caused by HPV type 5 65
  • 20. 192) Crumpled ears are associated with which disorder? A. Marfan syndrome B. Buschke-Ollendorf syndrome C. Cutis laxa D. Ehlers-Danlos Syndrome E. Congenital contractural arachnodactyly Correct Choice Congenital contractural arachnodactyly is an autosomal dominant disorder caused by mutations in fibrillin 2. Affected patients have long limbs, arachnodactyly, scoliosis, and crumpled ears 193) Which of the following conditions is inherited in an X-linked dominant (XD) manner? A. Focal dermal hypoplasia B. Chodrodysplasia punctata C. None of the answers are correct D. All of the answers are correctCorrect Choice E. CHILD Syndrome All of the syndromes listed are XD. Other XD syndromes are: Incontinentia pigmenti and Bazex syndrome 194) What is the characteristic radiographic finding in type I Gaucher disease? A. Osteopoikilosis B. Melorheostosis C. Enchondromas D. Ehrlenmeyer flask deformityCorrect Choice E. Supernumerary vertebrae with extra ribs The Ehrlenmeyer flask deformity is found in the femoral midshaft as well as aseptic necrosis of the femoral head and widening of the distal femur. Endochondromas are seen in Maffucci syndrome, Osteopoikilosis in Buschke-Ollendorf syndrome, Melorheostosis (linear hyperostosis under affected skin) in linear scleroderma and supernumerary vertebrae with extra ribs in incontinentia pigmenti 195) A child presents with pretibial hyperpigmentation, ataxia, decreased motor coordination, cirrhosis, and decreased motor coordination. The physical exam which would reveal the most specific finding for this disease is: A. Slit-lamp eye examCorrect Choice B. Renal ultrasound C. Hearing test D. Colonoscopy E. EKG 66
  • 21. Wilson’s disease (also known as hepatolenticular degeneration) is an autosomal recessive disorder result in defective biliary excretion of copper, leading to copper accumulation in the liver, brain and cornea. Clinical features include hepatomegaly, cirrhosis, ataxia, dysarthria, decreased motor coordination, pretibial hyperpigmentation, blue lunulae, and copper deposition in the cornea— Kayser-Fleisher ring, which can be diagnosed using a slit-lamp 196) Eyelid string of pearls are seen in which of the following conditions? A. Hutchinson-Gilford syndrome B. Lipoid proteinosis Correct Choice C. Focal dermal hypoplasia D. Gaucher’s disease E. Beare-Stevenson cutis gyrata syndrome Lipoid proteinosis is an autosomal recessive condition characterized by yellow papules on the face and oropharynx, eyelid string of pearls, hoarse voice, verrucous nodules of elbows and knees, and bean-shaped temporal and hippocampal calcification with occasional seizures. Histologically, there are PAS+ deposits in the affected tissue 197) What finding is seen on brain imaging of patients with Papillon-Lefevre Syndrome? A. calcification of the falx cerebri B. calcification of the hippocampus C. tram track calcifications D. calcification of the duraCorrect Choice E. agenesis of the corpus callosum Pappilon Lefevre Syndrome is an autosomal recessive syndrome characterized by transgredient PPK and periodontitis. There is a defect in cathepsin C. One sees dural calcification at the tentorium and choroid attachments. Tram track calcifications are seen in STurge-WEber. CAlcification of the falx cerebri and agenesis of the corpus callosum is seen in basal cell nevus syndrome. Hippocampal calcification is seen in lipoid proteinosi 198) A patient with Klinefelter Syndrome may be expected to experience which of the following: A. Recurrent leg ulcersCorrect Choice B. Scarring alopecia C. Recurrent pulmonary infections D. Gastroesophageal reflux E. Pulmonary valve stenosis Klinefelter syndrome results from nondisjunction during meiosis, leading to the XXY genotype. Patients are characteristically tall (long lower extremities) with scant body and pubic hair. Klinefelter patients have numerous varicosities predisposing them to recurrent leg ulcers 199) Which of the following is the first symptom of ataxia telangiectasias? 67
  • 22. A. Facial telangiectases B. Conjunctival telangictases C. Breast cancer D. Hematologic malgignancy E. Cerebellar ataxia Correct Choice Ataxia Telangiectasia (Louis-Bar syndrome) is an autosomal recessive disorder usually caused by mutations in the ATM gene, which is a chromosomal strand break repair enzyme. Cerebellar ataxia is the first sign, followed by telangiectases of the conjunctiva and skin. Thymic hypoplasia predisposes to increased infections. There is increased sensitivity to ionizing radiation resulting in hematologic and solid tumors. Female carriers have increased risk of breast cancer 200) Which type of epidermolysis bullosa simplex is associated with early death? A. Weber-Cockayne B. Dowling-Maera Correct Choice C. Non-Herlitz variant D. Ogna variant E. Generalized (Koebner) The Dowling-Maera variant of epidermolysis bullosa simplex is associated with widespread bullae, significant mucous membrane and laryngeal/esophageal involvement, nail dystrophy, and early death 201) Patients with Darier’s disease are at increased risk for: A. Melanoma B. Basal cell carcinoma C. Lipid abnormalities D. Kaposi’s varicelliform eruptionCorrect Choice E. Decreased life span Kaposi’s varicelliform eruption is the condition in which viral infection occurs in a patient with a pre- existing chronic dermatitis. Darier’s disease is an autosomal dominant genodermatosis caused by a mutation in ATP2A2 which encodes SERCA2. Cutaneous manifestations of warty, hyperkeratotic papules in a seborrheic dermatitis, which may be infected with HSV or bacteria 202) A patient with port wine stain on a lower extremity, hemihypertrophy of the limb and lymphatic and deep venouse insufficiency of the affected limb would be considered to have Klippel- Trenaunay-Weber syndrome. What additional feature would need to be present to define the patient as having Parkes-Weber syndrome? A. Macroglossia B. Multiple cafe-au-lait macules C. Arteriovenous fistulasCorrect Choice 68
  • 23. D. Cutis marmorata E. Distichiasis Parkes-Weber syndrome has the additional feature of arteriovenous fistulas. The remaining features are not part of these syndromes 203) What is the first sign seen in children with ataxia telangectasia syndrome? A. nystagmus B. cutaneous and bulbar conjunctival telangectasias Correct Choice C. recurrent viral or bacterial infections D. oculomotor apraxia E. cerebellar ataxia Telangectasias are the first sign of ataxia telangectasia syndrome (Louis-Bar). Other findings other than those listed above include: increased risk of lymphoma or breast carcinoma in heterozygotes, granulomas, cafe au lait macules. IgA, IgG2 and IgE will be decreased or absent. There is an increased sensitivity to ionizing radiation 204) A patient with this autosomal recessive disorder caused by a defect in helicase is an increased risk for which malignancy? A. Medullary thyroid carcinoma B. Renal cell carcinoma C. Acute leukemiaCorrect Choice D. Squamous cell carcinoma of the lung E. Prostate carcinoma Bloom's syndrome is an autosomal recessive disorder caused by a mutation in DNA helicase. It is characterized by photodistributed erythema in a butterfly distribution, malar hypoplasia with a prominent nose, high pitched voice, and an increased risk for malignancy (acute leukemia, lymphoma, and GI adenocarcinoma 205) In biopsies from blisters in patients with junctional epidermolysis bullosa, the split is found in the: A. Basal cell layer of the epidermis B. Lamina densa C. Lamina lucidaCorrect Choice D. Squamous cell layer of the epidermis E. None of the answers are correct The split seen in junctional epidermolysis bullosa is in the lamina lucida. The other locations can be involved in blistering disease, but not junctional epidermolysis bullosa 69
  • 24. 206) The NEMO gene is defective in Bloch-Sulzberger syndrome. What other syndrome has been linked with defects in the NEMO gene? A. Hypomelanosis of Ito B. Tuberous sclerosis C. Piebaldism D. Waardenburg syndrome E. Hypohidrotic ectodermal dysplasia with immune deficiency Correct Choice Hypohidrotic ectodermal dysplasia with immune deficiency, is caused by mutations in the NEMO (IKK-gamma gene). As opposed to the X-linked dominant inheritance of Bloch-Sulzberger syndrome (incontinentia pigmenti), this is a X-linked recessive disorder. Hypomelanosis of Ito is sporadically inherited and is not linked with a gene defect. Tuberous sclerosis is autosomal dominant and has been linked to defects in tuberin and hamartin tumor suppressor genes. Waardenburg syndrome has four subtypes, linked with the PAX-3, MITF and SOX10/endothelin-3 receptor genes. Piebaldism is linked to defects in the c-kit protooncogene 207) What is the gene defect in harlequin fetus? A. transglutaminase B. none of these answers are correct C. ABCC6 D. steroid sulfatse E. ABCA12Correct Choice Harlequin fetus is an autosomal recessive disorder. The gene defect is ABCA12 208) Regarding the inheritance of Ehlers-Danlos syndrome, which subtype is inherited in an X- linked recessive manner? A. The type with the lysyl oxidase deficiency Correct Choice B. The type with a collagen 5 defect C. The type with the lysyl hydroxylase deficiency D. The type with a defect in procollagen aminopeptidase E. The type with a collagen 3 defect Lysyl oxidase deficiency is related to type V or X-linked Ehlers-Danlos syndrome (EDS). Type IX, also has x-linked recessive inheritance with mild symptoms of EDS with occipital exostoses and hernias. Lysyl hydroxylase deficiency is seen in type VI EDS, linked with severe kyphoscoliosis, retinal detachment and other eye abnormalities. Collagen V deficiencies are seen in type I (Gravis) and type II (Mitis) EDS, associated with skin elasticity, gaping wounds, hypermobile joints, Gorlin’s sign, blue sclera and mitral valve prolapse 209) Anodontia is a bone finding seen in which of the following conditions: A. Tuberous sclerosis 70
  • 25. B. Letterer-Siwe disease C. Hyper-IgE syndrome D. Jackson Sertoli syndrome E. Hypomelanosis of ItoCorrect Choice Hypomelanosis of Ito, or Incontinentia pigmenti achromians is a condition characterized by marble- cake hypopigmentation, epilepsy, alopecia, scoliosis and mental/motor retardation. The characteristic dental abnormality is anodontia. The remaining syndromes are not associated with anodontia 210) Multiple cylindromas and trichoepitheliomas are associated with which of the following syndromes? A. Gardner B. Cronkhite-Canada C. Brook-SpieglerCorrect Choice D. Cowdens E. Nicolau-Balus These findings are characteristic of Brook-Spiegler syndrome. Gardner syndrome is associated with gastrointestinal carcinoma and adnexal neoplasms, Cronkhite-Canada syndrome, Nicolau-Balus and Cowdens are not associated with cylindromas or trichoepitheliomas 211) Which of the following features is not associated with Cornelia de Lange Syndrome? A. Normal intelligenceCorrect Choice B. Fifth finger clinodactyly C. Recurrent lung infections D. Cryptorchidism E. Characteristic facies with downturned mouth, hirsutism, synophrys, trichomegaly, anteverted nostrils, long philtrum and low set ears Children with Cornelia de Lange are usually severly retarded with an IQ <35. In addition to the features listed above, other features include cutis marmorata, hypoplastic nipples and umbilicus, low-pitched cry in infancy and congenital heart defects. While most cases are inherited in a sporadic manner, those cases which are familial are thought to be autosomal dominant and associated with the NIPBL (nipped-beta-like) gene. Prognosis is poor with premature death often secondary to sspiration or recurrent pulmonary infection 212) Erythematous keratotic plaques of KID Syndrome most commonly occur in which location? A. Chest B. Neck C. Back D. FaceCorrect Choice 71
  • 26. E. Abdomen The erythematous keratotic plaques of KID Syndome occur on face, extremities > trunk. KID Syndrome is characterized by keratitis, icthyosis, and deafness. It is inherited in an autosomal dominant fashion and caused by a mutation in the GJB2 gene which encodes connexin 26 213) Findings of milia, cylindromas and the condition shown in the pathology image are characteristic of which of the following syndromes? A. Brook-Spiegler syndrome B. Gorlin's syndrome C. Rombo syndrome D. Rasmusen syndromeCorrect Choice E. Familial cylindromatosis Rasmusen syndrome is characterized by milia, trichoepitheliomas and cylindromas. The other options have trichoepitheliomas as a feature of the syndrome, but not with the other listed findings 214) In addition to pheochromocytoma and medullary thyroid carcinoma and the skin finding in the image in a patient with a marfanoid body habitus is associated with which of the following syndromes? A. MEN type I B. Basex syndrome C. Cowden syndrome D. MEN type IIaCorrect Choice E. MEN type IIb The image show amyloidosis, which is found in addition to pheochromocytoma, medullary thyroid carcinoma and parathyroid abnormalities in Sipple syndrome or MEN type IIa. Cowden disease is not associated with any of the listed findings. 215) Argininosuccinic aciduria is associated with a hair abnormality shown in the image. Which of the following hair finding is seen in these patients? A. Trichorrhexis nodosaCorrect Choice B. Trichoschisis C. Trichorrhexis invaginata D. Pili torti E. None of these options are correct Trichorrhexis nodosa is shown in the figure and is found in ~50% of affected patients with argininosuccinic aciduria. Affected individuals will have short, broken scalp hairs, often increased with late onset disease. Usually these findings are increased in the occipital region of the scalp. The remaining options are not found in affected individual 216) The most common cutaneous association with monilethrix is: 72
  • 27. A. Atrophy B. Eczema C. Hypopigmentation D. Keratosis PilarisCorrect Choice E. Hyperpigmentation Monilethrix is an autosomal dominant condition which, by definition, presents with “beaded” hear. Clinically, patients present with short, sparse lusterless hair. Keratosis pilaris is the most common associated feature 217) An infant with doughy, redundant skin and short sparse hairs is likely to show which features on x-ray? A. Osteopoikilosis B. Sphenoid wing dysplasia C. Stippled epiphyses D. Periosteal thickening E. Metaphyseal widening in the long bonesCorrect Choice The patient described has Menkes Kinky Hair syndrome, an X-linked recessive disease due to a defect in an intestinal copper transport protein. Clinical features include pili torti, short, brittle “steel-wool” hair, and spare eyelashes and sparse broken eyebrows. The skin is often hypopigmented with a soft, “doughy” consistency and redundancy. Musculoskeletal manifestations include metaphyseal widening with spurs in the long bones 218) Familial macular and lichen amyloidosis is a feature of which of the following conditions? A. Sipple syndrome Correct Choice B. Peutz-Jeghers syndrome C. Birt-Hogg-Dube syndrome D. Dyskeratosis congenita E. Marfan syndrome Sipple syndrome (MEN 2a) is caused by autosomal dominant mutations in the ret protooncogene. Patients develop parathyroid cancers, pheochromocytomas, and medullary cancer of the thyroid gland. Familial macular and lichen amyloidosis is also a feature of this syndrome 219) Which of the following is a feature of Neurofibromatosis type II? A. Lisch nodules B. Optic gliomas C. Juvenile posterior subcapsular lenticular opacities Correct Choice D. Congenital hypertrophy of the retinal pigment epithelium E. Lester iris 73
  • 28. Neurofibromatosis type II is an autosomal dominant disorder caused by mutations in schwannomin/ merlin. Clinical features include cutaneous schwannomas and neurofibromas, bilateral vestibular schwannomas, and juvenile posterior subcapsular lenticular opacities 220) Findings of dysplastic nevi and melanoma inherited in an autosomal dominant fashion is linked with which of the following? A. Pancreatic malignancyCorrect Choice B. Renal cell carcinoma C. Breast malignancy D. Thyroid malignancy E. Colon carcinoma Familial dysplastic nevi/melanoma syndrome is linked with increased risk of pancreatic cancers and astrocytomas. There are no reported increased risks for the other types of cancers listed 221) Which gene is defective in Wiskott-Aldrich syndrome? A. NCF2 B. NCF1 C. WASCorrect Choice D. CYBB E. CYBA The WAS gene is defective in Wiskott-Aldrich syndrome. WAS is an Arp2/3 complex interacting protein. The remaining options are genes related to Chronic Granulomatous Disease and are not active in the pathogenesis of Wiskott-Aldrich syndrome 222) Refsum syndrome is due to a deficiency in phytanyl coenzyme A hydroxylase. Treatment for this condition is: A. Diet low in green vegetables, dairy and ruminant fats Correct Choice B. No treatment is available at this time C. Enzyme replacement D. Avoid phenylalanine E. Diet high in green vegetables, dairy and ruminant fats Treatment is with a diet low in green vegetables, dairy and ruminant fats is the treatment of choice for Refsum syndrome. Avoidance of specific amino acids is not helpful 223) Dermatofibrosis lenticularis disseminata and osteopoikilosis are findings seen with mutations of which of the following genes? A. Lysyl hydroxylase B. Lysyl oxidase 74
  • 29. C. LEMD3Correct Choice D. Fibrillin 2 E. ABCC6 Buschke-Ollendorf syndrome is caused by a loss-of-function mutation in LEMD3 224) An infant girl of short stature and shortened 4th and 5th metacarpals is being evaluated for coarctation of the aorta and horseshoe kidneys. Physical examination most likely reveals: A. Arachnodactyly B. Webbed neckCorrect Choice C. Hemangioma D. Giant congenital melanocytic nevus E. Alopecia Turner’s syndrome results from nondysjunction during gametogeneiss leading to the XO genotype. Clinical features include short stature, redundant neck folds/webbed neck, multiple pigmented nevi, low set hairline, triangular facies, low-set ears, ptosis, wide-set nipples, shortened 4th and 5th metacarpals, hypoplasia of lymphatics, coarctation of the aorta, and horseshoe kidneys 225) Mutations in nuclear lamins are associated with which clinical feature? A. Lipomatosis B. Sarcomas C. Aplasia cutis congenita D. Lipodystrophy Correct Choice E. Lymphedema Bernardinelli-Seip congenital lipodystrophy and familial partial lipodystrophy are caused by mutations in nuclear lamins. The former is characterized by generalized lipodystrophy and the latter is characterized by by partial lipodystrophy 226) A patient presents with multiple tumors on the scalp with pathology shown. Which of the following genes is most likely mutated? A. NEMO B. APC C. CYLDCorrect Choice D. SPINK5 E. PTEN The pathology shown is that of a cylindroma. Multiple cylindromas of the scalp, associated with eccrine spiradenomas can occur in the autosomal dominant familial cylindromatosis sydrome. The remaining genes are not associated with formation of cylindromas 75
  • 30. 227) The most common ocular association with cutis marmorata telangiectatica congenital is: A. Cataracts B. Retinoblastoma C. GlaucomaCorrect Choice D. Corneal opacity E. Angioid streaks Glaucoma is the most common associated eye finding in CMTC patients. Glaucoma is also seen in patients with neurofibromatosis type 1 and Sturge Weber patients 228) The following enzyme defect is most commonly seen in CHILD Syndrome. A. 3-beta-hydroxysteroid isomerase B. NAD oxido reductase C. 3-beta-hydroxysteroid dehydrogenaseCorrect Choice D. Aryl sulfatase E E. DNA helicase CHILD Syndrome is a X-linked dominant disorder characterized by unilateral ichthyosiform erythroderma, ipsilateral limb deformity, and ipsilateral organ hypoplasia. The most commom gene defect is NSDHL which encodes 3-beta hydroxysteroid dehydrogenase. EBP gene defects which encode 3-beta-hydroxysteroid isomerase have been described, however this is the usual defect in Conradi-Hunermann Syndrome. Aryl sulfatase E is mutated in X-linked recessive chondrodysplasia punctata 229) Which of the following syndromes demonstrate atrophoderma vermiculatum? A. All of these answers are correctCorrect Choice B. Nicolau-Balus Syndrome C. ROMBO Syndrome D. Tuzun Syndrome E. Braun-Falco-Marghescu Syndrome All of the listed syndromes include atrophoderma vermiculatum as part of their constellation of symptoms. Atrophoderma vermiculatum is characterized by honeycomb pattern of atrophic scars on the face. Tuzun Syndrome also has scrotal tongue. ROMBO has BCCs, milia, peripheral vasodilation, trichoepitheliomas. Nicolau-Balus has eruptive syringomas and milia. Braun-Falco-Marghescu has keratosis pilaris and palmoplantar hyperkeratosis 230) Which opthamologic disease is associated with this disorder? A. Posterior subcasular lentiular opacity B. Cataracts C. GlaucomaCorrect Choice 76
  • 31. D. Ectopia lentis E. Retinitis pigmentosa Sturge-Weber syndrome is a sporadic disease characterized by a capillary malformation in the trigeminal distribution. Patients may have associated cerebral atrophy, vascular malformations of the leptomeninges, and seizures. All patients with Sturge-Weber should be referred to the opthamologist for glaucoma screening 231) Ivory-colored papules between the angles of the scapulae are characteristic of which syndrome: A. Scheie B. Morquio C. HunterCorrect Choice D. Hurler E. Sanfilippo These syndromes are all mucopolysaccharidoses. These papules are characteristic of Hunter syndrome which is caused by a deficiency in iduronate sulfatase 232) You receive a hospital consult from the gastroenterology service for a 42-year old woman with esophageal cancer. They would like your opinion on the yellow, thickened areas on her palms and soles in areas of pressure. When you speak with her, she says that her father had similar problems and it runs in her family. Which of the following is defective? A. Connexin 31 B. Connexin 30.3 C. TOC geneCorrect Choice D. Plakoglobin E. Desmoplakin This case describes Howell-Evans syndrome. This AD syndrome characteristically has a PPK in areas of pressure, oral leukoplakia and esophageal carcinomas. Desmoplakin is defective in Carvajal syndrome and plakoglobin in Naxos syndrome. Connexin 31 and 30.3 are linked to erythrokeratoderma variabilis, which includes a PPK, but not esophageal carcinoma 233) Defects in Fibrillin 2 are linked with: A. Congenital contractural arachnodactylyCorrect Choice B. Lipoid proteinosis C. Arthrochalasis multiplex congenita D. Occipital horn syndrome E. Cutis Laxa Fibrillin 2 defects arelinked primarily with congenital contractural arachnodactyly. This syndrome is associated with long limbs, arachnodactyly, scoliosis and crumpled ears. Occasionally, fibrillin 2 can be associated with Marfan syndrome also. The other conditions are not linked to fibrillin mutations 77
  • 32. 234) A patient with multiple deeply pigmented papules has a skin biopsy which reveals an epitheloid blue nevus. The next appropriate step is: A. Order an echocardiogramCorrect Choice B. Refer to genetics C. Schedule prophylactic excision of the lesion D. Begin a malignancy work-up E. Reassure the patient and follow up as needed Epithelioid blue nevi have been reported with and without association with cardiac myxomas as a component of the Carney complex (NAME/LAMB syndrome). Carney complex is an autosomal dominant disorder caused by mutations in PRKAR1A. Patients have cutaneous and atrial myxomas, blue nevi, ephelides, adrenocortical disease, and testicular tumors 235) The nucleotide excision DNA repair pathway is defective in which disease A. Xeroderma pigmentosaCorrect Choice B. Bourneville's disease C. Severe combined deficiency syndrome D. Griscelli syndrome E. Sjogren-Larssen syndrome The pathogensis of Xeroderma Pigmentosum shows mutations i genes encoding DNA repair enzymes, leading to defective DNA excision repair upon exposure to UV radiation (D). Severe combined deficiency syndrome-major defect in cell-mediated and humoral immunity; most lack antibody-dependent cellular cytotoxicity and natural killer cell function (thus (B) is incorrect). The pathogensis for Griscelli Syndrome is a mutation in gene encoding for myosin Va or RAB27 a (thus (C) is incorrect). Sjogren-Larsson Syndrome has mutations in the FALDH gene (thus (E) is incorrect). Bourneville's syndrome (AKA Tuberous Sclerosis) shows a mutation in either TSC1 ancoding hamartin or TSC2 encoding tuberin (thus (A) is incorrect 236) A child presenting with the scalp findings shown and a right arm hypoplasia would be diagnosed with which of the following syndromes? A. Bart's syndrome B. Progeria C. Adams-Oliver syndromeCorrect Choice D. None of these options are correct E. Dunnigan syndrome Adams-Oliver syndrome is defined by aplasia cutis congenita (ACC) (shown in the image), usually of the midline scalp with limb hypoplasia. Bart's syndrome also has ACC as a finding, but it is usually present on the lower extremities and associated dominant dystrophic epidermolysis bullosa. Progeria is a premature aging syndrome associated with a lamin-A mutation. Dunnigan syndrome is also known as Familial partial lipodystrophy and is associated with a mutation in the BSCL2 gene. Neither are associated with findings of ACC 78
  • 33. 237) Papillon-Lefevre and Haim-Munk syndromes have which of the following symptoms? A. Pseudoainhum B. Esophageal cancer C. Eccrine syringofibradenoma D. Right-ventricular cardiomyopathy E. Periodontitis with tooth loss Correct Choice Right-ventricular cardiomyopathy is associated with Naxos syndrome, pseudoainhum is associated with Vohwinkel syndrome, esophageal cancer is associated with Howel-Evans syndrome, and eccrine syringofibradenomas are associated with Schopf-Schulz-Passarge syndrome. Periodontitis with tooth loss is associated with Papillon-Lefevre and Haim-Munk syndromes, which are caused by mutations in Cathepsin C 238) The development of which malignancy is most commonly associated with lymphomatoid papulosis? A. non-Hodgkin's lymphoma B. immunoblastic lymphoma C. mycosis fungoidesCorrect Choice D. Waldenstrom's macroglobulinemia E. multiple myeloma Lymphomatoid papulosis is a recurrant eruption of unclear etiology characterized by the appearance of red-brown papules and nodules which spontaneously disappear in 3 to 8 weeks. It is notable for histologic features which suggest a CD30 positive malignant lymphoma. There is controversy regarding whether lymphomatoid papulosis (LyP) is a malignant, premalignant or benign condition. The disease may last from months to years and in up to 20% of patients, it may be preceded by, followed by, or associated with another type of cutaneous malignancy. Generally, this is mycosis fungoides, a CD30-positive large T-cell lymphoma or Hodgkin's disease. Because of this potential risk, long-term follow-up of these patients is required. (Fitzpatrick's Dermatology in General Medicine, 5th ed 1999 p1269) (Bolognia Dermatology 1st ed 2003, p1935-1937) 239) Pseudoxanthoma elasticum (PXE) can be transmitted in an autosomal dominant, recessive or sporadic manner. Which of the following genes is mutated in PXE? A. Collagen III B. Collagen I C. ABCC6Correct Choice D. MAN1 E. Collagen V ABCC6 is mutated in PXE. Other findings include fragmented and calcified elastin in skin, eyes, arteries. There is the appearance of plucked chicken skin on the flexures and yellow papules on the mucous membranes. Angioid streaks are present in the eye. Other findings include gastric hemorrhage and arterial disease. The others are not involved in PXE 79
  • 34. 240) Pili trianguli et canaliculi is characteristic of which of the following syndromes? A. Uncombable hair syndromeCorrect Choice B. Bjornstad syndrome C. Menkes kinky hair syndrome D. Netherton's syndrome E. Leiners disease Uncombable hair syndrome is characterized by pili trianguli et canaliculi. On examination of the hair, it is triangular with a canal-like groove runs along the shaft. The clinical findings are that of blond, shiny, "spun glass" hair. It is an autosomal dominant syndrome with no known gene locus. The remaining syndromes do not have this hair finding present 241) A BSCL2 gene mutation with the cutaneous findings of generalized lipodystrophy, hyperlipemia, hepatomegaly, acanthosis nigricans, elevated basal metabolic rate and non-ketotic insulin resistant diabetes mellitus are characteristic of which of the following syndromes? A. None of the answers are correct B. Familial partial lipodystrophy C. All of the answers are correct D. Bjornstad syndrome E. Berardinelli-Seip congenital lipodystrophyCorrect Choice Berardinelli-Seip congenital lipodystrophy is described above. Familial partial lipodystrophy is characterized by a defect in LMNA and has symmetric lipoatrophy of trunk and limbs with sparing of neck, shoulders, buffalo hump area and genitalia, tuboeruptive xanthomas, acanthosis nigricans and hypertriglyceridemia. Bjornstad syndrome is characterized by pili torti and deafness 242) Collagen III is mutated in which type(s) of Ehlers-Danlos syndrome (EDS)? A. Periodontitis (type VIII) B. None of the answers are correct C. Vascular (type IV) D. Benign Hypermobile (type III) E. All of the answers are correctCorrect Choice Collagen III is mutated in all three types of EDS listed. Benign hypermobile type EDS is associated with hypermobile joints and is autosomal dominant (AD) in transmission. Vascular type EDS is associated with arterial and visceral rupture leading to early death, and visible venous patterns. It is transmitted autosomal recessive (AR) or AD. Periodontitis type EDS is associated with mild EDS symptoms and periodontitis 243) What is the classic radiologic findings associated with this disorer? A. Calcifications of the falx-cerebri 80
  • 35. B. Osteopatha striata C. Dural calcifications D. Osteopoikilosis E. Tram-track calcifications of the temporal and occipital cortexCorrect Choice Sturge-Weber syndrome is a sporadic disroder characterized by a facial capillary malformation in a trigeminal nerve distribution. Patients with Sturge-Weber may have cerebral atrophy, ipsilateral vascular malformations of the leptomeninges, seizures, and glaucoma. The classic radiologic finding is tram-track calcifications of the temporal and occipital cortex 244) Findings of eyelid papules (string of pearls) and a hoarse cry in infants is characteristic of which of the following syndromes? A. Amyloidosis B. Pseudoxanthoma elasticum C. None of these answers are correct D. Lipoid proteinosisCorrect Choice E. Disseminated xanthomas Findings of the eyelid string of pearls and a hoarse cry during the first years of life (due to vocal cord infiltration) is characteristic of Lipoid Proteinosis (AKA Urbach-Wiethe disease or Hyalinosis cutis et mucosae). It is an autosomal recessive condition with mutations in the extracellular matrix protein 1 gene. Other findings include calcifications of the temporal lobe and hippocampus, hairloss, atrophic scars and waxy papules on the face, verrucous nodules and a thick tongue. The other conditions could be considered on the differential for Lipoid Proteinosis, but do not have the findings described above 245) A 12 year-old boy with pits on his palms and lateral fingers may have: A. Secondary syphilis B. A corynebacteria infection C. A hereditary keratoderma D. Arsenic exposure E. An inherited cancer syndromeCorrect Choice Basal cell nevus syndrome is an autosomal dominant disease caused by mutations in the PTCH1 gene. Clinically, patients may have numerous basal cell carcinomas, palmoplantar pits, jaw cysts, frontal bossing, bifid ribs, calcification of falx cerebri, medulloblastoma, ovarian fibromas and fibrosarcomas 246) A patient with colon cancer is diagnosed with Muir-Torre syndrome. Which of the following cutaneous lesions might the patient have? A. Tricholemmomas B. Basal cell carcinomas C. Seborrheic keratoses 81
  • 36. D. Keratoacanthomas Correct Choice E. Arsenical keratoses Muir-Torre syndrome is an autosomal dominant disease caused by mutations in MSH2 and MSH1, DNA mismatch repair genes. Clinically, patients have multiple sebaceous tumors (adenomas are most common), keratoacanthomas, and are at risk for adenocarcinoma of the colon 247) A patient with multiple sebaceous adenomas should be screened with which of the following examinations? A. Colonoscopy Correct Choice B. Retinal examination C. Renal ultrasound D. MRI of the spine E. Laryngoscopy Muir-Torre syndrome is an autosomal dominant disorder caused by the HMSH2 and MLH1 DNA mismatch repair genes. Clinically, there are numerous sebaceous adenomas, epitheliomas and carcinomas and multiple keratoacanthomas associated with indolent colon and other visceral adenocarcinomas. Patients and first-degree relatives should be screened by colonoscopy as colonic adenocarcinomas may precede the development of cutaneous tumors 248) “Coast of Maine” café au lait macules are characteristic of which condition A. Hypomelanosis of Ito B. McCune-Albright syndrome Correct Choice C. Tuberous sclerosis D. Carney complex E. Gaucher’s syndrome McCune-Albright syndrome is a sporadic condition caused by somatic mutations in the Gs subunit of adenylate cyclase. Key clinical features include “coast of Maine” café au lait macules, polyostotic fibrous dysplasia, and precocious puberty. 249) Patients with junctional epidermolysis bullosa have been found to have mutations in: A. All of the answers are correctCorrect Choice B. laminin 5 C. collagen 17 D. bullous pemphigoid antigen 2 E. BP180 All of the answers are correct. Laminin 5 is a protein integral in the adhesion of the dermis to the epidermis. Also involved in junctional epidermolysis bullosa is bullous pemphigoid antigen 2, collagen 17 and BP180, which are synonymous for the same structure 82
  • 37. 250) Which of the following eye findings is caused by the rupture of Bruch's membrane? A. Angioid streaksCorrect Choice B. Blue sclerae C. Keratoconus D. Ruptured globe E. Retinal detachment The rupture of Bruch's membrane causes angioid streaks in pseudoxanthoma elasticum. Bruch's membrane is the innermost layer of choroid with a central layer of elastic fibers. The other findings are found in Ehlers-Danlos syndrome and are not related to Bruch's membrane 251) A 5 month old girl presents with failure to thrive. She has had life-long atopic dermatitis treated with topical hydrocortisone cream and has persisent hypernatremia. On your exam, she has generalized erythema and scaling of her body and trichorrhexis invaginata on examination of hairs from her eyebrows. Which syndrome is she most likely to have? A. Netherton SyndromeCorrect Choice B. Omenn Syndrome C. Severe atopic dermatitis D. Wiskott-Aldrich Syndrome E. Leiner syndrome Netherton syndrome is caused by a mutation in the SPINK5 gene, encoding LEKT1. This is a serine protease inhibitor which is important in downregulating inflammation. Early presentation is with failure to thrive, generalized erythema/scale, hypernatremia, and sparse hair with the characteristic finding of trichorrhexis invaginata. Pili torti and trichorrhexis nodosa also can be seen. Eyebrow hair is most commonly affected. Omenn syndrome is an autosomal recessive form of severe combined immunodeficiency (SCID) with findings of failure to thrive, erythroderma, scaling, chronic diarrhea, lymphadenopathy, and hepatosplenomegaly. Leiner syndrome can present with failure to thrive, immunodeficiency and seborrheic dermatitis. Wiskott-Aldrich syndrome is an x-linked recessive condition with mutations of the WAS gene. Presentation includes atopic dermatitis, thrombocytopenia, recurrent bacterial infection, lymphoreticular malignancy with non-Hodgkin's lymphoma being the most common, and increased IgA, D and E. With the characteristic hair changes, atopic dermatitis alone is not the most likely diagnosis 252) Patients with hemochromatosis are at increased risk for which of the following? A. Polyarthritis B. Yersenia infections C. Vibrio vulnificus infections D. Generalized metallic-grey hyperpigmentation E. All of these options are correctCorrect Choice Patients with hemochromatosis have increased intestinal iron absorption leading to systemic iron overload. Signs inclued a generalized metallic-grey hyperpigmentation, koilonychia, alopecia (especially pubic/axillary hair) cardiac failure/arrythmias/heart block, hepatomegaly with crrhosis, diabetes (bronze diabetes), polyarthritis with chondrocalcinosis and are susceptible to Vibrio vulnificus and Yersinia infections 83
  • 38. 253) The diagnostic test for chronic granulomatous disease is: A. Bone marrow biopsy B. Histamine skin test C. Potassium hydroxide D. Nitroblue tetrazolium reduction assayCorrect Choice E. Dimethylglyoxime test Chronic granulomatous disease is characterized by a defect in the ability to kill catalase positive organisms within phagocytic leukocytes. This results from a neutrophilic defect in the cytochrome found in the NADPH oxidative pathway responsible for a respiratory burst. The nitroblue tetrazolium (NBT) reduction assay demonstrates the leukocyes’ ability to reduce the dye and produce a blue color change. Patients with chronic granulomatous disease are unable to redue the dye 254) A 50 year man presents with generalized metallic-grey hyperpigmentation. His past medical history includes diabetes, hepatomegaly and arrythmias. Laboratory tests should include: A. Iron levelsCorrect Choice B. Arsenic levels C. Copper levels D. Lead levels E. Cyanide levels Hemochromatosis is an autosomal recessive disease resulting in increased intestinal iron absorption and iron deposition in a variety of organs. Clinical features include generalized metallic-grey hyperpigmentation, koilonychia, sparse or absent hair, hepatomegaly, cardiac failure/arrhythmias, insulin-dependent diabetes, hypogonadism and polyarthritis 255) Which of the following conditions is worsened by ingestion of lithium? A. Darier’s DiseaseCorrect Choice B. Hereditary lymphedema (Nonne-Milroy disease) C. Hailey-Hailey Disease D. Haim-Munk syndrome E. Epidermolytic hyperkeratosis Patients with Darier’s disease should not be treated with lithium due to its worsening or in some cases unmasking the disease. The mechanism for this is not known. Lithium treatment does not worsen the other listed conditions 256) Which of the following is caused by a defect in keratins 4 & 13? A. Epidermolysis bullosa simplex B. Epidermolysis bullosa simplex with myotonic dystrophy 84
  • 39. C. Junctional EB with pyloric atresia D. White sponge nevusCorrect Choice E. Clouston’s syndrome White sponge nevus is caused by a defect in keratins 4 & 13. The remaining entities have the corresponding defects: EB simplex—keratins 5 & 14 EB simplex with myotonic dystrophy—plectin Junctional EB with pyloric atresia—Integrin ?-6, ?4 Clouston’s syndrome (hidrotic ectodermal dysplasia)—connexin 30. 257) The main cause of death in patients with dyskeratosis congenita is which of the following? A. Leukemia B. Pancytopenia Correct Choice C. Oral squamous cell carcinoma D. Atherosclerotic heart disease E. Renal cell carcinoma Dyskeratosis congenita is usually inherited in an X-recessive fashion due to mutations in the dyskerin gene, which is involved in ribosomal RNA synthesis. The less common autosomal dominant form is caused by mutations in the telomerase gene. Clinically, there is reticulated pigmentation of skin, poikiloderma, alopecia, nail atrophy, premalignant oral leukoplakia, and Fanconi-type pancytopenia resulting in early death 258) Which of the following is NOT a characteristic skin finding in patients with Down Syndrome? A. Elastosis perforans serpiginosa B. Syringomas C. Flat nipples D. Single palmar crease E. Small tongueCorrect Choice Down syndrome is caused by nondisjunction and results in trisomy 21. Clinical features include single palmar crease, flat nipples, increased nuchal skin folds, syringomas, elastosis perforans serpiginosa, xerosis, epicanthic folds of eyes, protruding scrotal tongue and fissured thickened lips. 259) Which vascular disorder is characterized by facial vascular malformation and ipsilateral intracranial and retinal arteriovenous malformations(AVMs)? A. Encephalotrigeminal angiomatosis B. Von Lohuizen’s disease C. PHACES D. Bonnet Dechaune Blanc syndromeCorrect Choice E. Sturge-Weber syndome 85
  • 40. Bonnet Bechaune Blanc syndome, also know as Wyburn-Mason syndrome, is characterized by a facial vascular malformation and ipsilateral intracranial and retinal AVMs. Encephalotrigeminal angiomatosis is another name for Sturge-Weber. Von Lohuizen's disease is another name for cutis marmorata telangiectatica congenita 260) The syndrome characterized by generalized mild hyperkeratosis, erythematous keratotic plaques, palmoplantar keratoderma, non-progressive sensorineural deafness, progressive bilateral keratitis with secondary blindness is: A. Vohwinkel syndrome B. CHILD syndrome C. KID syndromeCorrect Choice D. Refsum syndrome E. Erythrokeratoderma variabilis KID syndrome is described above. It is an autosomal dominant mutation in connexin 26. Vohwinkel syndrome is also a connexin 26 mutation, but is characterized by diffuse honeycombed palmoplantar keratoderma, pseudoainhum, starfish-shaped keratotic plaques over joints and deafness. Erythrokeratoderma variabilis is an autosomal dominant mutation in connexin 31 and 30.3 characterized by erythematous migratory patches, fixed hyperkeratotic plaques and a palmoplantar keratoderma. CHILD syndrome is an X-linked dominant mutation condition due to a mutation in NAD(P)H Steroid dehydrogenas-like protein, lethal in males. Unilateral ichthyosiform erythroderma, limb/visceral hypoplasias are characteristic. Refsum syndrome is an autosomal recessive condition with a mutation in phytanoyl coenzyme A hydroxylase characterized by mild ichthyosis, cerebellar ataxia, peripheral neuropathy, retinitis pigmentosa (salt & pepper) and deafness 261) The most common cardiovascular defect in patients with Noonan syndrome is: A. Ventricular septal defect B. Aortic stenosis C. Atrial septal defect D. Enlarged aorta E. Pulmonic valve stenosisCorrect Choice Noonan’s syndrome is also known as cardiofaciocutaneous syndrome. Patients have short stature, ptosis, hypertelorism, low-set ears, thick lips and curly hair. Pulmonic valve stenosis is the most common cardiovascular defect, with atrial septal defects also common 262) What nail change is seen in patients with Mal de Meleda Syndrome? A. leukonychia B. pterygium C. KoilonychiaCorrect Choice D. Longitudinal ridging E. Onycholysis 86
  • 41. Mal de Meleda is an autosomal recessive disease characterized by transgedient malodorous PPK, hyperhidrosis, keratotic plaques at knees and elbows, subungual hyperkeratosis, and koilonychia. The gene defect is SLURP 1 263) Which of the following laboratory test might prove useful in the diagnosis of Fabry disease: A. Stool guaiac B. Bleeding time C. Fasting lipids D. Urinary sediment exam with polarizing light microscopyCorrect Choice E. Complete blood count with differential Patients with Fabry disease have a defect in the alpha-galactosidase A enzyme, leading to an accumulation of glycosphingolipids in all tissues. Although patients are at increased risk for myocardial infrctions and strokes, the serum lipid levels are normal. Ischemic events occur as a result of glycosphingolipid accumulation in endothelial cells leading to swelling. In the brain, strokes occur from direct vessel occlusion or stretching and distention of branches of dolichoectatic parent vessels. Deposits in the kidneys leads to progressive renal failure with urine exam exhibiting proteinuria and birefringent lipid globules (“maltese crosses”) seen with polarizing light microscopy 264) The x-linked recessive type of dyskeratosis congenita is: A. PTEN B. dyskerinCorrect Choice C. CDKN2A D. Menin E. TERC The dyskerin gene, whose product is involved in ribosomal RNA synthesis, is mutated in X-linked recessive dyskeratosis congenita. TERC is linked with autosomal domininant transmission of the syndrome. CDKN2A is involved in familial dysplastic nevi/melanoma syndrome, PTEN in Cowden syndrome and Menin in MEN type I 265) Which of the following conditions is inherited in an X-linked recessive manner? A. Sjogren-Larsson syndrome B. Ichthyosis vulgaris C. Wiskott-Aldrich SyndromeCorrect Choice D. Epidermolysis bullosa simplex E. Netherton’s Syndrome Wiskott-Aldrich Syndrome is inherited in an X-linked recessive manner. Epidermolysis bullosa simplex and ichthyosis vulgaris are inherited in an autosomal dominant (AD) manner. Sjogren- Larsson and Netherton’s syndrome are inherited in an autosomal recessive manner 87
  • 42. 266) What is this syndrome which is histologically characterized by widely dispersed granular material amidst normal fibers? A. Focal Dermal Hypoplasia B. Pseudoxanthoma ElasticumCorrect Choice C. Buschke-Ollendorf Syndrome D. Lipoid Proteinosis E. Ehlers Danlos Syndrome Pseudoxanthoma elasticum is genodermatosis characterized by redundant skin, angioid streaks, yellow papules on the mucous membranes and bleeding from gastric artery. On histology, readily apparent denerative changes of the elastic fibers are prominent, even without special stains 267) The most common neoplasm seen in Maffucci Syndrome is: A. EnchondromasCorrect Choice B. Angiosarcomas C. chondrosarcoma D. Lymphangiosarcomas E. Osteosarcomas Maffucci syndrome comprises of superficial and deep venous malformations, enchondromas, and short stature. Enchondromas are the most common neoplasm, while chondrosarcomas are the most common malignancies 268) Which of the following syndromes is X-linked dominant? A. X-linked icthyosis B. anhidrotic ectodermal dysplasia C. dyskeratosis congenita D. Menkes kinky hair syndrome E. orofaciodigital syndrome 1Correct Choice Orofaciodigital sydrome 1 is an X-linked dominantly inherited disorder caused by a defect in the CXORF5 gene. The rest of the above conditions are inherited in an x-linked recessive pattern. 269) Which malignancy is seen in approximately 15-20% of people with the disease characterized by a defect in a parathyroid hormone receptor protein? A. osteosarcoma B. chondrosarcomaCorrect Choice C. angiosarcoma D. rhabdomyosarcoma E. epitheliod sarcoma 88
  • 43. Approximately 15-20% of patients with Maffucci's syndrome will develop chondrosarcoma. Maffucci's syndrome is due to a defect in a parathyroid hormone receptor protein 270) Medulloblastoma is seen in which syndrome? A. Basal cell nevus syndromeCorrect Choice B. Gardner’s syndrome C. Multiple endocrine neoplasia 2b D. Muir-Torre syndrome E. Neurofibromatosis Type 1 Basal cell nevus syndrome is an autosomal dominant syndrome caused by a mutation in PTC gene, which acts in the Sonic hedgehog pathway. Cutaneous manifestations of this genodermatosis include basal cell carcinomas, palmoplantar pits, epidermoids cysts. Other findings include odotogenic cysts, frontal bossing, bifid ribs, calcification of the falx cerebri and medulloblastomas 271) A 2 year old girl presents with sunken eyes, large ears, microcephaly and a photodistributed eruption on her face. Eye exam reveals “salt and pepper” retina. The gene responsible for this syndrome codes for a: A. Transcription factor B. DNA helicaseCorrect Choice C. Lysosomal protease D. Surface glycoprotein E. Mismatch repair gene The patient described has Cockayne syndrome, an autosomal recessive disorder believed to be due to a mutation in either DNA helicase or defective nucleotide excision repair. UV irradiated cells have decreased DNA and RNA synthesis and increased chromosomal breaks. Clinical features include cachectic dwarfism with microcephaly, thin nose, large ears, photosensitive eruption, cataracts, salt & pepper retina, and diffuse demyelination 272) Which of the following findings is characteristic of a mutation in lamin A? A. Lipoatrophic sclerodermoid skin B. All of the answers are correctCorrect Choice C. Craniomegaly with small face D. Severe premature atherosclerosis with early death E. Alopecia A mutation in Lamin A causes Progeria (Hutchinson-Gilford syndrome). Other findings include nail atrophy and muscle/bone wasting. Presentation is in the first or second year of life. An increased urine hyaluronic acid can be helpful in diagnosis 273) Which disease is found more commonly in mothers of patients with chronic granulomatous disease? 89
  • 44. A. Erythema nodosum B. Sarcoidosis C. Discoid lupus erythematousCorrect Choice D. Wegener's disease E. Churg-Straus disease Female carriers of chronic granulomatous disease have an increase incidence of discoid lupus, infections and apthous stomatitis 274) Osteopathia striata is seen in which of the following disorders? A. Aplasia cutis congenita B. Scleroderma C. Focal dermal hypoplasia Correct Choice D. Albright’s syndrome E. Gaucher’s disease Focal dermal hypoplasia (Goltz syndrome) is an X-linked dominant disorder that is lethal in males. There is linear atrophy following Blaschko’s lines with areas of fat herniation with underlying osteopathia striata, which is radiologically characterized by linear bony hyperdensity. Other features include mucocutaneous papillomas and pits, alopecia, nail dystrophy, tooth abnormalities, and colobomas 275) Xeroderma pigmentosum (XP) variant is different than classic XP in which of the following way? A. Increased chromosomal breakage and sister chromatid exchanges B. Defective DNA nucleotide excision repair of the global genome C. Defective DNA nucleotide excision repair of actively transcribing genes D. Defective post-replication repairCorrect Choice E. Low IgM XP variant is DNA nucleotide excision repair proficient, but the defect is in post replication repair of DNA. Increased chromosomal breakage and sister chromatid exchanges is found in Bloom’s syndrome, an autosomal recessive syndrome caused by a defect in BLM gene, whose product functions as a helicase. Clinical findings include: Telangiectasias, short stature, malar erythema, recurrent infection, increased frequency of leukemia and lymphoma, normal intelligence. Defective DNA nucleotide excision repair of actively transcribing genes is a feature of Cockayne’s syndrome, an autosomal recessive syndrome with clinical findings including: Cachexia, short stature, pigmentary retinal degeneration, progressive deafness and no increase in neoplasms. Xeroderma pigmentosum has seven different complementation groups (A-G), each associated with a different form of impairment of DNA nucleotide excision repair 276) A child presents with sparse, short hair and sensorineural deafness. On microscopic examination of the hair, pili torti is noted. Which of the following syndromes is the most likely diagnosis? A. None of the options are correct 90
  • 45. B. Menkes kinky hair syndrome C. Bjornstad syndromeCorrect Choice D. Argininosuccinic aciduria E. Trichothiodystrophy Bjornstad syndrome is the most likely diagnosis. This rare syndrome (~25 cases) is autosomal recessive. Findings are of deafness and pili torti. The most common hair finding in Menkes syndrome is pili torti, but it is not associated with hearing loss. Argininosuccinic aciduria is associated with trichorrhexis nodosa and has no associated hearing loss 277) Which genetic defect could explain these cutaneous findings in addition to abnormal immunoglobulin levels, recurrent respiratory infections, hypogonadism, and an increased risk of leukemia and lymphoma? A. Adenosine deaminase B. ERCC6 C. WAS gene D. RecQL3Correct Choice E. NADPH oxidase Bloom's syndrome is an autosomal recessive disorder caused by mutations in the RecQL3 gene encoding a DNA helicase. Clinically, individuals with Bloom's syndrome have a photodistributed erythema with telangectasia on the malar eminences. The may also have decreased IgM and IgA levels, hypogonadism, and an increased risk for leukemia and lymphoma 278) A patient with multiple lentigines and blue nevi may also have: A. mental retardation B. deafness C. atrial myxomaCorrect Choice D. pulmonary valve stenosis E. GI malignancy This patient may have a constellation of features associated with NAME syndrome, otherwise known as Carney complex or LAMB syndrome. This condition is inherited in an autosomal dominant pattern and is due to a defect in the PRKAR1A gene. This condition is characterized by the following features: blue Nevi, Atrial myxomas, cutaneous Myxomas, and Ephelides. In addition, testicular tumors are seen as well as sexual precocity. Finally, patients may have endocrine abnormalities including pigmented nodular adrenocorticoal disease and Cushing syndrome as well as pituitary adenomas. Deafness, pulmonary stenosis, GI malignancies, and mental retardation are not features of this condition. (Spitz's Genodermatoses. 1996, p70-71) (Abdelmalek, N. J Am Acad Dermatol 2002;46:161-183). 279) Ehlers-Danlos Syndrome with congenital adrenal hyperplasia is caused by mutations affecting which of the following? 91
  • 46. A. Fibronectin B. Lysyl oxidase C. Lysyl hydroxylase D. Tenascin-X Correct Choice E. Collagen 5 Ehlers-Danlos Syndrome type with congenital adrenal hyperplasia is caused by mutations in tenascin-X 280) Which of the following diseases is seen only in females? A. Hypomelanosis of Ito B. Carney complex C. Piebaldism D. Griscelli syndrome E. Incontinentia pigmenti Correct Choice Incontinentia pigmenti (Bloch-Sulzberger syndrome) is an X-linked dominant disease that is lethal in males. There are four stages: vesicular, verrucous, hyperpigmented and hypopigmented. Also seen are peg and conical teeth, eye abnormalities, CNS defects, and alopecia. This condition is caused by mutations in the NEMO gene 281) Which of the following is caused by a defect in cathepsin C? A. Muir-Torre B. Nail-Patella syndrome C. Rubenstein-Taybi D. Papillon-LefevreCorrect Choice E. Bullous Icthyosis of Siemens Papillon-Lefevre is a palmoplantar keratoderma caused by a mutation in chromosome 11q14, leading to a defect in Cathepsin C, a lysosomal enzyme. Clinical manifestations of papillon lefevre include sharply demarcated palmoplantar keratoderma with extension to dorsal surface (transgrediens), spare hair, periodontitis, and pyoderma 282) Which of the following syndromes is characterized by follicular atrophoderma, hypohidrosis, hypotrichosis and multiple basal cell carcinomas? A. Bazex syndromeCorrect Choice B. Rombo syndrome C. Incontinentia Pigmenti D. Rasmusen syndrome E. Gorlin syndrome Bazex syndrome has the findings of follicular atrophoderma, hypohidrosis, hypotrichosis and multiple basal cell carcinomas (BCC). Rombo syndrome is associated with BCC and hypotrichosis, 92
  • 47. but not the other listed findings. The atrophoderma in Rombo syndrome is vermicular, not follicular. Rasmusen syndrome is not associated with BCC 283) Patients with x-linked icthyosis are more prone to getting which two malignancies? A. Renal cell and ALL B. Testicular and AML C. Pancreatic and acute myelogenous leukemia (AML) D. Testicular and ALLCorrect Choice E. Pancreatic and acute lymphocytic leukemia (ALL) Patients with x-linked icthyosis have a 20% chance of having cryptorchidism and are more prone to both testicular cancer and acute lymphocytic leukemia 284) Odontogenic cysts and palmoplantar pits are seen in: A. Bloom’s Syndrome B. Refsum syndrome C. Gorlin SyndromeCorrect Choice D. Gardner’s syndrome E. Goltz Syndrome Gorlin syndrome (Basal Cell Nevus Syndrome)is an autosomal recessive disorder due to a defect in the PTCH gene whose function normally inhibits “SMOOTHENED” signaling. Odontogenic cysts and palmoplantar pits are characteristic features, in addition to multiple basal cell carcinomas. Other features include: frontal bossing, kyphoscoliosis, calcification of falx cerebri, hypertelorism, ovarian fibromas and rarely mental retardation 285) Comma shaped corneal opacities are seen in what disease? A. Refsum Syndrome B. Proteus syndrome C. Sjogren-Larson Syndrome D. X-linked ichthyosisCorrect Choice E. Pseudoxanthoma elasticum X-linked ichthyosis is a X-linked recessive disorder secondary to steroid sulfatase deficiency characterized by brown adherent scale. Additional findings include comma-shaped corneal opacities, cryptorchisdism, and failure to progress during labor. 286) Which of the following is caused by a mutation in a gene that leads to defective NF-KB activation? A. Vohwinkel’s B. MEN IIa 93